P5-Obgyne

Réussis tes devoirs et examens dès maintenant avec Quizwiz!

Salpingectomy done what to do when discharging the patient? A. Weekly βHCG

: A. Toronto Notes 2020, P516 36

Case asked about causes of these contractions with CTG picture and BP 90/70? A. Epidural anesthesia B. Oxytocin C. Mg Sulfate

: ? A prolonged deceleration is defined as a deceleration that lasts more than 3 minutes: If it lasts between 2-3 minutes it is classed as non-reassuring. If it lasts longer than 3 minutes it is immediately classed as abnormal Prolonged deceleration with maternal hypotension, normal contraction → epidural Prolonged deceleration with maternal hypotension, hypercontraction → Oxytocin 39

Down syndrome what will you find ? A. High βHCG B. High inhibin

: A Note: trisomy 21 findings are Low AFP

Pregnant lady. Effacement 70%. Fully dilated. Position -3. Management: A. Vacuum suction B. Forceps C. CS

: A or C? C: 2nd stage of labor A: Observe or Vacuum, if fetal head obvious >>forceps B4 said it is C. 22

Woman with cystocele features . What next step? A. Vaginal exam and speculum B. Hysteroscopy C. Laparoscopy

: A. 18

Abruptio placenta management: A. Multidisciplinary team

: A. 20

Pregnant with recurrent UTI what to do? A. Septic workup and treatment

: A. 49

What is the immunoglobulin that predominant in breast milk? A. IgA B. IgM C. IgG

: A. 50

CTG, late decelerations. Asking for diagnosis? A. Uteroplacental insufficiency

: A. Toronto Notes 2020, P880 43

What prevents variceal bleeding and its recurrence ? A. Nadolol

: A. - Beta blockers such as propranolol will not do anything for the current episode of bleeding. Propranolol or nadolol is used to prevent subsequent episodes of bleeding. - Octreotide (somatostatin) to decreases portal pressure. - Banding performed by endoscopy obliterates esophageal varices. - Transjugular intrahepatic portosystemic shunting (TIPS) is used to decrease portal pressure in those who are not controlled by octreotide and banding. - Antibiotics to prevent spontaneous bacterial peritonitis with ascites. Master the Boards USMLE Step 2 CK - 5th Ed, P61 4

Other scenario where she can't reach hospital, size 3cm , βHCG 5000 what to do? A. Surgical B. Admit and medical C. Appointment

: A. 3.5 cm or larger Ectopic is exclusion criteria for Methotrexate. The larger the ectopic, the greater the risk of treatment failure with methotrexate. Surgery is done to try to preserve the fallopian tube by cutting a hole in it(salpingostomy). However, removal of the whole fallopian tube (salpingectomy) may be necessary. patient may require laparotomy if unstable. Master the Boards USMLE Step 2 CK - 5th Ed, P900. 37

HIV pregnant woman. What is correct statement? A. Advise to do CS to decrease chance of transmission B. Breastfeeding will not harm the baby C. Taking HIV drugs during pregnancy is useless

: A. AZT or nevirapine in pregnant women with HIV

Mother HIV, what to do to decrease fetal infection? A. Tell mother CS may decrease risk of baby infection

: A. AZT or nevirapine in pregnant women with HIV

Girl with dysmenorrhea and has some days absence from school due to the symptoms. What is the treatment? A. NSAIDs B. OCP C. SSRI

: A. Above table, Toronto Notes 2020, P506 26

Menopause present with vaginal bleeding (fresh blood). Next? A. Endometrial biopsy B. D&C C. Nothing to do

: A. Always menopause with vaginal bleeding → do Biopsy True , but for Severe bleeding management → IV estrogen 28

Asymptomatic patient with irregular cervical polyps, your next step? A. Polyps excision at the clinic B. Pap smear C. HBV

: A. BENIGN CERVICAL LESIONS • Nabothian cyst/inclusion cyst: no treatment required • endocervical polyps: treatment is polypectomy (office procedure) Toronto Notes 2020, P538 7

PCOS (patient have PCOS not taking medication cause of endometrial cancer? A. Unopposed estrogen

: A. Hacker & Moore's Ess. of OB&GYN.Ch 33. 33

Valvular carcinoma in situ management: A. Wide local excision

: A. Hacker & Moore's Ess. of OB&GYN.Ch 40. 32

It should be Estrogen only 81. Patient with bilateral breast tenderness. Everything is normal except high prolactin. What is your radiological CT assessment? A. Lumbar spine B. Chest C. Pelvis D. Rulitica cacitica (something like that??) If there was sella turcica chose it. Search sellar mass 82. What is the commonest site of fibroid? A. Intramural B. Submural C. Subserosal

: A. Hacker & Moore's Ess. of OB&GYN.Ch19 83. Vaginal dryness → estrogen cream When the patient's main concerns are with genitourinary symptoms, vaginal estrogen cream, tablets, or rings may be used on an "as needed" basis without necessarily adding a progestin Hacker & Moore's Ess. of OB&GYN.Ch35 84. Endometrial polyp diagnosis? A. Laparoscopic B. Hysteroscopy A,: B. They can be more clearly recognized on saline infusion sonography (SIS) or visualized directly by hysteroscopy. Hacker & Moore's Ess. of OB&GYN.Ch19 85. 15 year old girl brought by her mother due to the concern of her daughter's primary amenorrhea and short stature. Upon examination, she is short with short neck and midline hair and hypertension. Her parents are also short. What is the most likely diagnosis? A. Familial short stature B. Turner syndrome C. Hypothyroidism A,: B. First Aid USMLE Step 2 CK - 10th Ed, P395 86. US of pregnant woman gestational age 13 weeks with browning discharge and closed os: • Missed abortion (if no cardiac activity) • Threatened abortion (if viable fetus) 87. Uterine atony what to give? A. Oxytocin A,: A. Most common cause of PPH (70-80%). Avoid with active management of 3rd stage of labour with 1) oxytocin administration 2) uterine massage 3) umbilical cord traction. Toronto Notes 2020.P889 88. A few questions about eclampsia. how to manage the high blood pressure. Toronto Notes 2020.P870 First Aid USMLE Step 2 CK - 10th Ed, P335 89. Pregnant post fall developed uterine tenderness and vaginal bleeding. Her pregnancy history is unremarkable of any disease. She is a smoker: A. Placenta previa B. Placental abruption A,: B. Toronto Notes 2020.P860 90. Pregnant with sac without embryo picture admits for procedure. What is the most common complication that may happen from the procedure? A. Uterine rupture B. Amniotic embolism A,: A (the suggested A, was B) An ultrasound will show an empty gestational sac. Some women choose to wait for the miscarriage to happen naturally, while others take medication to trigger the miscarriage. In some cases, a procedure called dilation and curettage (D&C) is used to remove the placental tissues. Uterine rupture is a complication of D&C. Others: Hemorrhage, infection, and cervical laceration. Hacker & Moore's Ess. of OB&GYN. If hx suggest? approximately 2 percent of patients treated by evacuation develop cardiopulmonary symptoms, including chest pain, dyspnea, tachypnea, and tachycardia. Respiratory distress after uterine evacuation is usually attributed to trophoblastic embolization. UpToDate 91. Patient after total abdominal hysterectomy with bilateral salpingo-oophorectomy (TAHBSO) developed uterine leak: A. Ureterovaginal fistula B. Urethrovaginal fistula C. Vesicovaginal fistula A,: Depends: • During urination: urethrovaginal fistula • Continuous: vesicovaginal fistula Hacker & Moore's Ess. of OB&GYN. 92. Pregnant woman came with signs of domestic abuse. What is the most common time of abuse from the spouse? A. Pregnancy B. After retirement C. After family visit A,: A. UpToDate. 93. Valvular lesion, round, well defined patch, scales, suspect psoriasis, ask about how to confirm the diagnosis. A. Clinical diagnosis B. Biopsy A,: A. Hacker & Moore's Ess. of OB&GYN 94. Most common cause of intermenstrual bleeding in a 13 year old. A. Ovulation B. Bleeding A,: A. UpToDate. 95. Patient came with suspicious lesion on the cervix. What to do? A. Chest CT with contrast B. PET whole body C. Wide local excision A,: C Hacker & Moore's Ess. of OB&GYN 96. Primigravida at 29 weeks of gestation felt some chest pain. CXR showed large heart and then she died. Diagnosis? A. Peripartum cardiomyopathy B. Edema C. Embolism A,: A. It is unknown why there are antibodies made against the myocardium in some pregnant women. The LV dysfunction is often reversible and short term. If the LV dysfunction does not improve, then the person must undergo cardiac transplantation. Repeat pregnancy in a woman with peripartum cardiomyopathy will provoke enormous antibody production against the myocardium. The medical therapy consists of the same drugs as used for dilated cardiomyopathy of any cause. Master the Boards USMLE Step 2 CK - 5th Ed, P222 97. Pic of GTD going to do suction and evacuation. What possible complication? A. Amniotic embolism B. Perforation C. Hemorrhage A,: B ( A is possible ) suction evacuation followed by sharp curettage of the uterine cavity, regardless of the duration of pregnancy. This technique is associated with a low incidence of uterine perforation and trophoblastic embolization. Hacker & Moore's Ess. of OB&GYN.Ch42 98. Smoker female who is pregnant. She had a history of subfertility and a family history of diabetes mellitus. Which factor indicates high risk? A. Smoking B. History of subfertility C. Family history of DM A,: A. 99. Pregnant at 39 weeks came for regular follow up and has higher blood pressure (I am sure systolic was 150) with a pic of urine dipstick showing proteinuria and I think infection. What is appropriate management? A. Admit for observation B. Induction of labor by oxytocin C. Immediate CS D. Home discharge A,: B. Hacker & Moore's Ess. of OB&GYN. 100. Pregnant with 2cm dilatation 50% effacement -2 station and I think mid posterior position: A. Induction of labor with IV oxytocin B. Artificial ROM C. CS A,: B. Prostaglandin if present 101. The scenario of female in 13 years of age and has 9 month amenorrhea and a lot of investigation were done and all was normal except high FSH and LH. This lady has a risk for: A. Endometrial cancer B. Multiple gestations C. Ovarian cancer A,: A. Assuming she has PCOS LH hypersecretion increases serum LH levels in up to 70% of patients with PCOS, with elevated LH pulse amplitude and frequency inducing a two- to threefold elevation in circulating LH over FSH levels. Increased LH pulse frequency in PCOS, from enhanced hypothalamic GnRH pulsatile release, occurs as the result of reduced steroid hormone negative feedback on LH secretion from hyperandrogenism. Hacker & Moore's Ess. of OB&GYN.Ch33 102. Case of 7 week pregnant with bleeding, the passage of tissue, vital was stable, by examination you find blood and tissue and uterine size between 7-8 week adnexa was free and normal: A. Incomplete abortion B. Complete mole C. Inevitable abortion A,: A. First Aid USMLE Step 2 CK - 10th Ed, P325 103. Lactating woman with positive E.coli what to give? A. Nitro B. Cipro C. TMP A,: A. A&C contraindicated in G6PD. uncomplicated UTI ■ first line: amoxicillin (250-500 mg PO q8h x 7 d) ■ alternatives: nitrofurantoin (100 mg PO bid x 7 d) or cephalosporins ■ follow with monthly urine cultures • pyelonephritis ■ hospitalization and IV antibiotics Toronto Notes 2020.P874 104. Patient taking paroxetine for depression know she is pregnant. A. Discontinue due to risk of IUGR B. Continue the same C. Discontinue due to risk of (cardiac disorders) D. Continue and add lithium A,: B. UpToDate 105. 28-year-old married female complains of RLQ pain and drowsiness, on exam abdominal \ tenderness what's the diagnosis? A. Rupture of ovarian cyst B. Ectopic pregnancy A,: most likely B The clinical presentation of tubal ectopic pregnancies can vary from subtle lower abdominal discomfort and light uterine spotting to symptoms consistent with hypovolemic shock due to massive internal hemorrhage from tubal rupture. Hacker & Moore's Ess. of OB&GYN.Ch24 106. Advanced ovarian epithelial cancer. What's the treatment? A. Surgery B. Chemo C. Surgery then chemo D. Surgery then radio A,: C. Toronto Notes 2020.P536 107. Female with a history of recurrent abortions and bilateral leg thrombosis. aPTT is high. What is the diagnosis? A. Antiphospholipid syndrome B. Antithrombin deficiency C. Protein (something) A,: A. Recurrent SAB: ■ Late (12-20 weeks): Hypercoagulable states (eg, antiphospholipid syndrome, SLE, factor V Leiden, protein S deficiency). First Aid USMLE Step 2 CK - 10th Ed, P324 108. Increased aPTT and PT. Fetus is dead inside his mother's womb (no fetal movement). Mother developed respiratory distress. What is the diagnosis? A. DIC B. Amniotic embolism A,: A. If the fibrinogen level is decreasing, it may be an early sign of consumptive coagulopathy in cases of fetal demise. An elevated prothrombin and partial thromboplastin time, the presence of Fibrinogen-fibrin degradation products, and a decreased platelet count may clarify the diagnosis. Hacker & Moore's Ess. of OB&GYN.Ch12 109. Pregnant in the third trimester. Painful bleeding, (check BP could be a hint): A. Red degeneration B. Abruptio placentae A,: B. Toronto Notes 2020.P860 110. Safe Vaccine in pregnant: A. OPV B. Hep B C. Varicella A,: B. Toronto Notes 2020.P857 111. Missed pregnancy. How to deal with it pic: A. Follow up after 4 weeks B. Admission and observation A,: ? Manual uterine aspiration (MUA) if < 12 weeks or D&C

Lady with mitral stenosis , asthma and herpes the doctor did forceps delivery. Why did the doctor do forceps ? A. Because of the mitral stenosis

: A. Indications of operative delivery: - Maternal ■ need to avoid voluntary expulsive effort (e.g. cardiac/cerebrovascular disease) ■ exhaustion, lack of cooperation, and excessive analgesia may impair pushing effort - Fetal: ■ atypical or abnormal fetal heart rate tracing, evidence of fetal compromise ■ consider if second stage is prolonged, as this may be due to poor contractions or failure of fetal head to rotate Toronto Notes 2020, P887 3

Normal CTG → but patient have minimal vaginal bleeding what to do? A. Admit B. Appointment C. Surgery

: A. N.B: Admit & observe for 24 hours 38

may also use misoprostol or expectant management in inevitable and missed SAB. First Aid USMLE Step 2 CK - 10th Ed, P325 78. Pregnant lady in the last trimester. She had pneumonia. What is the type of immunization for her fetus? A. Passive natural B. Active natural C. Passive secondary D. Natural secondary

: A. Passive immunity : transplacental immune globulin (Ig)G antibody transfer from vaccinated mother to fetus. 79. 17-hydroxyprogesterone in patient with congenital adrenal hyperplasia. What enzyme causes this? A. 21-hydroxylase deficiency A,: A. 21-hydroxylase deficiency: Accounts for ∼ 90% of CAH cases

Woman prior to menses had abdominal pain irritability and can't do usual activity all these resolve in third day on menses? What is the management? A. SSRI

: A. Premenstrual Syndrome Physiological and emotional disturbances that occur 1-2 wk prior to menses and last until a few days after onset of menses

Fishy odor with white yellow discharge, diagnosis? A. Bacterial vaginosis B. Candida C. Trichomonas

: A. Table below First Aid USMLE Step 2 CK - 10th Ed, P371 53

Case of trichomonas vaginalis with flagella, what is the treatment? A. Metronidazole B. Fluconazole cream

: A. Table below First Aid USMLE Step 2 CK - 10th Ed, P371 55. When to start screening for osteoporosis for women? A. 65 years A,: A. Dual-energy x-ray absorptiometry (DEXA). Recommended as screening test for all women > 65 years of age and men > 70 years of age, and those with other risk factors for osteoporosis. First Aid USMLE Step 2 CK - 10th Ed, P103 56. Pregnant episiotomy until rectum what degree of tear? A. 1st degree B. 2nd degree C. 3rd degree D. 4th degree A,: D. incision in the perineal body at the time of delivery. Essentially a controlled second-degree laceration. Midline: incision through central tendinous portion of perineal body and insertions of superficial transverse perineal and bulbocavernosus muscles. It heals better, but increases risk of 3rd/4th degree tears. Mediolateral: incision through bulbocavernosus, superficial transverse perineal muscle, and levator ani. Reduced risk of extensive tear but more painful Extension to the anal sphincter (third degree) or rectum (fourth degree) First Aid USMLE Step 2 CK - 10th Ed, P349 57. Pregnant 37 weeks came with IUFD, feeling guilty because she smoke 5 cigarettes / day, what to tell her? A. Smoking is not related to IUFD B. Smoking increase rate of IUFD by 10 times C. She should stop smoking in next pregnancy D. Cause is not known yet and she needs more investigation A,: C. Assist/encourage to reduce or quit smoking. - increased risk of decreased birth weight, placenta previa/abruption, spontaneous abortion, preterm labour, and stillbirth. Toronto Notes 2020. 58. Nursing mother developed breast pain, wound hotness what organism? A. staph aureus A,: A. Toronto Notes 2020.P892 59. case of patient not pregnant with heavy vaginal bleeding. What's your appropriate management? A. Medroxyprogesterone B. Blood transfusion C. Conjugated estrogen A,: C First Aid USMLE Step 2 CK - 10th Ed, P364 60. 32 or 33 gestational week came with PROM, appropriate next step: A. Tocolysis B. Antibiotics C. Steroid D. Surfactant A,: B. Admit for expectant management and monitor vitals q4h, daily NST, WBC count, increased surveillance. Avoid introducing infection by minimizing examinations ■ consider administration of betamethasone valerate (Celestone®) to accelerate maturity if <34 weeks if no evidence of infection. ■ consider tocolysis for 48 h to permit administration of steroids if PPROM induces labour. • screen women for UTIs, STIs, GBS infection and treat with appropriate antibiotics if positive (treat GBS at time of labour) • If not in labour or labour not indicated, consider antibiotics: penicillins or macrolide antibiotics are the antibiotics of choice • Deliver urgently if evidence of fetal distress and/or chorioamnionitis. Toronto Notes 2020.P863 61. CTG for baby show FHR 160, baby at risk of: A. Stillbirth B. Shoulder dystocia C. Large baby A,: A? The major risks of fetal tachycardia are hemodynamic compromise, RDS, asphyxia, and meconium aspiration development of hydrops and intrauterine fetal death. 62. Pregnant woman with symptomatic UTI: • Oral antibiotics if cystitis • IV antibiotics if pyelonephritis First Aid USMLE Step 2 CK - 10th Ed, P337 63. Pap smear showed Atypical squamous cells of undetermined significance (ASCUS). What to do next? A. HPV B. Colposcopy A,: A. Or we can repeat pap smear after 6-12 months. First Aid USMLE Step 2 CK 10th Ed, P376 64. When to order brain MRI in patient with hyperprolactinemia? A. Prolactin duple B. Visual disturbance A,: B. Explanation: Generally speaking magnetic resonance imaging (MRI) scan of the pituitary gland is performed once physiological, drug-induced, and secondary hyperprolactinemias have been excluded, or if there are clear symptoms of adenoma. Hacker & Moore's Ess. of OB&GYN.Ch 33, BMJ 65. Case of ectopic pregnancy and patient is hypotensive: A. Surgery A,: A. Surgery is done to try to preserve the fallopian tube salpingostomy. However, salpingectomy may be necessary. patient may require laparotomy if unstable. Master the Boards USMLE Step 2 CK - 5th Ed, P900. 66. Pregnant with vaginal bleeding, during investigation you found suspicious mass in cervix what is the next step? A. Pap smear B. Colposcopy C. Surgical management A,: A. Pregnant women should follow the routine cervical (Pap) screening regimen. Pregnancy is not a contraindication for colposcopy or cervical biopsy. However, endocervical curettage is deferred during pregnancy because of risk for preterm delivery. 2013 Canadian Task Force for Preventive Care Guidelines. if abnormal Pap or suspicious lesion, refer to colposcopy 67. Patient with postcoital bleeding. A. Colposcopy B. Inspect vagina and cervix A,: B? 68. What is the cause of persistent fetal bradycardia? A. Cord prolapse B. Placental insufficiency A,: A. Can be caused by congenital heart malformations or by severe hypoxia (2° to uterine hyperstimulation, cord prolapse, or rapid fetal descent). Hacker & Moore's Ess. of OB&GYN. 69. Case of galactorrhea caused by prolactinoma. Management? A. Cabergoline A,: A. First Aid USMLE Step 2 CK - 10th Ed, P111 70. Patient with signs of MS or hyperprolactinemia (I'm confused). Where will you do the investigation? A. MRI B. Pelvis C. Sitca tutnm D. Liver A,: A. 71. What is the most common cause of leiomyoma in pregnancy? A. Cystic B. Red C. Fatty D. Hyaline A,: B. Hacker & Moore's Ess. of OB&GYN.Ch19 72. Urge incontinence. First initial test? A. Urodynamic study B. Cystourethroscopy C. Urinalysis and culture A,: C. First Aid USMLE Step 2 CK - 10th Ed, P382 73. Most common complication after hysteroscopy? A. Infection B. Vaginal discharge C. Adhesions D. Perforation A,: D Hacker & Moore's Ess. of OB&GYN.Ch31 74. A diabetic pregnant lady. What is the most important complication for the baby? A. Macrosomia B. Shoulder dystocia A,: A. Macrosomia: maternal hyperglycemia leads to fetal hyperinsulinism resulting in accelerated anabolism. Hacker & Moore's Ess. of OB&GYN. 75. A pregnant lady with twins, what is the best position for vaginal delivery? A. Breech-breech B. Breech-cephalic C. Cephalic-cephalic D. Cephalic-breech A,: C. Hacker & Moore's Ess. of OB&GYN. 76. Woman complaining of protruding mass from the vagina, with cough and straining. What is the best initial management? A. Cotinine test B. Examination of vagina with speculum C. Retrograde urethrogram radiology A,: B. Hacker & Moore's Ess. of OB&GYN. 77. Patient with incomplete abortion. What is the initial management? A. IV fluid with surgical evacuation B. IV fluid with expectant management A,: B. Manual uterine aspiration (MUA) if < 12 weeks or D&C

Patient with white yellow discharge and fishy odor, positive Whiff test, with a picture of discharge analysis. What type of cell you will see? A. Epithelial cells B. Pseudohyphae

: A. Table below First Aid USMLE Step 2 CK - 10th Ed, P371 52

"classic" form is most severe and presents as a newborn infant girl with ambiguous genitalia and adrenal insufficiency (with or without life-threatening salt wasting). "Nonclassic" is a late-onset form that presents with androgen excess or could be asymptomatic. Cannot convert 17- hydroxyprogesterone to 11-deoxycortisol → ↓ cortisol synthesis → ↑ adrenal stimulation → ↑ ACTH and androgens. First Aid USMLE Step 2 CK - 10th Ed, P366 80. Patient did bilateral oophorectomy and salpingectomy. What is the type of hormone replacement therapy? A. Levonprogesterone IUD B. OCP C. Progesterone patch

: A. The A, here is Progesterone only (Levonprogesterone) / Mirena If the question was missing a clue about Hysterectomy

Scenario case of MgSo4. Why? A. To prevent seizures

: A. prevention of recurrent convulsions: to prevent further seizures and the possible complications of repeated seizure activity (e.g. rhabdomyolysis, metabolic acidosis, aspiration pneumonitis, etc.) MgSO4 is now the drug of choice. Toronto Notes 2020, P871 23

Pregnant with HF symptoms (SOB, LL edema, lung crepitations) A. Viral myocarditis B. Peripartum cardiomyopathy

: B. It is unknown why there are antibodies made against the myocardium in some pregnant women. The LV dysfunction is often reversible and short term. If the LV dysfunction does not improve, then the person must undergo cardiac transplantation. Repeat pregnancy in a woman with peripartum cardiomyopathy will provoke enormous antibody production against the myocardium. The medical therapy consists of the same drugs as used for dilated cardiomyopathy of any cause. Master the Boards USMLE Step 2 CK - 5th Ed, P222 14

may also use misoprostol or expectant management in inevitable and missed SAB. First Aid USMLE Step 2 CK - 10th Ed, P325 112. What kind of prevention in screening of pregnant lady with HIV? A. Primary B. Secondary C. Tertiary

: B. First Aid USMLE Step 2 CK - 10th Ed, P128 113. What to give for a pregnant lady with gestational diabetes mellitus and failed to control it with diet? A. Insulin A,: A. Patients with GDM are usually managed with diet and exercise alone, but if euglycemia cannot be achieved, an oral hypoglycemic agent or insulin should be added. Insulin is the medication of choice to maintain euglycemia in pregnancy and is the recommended therapy in women with pregestational diabetes. Hacker & Moore's Ess. of OB&GYN.Ch16 114. Pregnant lady suffers from cauliflower lesion. What is the best treatment for her? A. Cryotherapy B. Electrocautery C. Podophyllum resin (teratogenic) A,: A. Toronto Notes 2020.P164&342 115. 7 years old girl, came to clinic with her mom complained of appearance of hair in her pubic area, no breast development, no axillary hair, no adult odor. What's the diagnosis? A. Pernicious puberty? (maybe precocious) B. Early adrenarche C. Adrenal hyperplasia D. Normal puberty A,: B. In African American girls, thelarche and adrenarche can occur as early as 6 years of age, whereas in whites, they can occur as early 7 years of age. Hacker & Moore's Ess. of OB&GYN.Ch32 116. 3 Q scenarios about females with vaginal discharge • Clue cells: bacterial vaginosis • Greenish discharge: trichomoniasis • White discharge with itchiness: vulvovaginal candidiasis First Aid USMLE Step 2 CK - 10th Ed, P371 117. Female treated with psychotherapy for postpartum depression. Which of the following will be helpful? A. Add small dose of B. Encourage her for breastfeeding C. Involve family members in treatment A,: B or C? B4 said C and glory said B 118. Contraindication for IUD is having a history of: A. Previous CS B. Previous endometritis C. Previous pregnancy with IUD D. Abnormal uterine bleeding A,: D. First Aid USMLE Step 2 CK - 10th Ed, P358 119. CTG with variable deceleration, cause? A. Asphyxia B. Respiratory acidosis C. Metabolic acidosis D. Uteroplacental insufficiency A,: C. Hacker & Moore's Ess. of OB&GYN.Ch9 120. Family had grade 2 injury post-delivery after episiotomy. She developed painful hematoma. What to do? A. Surgical evacuation A,: A. 121. Large irregular mass in cervix of a young lady which was bleeding. What is the next step? A. Excision B. Take biopsy C. Test for HPV A,: C Pap smear → suspicious cells → colposcopy → Biopsy Cytological results of a Pap test that permits the examination of cells but not tissue structure. Cervical intraepithelial neoplasia (CIN) or cervical carcinoma is a histological diagnosis, requiring a tissue sample via biopsy of suspicious lesions seen during colposcopy. Toronto Notes 2020, P539. 122. Ectopic pregnancy (HCG and US) shows possibility to manage patient medically. But patient was hemodynamically unstable (just hypotension) and lives far away. What is absolutely contraindicated? A. Vital signs B. Living far away A,: B. Noncompliant patient is exclusion criteria for Methotrexate. Who knows if they will follow up? Patients need to return for evaluation to know if the treatment worked and if they need a second dose or surgery. Master the Boards USMLE Step 2 CK - 5th Ed, P900 123. Seizure in pregnancy. Give: A. Phenytoin B. Diazepam in status epilepticus C. MgSO4 A,: C. prevention of recurrent convulsions: to prevent further seizures and the possible complications of repeated seizure activity (e.g. rhabdomyolysis, metabolic acidosis, aspiration pneumonitis, etc.) MgSO4 is now the drug of choice. Toronto Notes 2020, P871 124. 37 week gestational age pregnant lady presents with abdominal pain and the fundal height is 39cm. No vaginal bleeding. What is the diagnosis? A. Abruptio placentae (it's concealed) A,: A. internal/concealed/occult (20%): blood dissects upward toward fetus, may or may not present with vaginal bleeding. Toronto Notes 2020, P860 125. Postpartum hemorrhage case, patient received oxytocin but still bleeding, there is laceration in vagina but there is blood coming from above, uterus feels boggy and tender, next step to manage: A. Manual exploration B. Oxytocin C. Carboprost A,: A? - B: Depends on how much oxytocin given before, we might give another oxytocin - C: Total blood loss between 1000 and 1500 mL Consider the next level of drugs (carboprost 250 µg IM or misoprostol 800 to 1000µg per rectum) and request additional laboratory testing (e.g., a coagulation panel) - Hacker & Moore's Ess. of OB&GYN.Ch10 - Toronto Notes 2020, P890 126. Definitive therapy for large pituitary adenoma secreting prolactin: A. Bromocriptine n B. Trans-sphenoidal surgery A,: B. The trans-sphenoidal route or microsurgical exploration of the sella turcica gives the best results. Hacker & Moore's Ess. of OB&GYN.Ch33 127. Pregnant with mitral stenosis which of the following occur in pregnancy? A. Increase RBC B. Increased stroke volume A,: B. Hacker & Moore's Ess. of OB&GYN.Ch6 128. Ascites, pleural effusion, ovarian tumor. What is the type of this syndrome? A. Sex stromal cord B. Epithelial C. Germinal A,: A. Tumors is derived from the specialized ovarian stroma. As such, they are often endocrinologically functional, many of them being capable of synthesizing gonadal or adrenal steroid hormones. Toronto Notes 2020, P537 129. Patient known case of hypertension on labetalol, controlled. What is the complication the patient may have? A. placental separation B. IUGR A,: B. Beta blockers should be used with caution because they may cause fetal growth restriction and may affect the interpretation of the NST. Growth-restricted fetuses are particularly prone to problems such as meconium aspiration, asphyxia, polycythemia, hypoglycemia, mental retardation, and the fetal heart rate tracing may show evidence of late decelerations or bradycardia. Hacker & Moore's Ess. of OB&GYN.Ch14 130. Patient with positive history of IHD, DM, HTN and HF admitted for hysterectomy and 2 days before the surgery she seems pale and dehydrated so, she has given IV fluid after the surgery she developed shortness of breath and examination reveals bilateral lung crepitations What they should have done? A. Consult Cardio and Pulmonary B. Daily asses that she needs IV fluid C. Postoperative Furosemide A,: B. HF 131. Neonate Male full term comes e symptoms poor feeding & others.what is to save him from liability of diagnosed necrotizing enterocolitis A. male sex B. full term C. young age A, is B Explanation: Necrotizing enterocolitis (NEC) is one of the most common gastrointestinal emergencies in the newborn infant. It is a disorder characterized by ischemic necrosis of the intestinal mucosa, It is more likely to happen with preterm infants, and the risk decreases as the GA increases. Source: illustrated textbook, Kaplan usmle step 2 132. Pregnant with UTI which drug contraindicated A. ciprofloxacin A, is A Explanation: Fluoroquinolone in general should not be used in pregnancy except in cases of infections with resistant micro-pathogens or complicated urinary tract infections during pregnancy. Source: PubMed 133. Patient 40-50 years old presented with post coital bleeding , weeks ago she was treated for genital wart by electrocautery what is the source of bleeding? A. Uterine cervix B. Uterine body C. Vulva Vagina A, is A Explanation: the cause could be a recurrence of genital warts where the median time for recurrence is 14 w, or bleeding from the electrocautery which heals in 2-4 w, or cervical originating cause (most common cause of postcoital bleeding) like cervical cancer as the risk of having hpv (multiple sexual partners) is indicated in the question which is more likely. Source: uptodate, PubMed 134. Woman with normal vitals and laps without any complaints except Some white vaginal discharge with normal PH negative whiff test no growth on culture? A- BV B- normal physiological discharge C- Trichomonas D- Chlamydia A, is B Explanation: chlamydia discharge is yellowish or greenish in color, and trachomonas has purulent discharge with vulvar pruritis and change in the Ph. Above negative tests exclude the other choices. Source: hacker and Moore p276 135. female patient complains of urinary dribbling, dyspareunia, dysuria. What is the most likely diagnosis? A. Overflow incontinence B. Urethral diverticulum C. Stress incontinence D. Urge incontinence A, is B Source: hacker and Moore p247 136. Baby was diagnosed with intrauterine fetal death using only US Further investigation A. Inform the mother B. C/S A, is A Explanation: US is an enough modality to diagnose intrauterine fetal death. in case the GA was >20w the mother have the choice to terminate the pregnancy immediately or delay the induction or wait for spontaneous labor, as 90% with have spontaneous labor by 2w, bare in mind careful assessment for DIC, mother also have the choice to do D/E if GA <28. If GA <20w surgical and medical management are equally effective and the choice is to the mother, but D/C is preferred for insuring complete evacuation in first 48h. Source: uptodate 137. One of this is confirmatory for pregnancy A. Uterus just above symphysis pubis B. Uterus between symphysis pubis and umbilicus C. Uterus at the umblicus D. Uterus just above the umblicus A, is A Explanation: the question could've meant to ask about where the uterus is first palpated in pregnancy. Source: uptodate 138. Picture of variable deceleration? Explanation: Source: hacker and Moore p132 139. Ectopic pregnancy is a defect in which process: A. Implantation A, is A Source: hacker and Moore p302 140. Pt with history of PID and treated for it is now coming in the clinic because of infertility despite regular sexual activity what investigation will you do first: A. Hysterosalpingogram A, is A Explanation: Hysterosalpingogram is among the first investigations to order as bmj mentioned along with

Case postmenopausal vaginal bleeding. Hysteroscope imaging showed 2 elongated endometrial projections. Diagnosis? A. Subserosal fibroid B. Endometrial polyps

: B. Her age goes more with endometrial polyps

Lady with cervical lesion. Best diagnostic test? A. Biopsy B. Colposcopy

: B. If only lesion → colposcopy If obvious suspicious mass → Biopsy colposcopy is a clinical procedure that facilitates identification and biopsy of suspicious cells. • in colposcopy: apply acetic acid and identify acetowhite lesions, punctation, mosaicism, and abnormal blood vessels to guide cervical biopsy. Toronto Notes 2020, P539 24

on examination there is no mass. She wishes to get pregnant. What's your next step ? A. Breast US B. CT C. Pap smear

: C? 5

16 year old girl with irregular menstrual cycle. She was uncomfortable and agitated. What is her problem? A. Mood B. Behavioral C. Anxiety

: C? 12

heavy menstrual bleeding or infertility, pelvic pressure related to uterine bulk and, rarely, pain secondary to degeneration, and cervical prolapse may happen with leomyosarcoma. However, the diagnosis of uterine sarcoma is considered in postmenopausal women with presumed uterine leiomyomas who have symptoms sufficiently bothersome to consider hysterectomy. Source: hacker and Moore p 251, uptodate 183. 28 years old female came to ER with heavy vaginal bleeding. She is nulliparous, pregnancy test negative, no pain, and regular cycle, but always with menorrhagia. How to stop the bleeding now? A. Give estrogen (just estrogen, no OCP) B. Give progesteron C. Give GnRH D. Inserted levenogestrel IUD

is A Explanation: AUB have a classification that Samarize the causes: PALM-COEIN Classification Visualizable by inspection or imaging: P: Polyps A: Adenomyosis L: Leiomyoma M: Malignancy Needs further workup: C: Coagulopathy O: Ovulatory disorders E: Endometrial I: Iatrogenic N: Not yet classified The most common cause is anovulation. Anovulation results in unopposed estrogen where there is continuous stimulation of the endometrium with no secretory phase. An estrogen dominant endometrium is structurally unstable as it increasingly thickens and eventually undergoes random estrogen breakthrough bleeding. administering progestin to stabilize the endometrium will stop the bleeding and prevent random breakdowns. Older recommendations uses high dose estrogen, however, newer recommendations say that it might worsen the hypercoagulable state the patient might be in. So high-dose progestin-only therapies have been recommended as first-line treatment for acute heavy menstrual bleeding, particularly in the outpatient setting. Source: hacker and Moore p 325, Kaplan step2 p245 184. Question about rheumatoid arthritis and its effect on pregnancy: A) abortion B) something antibodies C) retardation A, is A Explanation: the effect of RA on pregnancy varies depends on having a flare up in pregnancy. In general mothers who had RA were significantly having more preterm babies, also only some studies report an increase in abortion rate. On the other hand pregnancy has a good effect on reducing severity of RA, which is linked to the increase in sex hormones. Some medications are teratogenic if used during pregnancy like methotrexate and Leflunomide. Moreover, there is no antibodies transfer to child during pregnancy. Source: uptodate, hacker and Moore p 211 185. lactating mother came with mastitis 5 cm. What is the treatment? A. I&D B. Mastectomy C. needle aspiration D. Antibiotics A, is A Explanation: in this question the patient probably had an abscess which needs incision and drainage. Breast feeding can be continued by breast bump. Source: hacker and Moore p 116 186. Female patient presented with fever, supra pubic pain, and vaginal discharge. What is the diagnosis? A. Acute Cervicitis B. Acute Salpingitis C. Vaginal problem D. Vulva problem A, is B Explanation: the presentation matches salpingitis for cervicitis present with bleeding eather atraumatic or with minor cervical trauma. Source: hacker and Moore p 305, bmj 187. Patient with mitral stenosis wants to get pregnant. What is the physiological change that happens in pregnancy leading to heart failure? A. Increased RBC B. Increased stroke volume A, is B Source: hacker and Moore p 63 188. Pregnant lady on anti-thyroxine medication for 10 years yet her hyperthyroidism is not controlled. What should be done for this patient? A. double the dose B. Same dose A, is decrease the dose Explanation: a slight change to the goal of treatment during pregnancy is to maintain persistent but mild hyperthyroidism in the mother in an attempt to prevent fetal hypothyroidism since the fetal thyroid is more sensitive to the action of antithyroid drugs. Thionamides are associated with some congenital defects and hypothyroidism. However, hacker mentioned that PTU should be used to treat overt hyperthyroidism in the first trimester, and methimazole should be used in the second and third trimester. Source: hacker and Moore p 206, uptodate 189. Patient presented with mass in the labia majors with black color. What is the most likely diagnosis? A. Basal cell cancer B. Melanoma A, is B Explanation: squamous cell carcinoma is the most common type of tumors in the labia. vulvar lump, pruritus. The lesions may be raised, ulcerated, pigmented, or warty in appearance. Malignant melanoma is the second most common type of vulvar cancer. pigmented lesion on the vulva, unless it has been known to be present for a long time. Basal cell carcinomas of the vulva are rare. They commonly present as a rolled-edged "rodent" ulcer, although nodules and macules may occur. A possible A, could be SCC as it happens more in labia majors than in minora, while melanoma happens more in labia minora. Source: hacker and Moore p 452 190. Lactating mother with unilateral breast abscess. What should be done to breastfeeding? A. Stop breastfeeding B. Continue breastfeeding normally C. Continue feeding with the non affected breast A, is C Source: uptodate, hacker and Moore p 116 191. What's the most important thing to check before instrumental delivery ? A. Breech B. placenta abruption C. head pelvic disproportion A, is C Explanation: an estimate of the fetal size, and the adequacy of the maternal pelvis is mandatory. You should also not forget the presentation of fetus along with adequate anesthesia. Source: hacker and Moore p 230 192. 7 y old female presented with breast development, pubic hair started to appear, and acne. What is the diagnosis? A. Ovarian tumor B. Central C. Gonadotropin A, is B Explanation: in order to determine if it is true precocious puberty (central) or another source we give exogenous GnRH (a GnRH stimulation test) with a resultant rise in LH levels equivalent to those seen in older girls who are undergoing normal puberty. Refer to hacker and Moore p 377 for other causes. Source: hacker and Moore p 377 193. 30 years old female patient presented with policystic ovaries, and she is obese. She doesn't want to get pregnant. What is your action? A. Give metformin B. Progesterone only bills C. Combined estrogen & progesterone A, is C Source: bmj, hacker and Moore p 392 194. A fetus was discovered to be dead at one of the antenatal visits. The mother is crying. Her husband is upset. What is your role? A. discuss the possibility of future fetal malformations B. explore, identify their emotions, empathize. C. explore, recognize their emotions, find solutions. D. empathy is sufficient Suggested A, is B Source: no source was found 195. Primigravida patient presented with cervix 5 cm, fetus is in station 0 with cephalic presenting part. CTG showing one variable deceleration. This state continued for 4 hours, even the oxytocin had been taken. what is the management for this patient? A. stop oxytocin B. immediate CS C. follow up D. instrumental delivery A, is C Explanation: Cord compression with or without other sources of interrupted fetal oxygenation may result in recurrent variable decelerations with absent/minimal variability and no accelerations. Here there is no recurrent variable decelerations, so no fetal distress. As for this patient she seems to be in the latent phase of first stage, and oxytocin induction takes 6 or more hours in 70% of inductions to move to active phase, some patients reach up to 12 hours. Source: hacker and Moore p 106, uptodate 196. Which bacterial infection is associated with IUD in PID? A, is actinomyces Explanation: actinomyces is an IUD associated infection. If was discovered incidentally without PID symptoms, no need to take off the IUD, if there was PID symptoms, removal of IUD and penicillin is required. Source: uptodate 197. Pregnant lady at 22 weeks gestational age. Oral glucose challenge test after one hour, 2 hours, and 3 hours are high. What is next? A. Repeat same test B. HgA1c C. Fasting blood glucose D. Random blood glucose A, is C Explanation: OCTT is checking the patient's fasting blood glucose after an overnight fast, having the patient consume a 100-g glucose drink, and checking her glucose levels hourly for 3 hours. Source: hacker and Moore p 204 198. 48 years old female came with abnormal uterine bleeding. Transvaginal ultrasound shows subserous fibroid and endometrial thickness 14 mm. what is the diagnosis? A. Fibrosarcoma B. Myosarcoma C. Fibromyoma A, is B Explanation: myosarcoma suspected if a known fibroid uterus appears to be rapidly increasing in size especially postmenopausally, malignancy should be suspected. Usually diagnosed at the time of exploratory surgery. Source: hacker and Moore p 463 199. Female on OCP for 12 years, developed gradual RUQ pain (not something acute). CT was ordered, and it showed Hepatic adenoma 5 cm. How to manage? A. Stop OCP B. Excision C. Left hepatectomy D. Liver transplant A, is A Explanation: exposure to estrogen a risk facto for developing hepatocellular adenoma. The annual incidence of HCA in users of OCs is approximately 30 to 40 cases per million OC users in comparison with one case per million nonusers. Source: uptodate 200. What causes 3rd, 4th degree tear in labour? A. Squatting position Explanation: Perianal laceration is caused by baby stretching the outlet. Primipara and occipitoposterior position (OP) during delivery are the only statistically significant risks for the occurrence of severe perineal damage. High birth weight is a risk factor but it is not statistically significant. Also instrumental delivery in OP is risk factor. Source: https://www.ncbi.nlm.nih.gov/pubmed/19274544 201. long history of pregnant lady that received IV MgSO4. What drug reverse the action? A. Ca Gluconate A, is A Source: hacker and Moore p 190 202. Case about Molar pregnancy. What is the management? A. Medical B. Surgical C. Evacuation D. Observation A, is C Source: hacker and Moore p 469 203. Old female with diabetes mellitus. What is the most common cause of vaginal infection? A. Candida B. Bacterial vaginosis C. Trichomoniasis A, is A Source: hacker and Moore p 278 204. Female patient presented with cottage like vaginal white discharge. What could this patient have that may cause this? A. Diabetes mellitus B. Hypertension A, is A Explanation: description of candidiasis discharge Source: hacker and Moore p 278 205. Patient presented with polycystic ovaries. Which hormone is responsible for hirsutism? A. Testosterone B. Progesterone C. ACTH A, is A Source: hacker and Moore p 390 206. Female patient in the 2nd stage of labor with face presentation. What should be done? A. CS B. Oxytocin A, is A Explanation: face presentation depends on the mentum direction: Mentoposterior and transverse: CS Mentoanterior: vaginal / assisted (forceps) Source: hacker and Moore p 181 207. Tanner stage of puberty self-study Explanation: refer to hacker and Moore p 373 208. Female patient with history of chlamydial infection that wants to get pregnant. What should be done for this patient? A. Hysterosalpingography A, is A Explanation: chlamydia causes tubal adhesions and destroy the Celia causing infertility or ectopic pregnancy. Source: hacker and Moore p 402 209. When you examined a pregnant lady, you felt the nose and mouth. What is the presentation? A. Face presentation Source: hacker and Moore p 181 210. Pregnant lady underwent an episiotomy until rectum. what is the degree of tear? A. 1st degree B. 2nd degree C. 3rd degree D. 4th degree A, is D Explanation: First degree: a laceration involving the vaginal epithelium or perineal skin, Second degree: a laceration extending into the subepithelial tissues of the vagina or perineum with or without involvement of the muscles of the perineal body, Third degree: a laceration involving the anal sphincter, Fourth degree: a laceration involving the rectal mucosa Source: hacker and Moore p 110 211. Pregnant lady in her 12th week of gestation presented with bleeding. A lesion was found in the cervix. what should be done? A. Biopsy B. Pap smear C. Colposcopy A, not known There is a list of deferentials for this all of them are done, however, more information required to chose the most important on. Source: bmj 212. Pregnant lady with chlamydia infection. Where is the most common location in the baby that might be infected? A. Ear B. Eye C. Lung D. Liver A, is B Explanation: The most frequent clinical manifestation of C. trachomatis infection in the newborn is conjunctivitis. Source: uptodate 213. 32 years old female with amenorrhea. She has normal prolactin and TSH, but she has high FSH and LH. What is she at risk of? A. Osteoporosis B. Ovarian cancer C. Endometrial cancer A, is A or C Explanation: Endometrial cancer: if LH is 3 times FSH (prolonged PCOS) Osteoporosis: If FSH is 3 times LH (amenorrhea) Source: hacker and Moore p 411 214. Diabetic mother delivered a big baby. She has vaginal bleeding 800 ml: A. Primary postpartum hemorrhage B. Secondary postpartum hemorrhage A, is A Explanation: PPH occurring in the first 24 hours after delivery may be called primary or early PPH. PPH occurring from 24 hours to 12 weeks after delivery is usually called secondary, late. Source: uptodate 215. Female in labour. She is 4 cm dilated and 50% effaced, and there is no more progress: A. Artificial rupture of membrane B. Oxytocin A, is .... Explanation: latent phase is not prolonged until it reaches 20h for nulliparus, and 14h for multiparus. Recommended augmentation is therapeutic rest for women who are tired and uncomfortable in early latent phase and oxytocin for a woman who is well rested or has already received therapeutic rest. Depending on the duration, which is missing in the question we decide, eather wait or oxytocin. Source: uptodate 216. Female patient in labour. She is 10 cm dilated. There is no progress after 1 hour: A. CS B. Asses after 2 hr C. Ventouse delivery D. Forceps A, is B Explanation: normal duration on second stage is four hours with epidural anesthesia or three hours without epidural anesthesia in nulliparous women, and three hours with epidural anesthesia or two hours without epidural anesthesia in multiparus women. Examination should be done every 30m. After 60 to 90 minutes of pushing, we begin oxytocin augmentation if descent is minimal (ie, <1 cm) or absent and uterine contractions are less frequent than every 3m. We avoid operative delivery (vacuum, forceps, cesarean) in the second stage as long as the fetus continues to descend and/or rotate to a more favorable position for vaginal delivery, and the fetal heart rate pattern is not concerning. The three major categories of indication are prolonged second stage of labor, nonreassuring fetal status, and maternal cardiac or neurological disease If oxytocin was a choice it would be the best A,, however, in this question watching for 2h is required to consider the 3 other choices. Source: uptodate 217. Pregnant lady at 12th week of gestation with severe vomiting, beta-hCG is very high. US showed fetus with IUGR. What is the diagnosis? A. Choriocarcinoma B. Trophoblast tumor in placenta C. Hydatidiform mole A, is C Explanation: this is partial mole, These lesions, unlike complete moles, often present with a coexistent fetus. Source: hacker and Moore p 466 218. Pregnant woman at 9th weeks of gestation with hyperemesis gravidarum. Admitted with normal saline 10% glucose. What is the best diagnostic test? A. Ketone in urine B. Glucose in urine C. Protein in urine A, is A Explanation: criteria for diagnosis of hyperemesis are persistent vomiting accompanied by weight loss exceeding 5 percent of prepregnancy body weight and ketonuria unrelated to other causes. Alternatively, the diagnosis can be made in women with pregnancy-related vomiting that occurs more than three times per day with weight loss greater than 3 kg (6.2 lbs) or 5 percent of body weight and ketonuria. Source: uptodate, hacker and Moore p 213 219. Pregnant lady G3P2, 37 weeks gestation, with history of CS because of non reassuring CTG. She is in labour with 4 cm dilation. Presentation in breech. What is the absolute contraindication for ECV? A. History of CS B. Active labour C. Variable decelerations A, is all can be Explanation: • Severe oligohydramnios. • Nonreassuring fetal monitoring test results. • Hyperextended fetal head. • Significant fetal or uterine anomaly (eg, hydrocephaly, septate uterus). • Placental abruption. • Ruptured membranes. • Active labor with fetal descent. Allowed only if membranes are intact • Multiple gestation. Source: uptodate, hacker and Moore p 178 220. Patient with ectopic pregnancy. Beta-hCG 3500, 3cm. She lives 80 km from hospital. Why is it contraindicated to give her Methotrexate? A. Hospital accessibility A, is A Explanation: factors required for medical treatment: • Hemodynamic stability. • Serum beta-human chorionic gonadotropin (hCG) concentration ≤5000 milli- international units/mL. • No fetal cardiac activity detected on transvaginal ultrasound (TVUS). Ectopic mass size less than 3 to 4 cm is also commonly used as a patient selection criterion

1-put the patient into lateral recumbent, 2- reduce oxytocin infusion, 3- increase fluids. Category 3 management starts by the same management as category 2 first, then go series of steps discussed in hacker. Source: hacker and Moore p 133 142. A primigravida at 32weeks GA with BP of 150/90mmhg.There is associated edema of hands and legs.What to do? A. Diuretics B. B. Tabs Labetalol C. Continued evaluation

is C Explanation: further evaluation is needed as the position of the mother might affect blood pressure, and for measuring bp position either sitting or lateral decubitus. Swelling of hand and legs is a normal physiology in pregnancy further evaluation is needed for preeclampsia diagnosis and treatment. As for gestational HTN close monitoring only. Source: hacker and Moore p185 143. Placenta implanted deep into uterine wall. Diagnosis? A. Placenta accreta (superfici'l myometrium) B. Placenta increta (deep myometrium) C. Placenta percreta (serosa) A, is B Source: hacker and Moore p139 144. Post-delivery 6 weeks. Exclusive breast feeding. Doesn't want kids for at least 2 years. Which contraceptive method? A. Depo provera injection B. Transdermal patch C. Vaginal ring D. Oral combined contraceptive A, is A Explanation: choice A is an injection that is given every 12-14w which will be more convenient, thus more compliance and is safe with breastfeeding. B is changed every week, C is changed every month. Source: hacker and Moore p331, uptodate 145. Pregnant at the third trimester she is crohn's patient on 'azathioprine + biological agent ask about her child vaccines? A. give all B. delay till 3 month C. delay till 6month A, is not known Explanation: I found a research saying that biological agents are safe for child, but I can't be sure about azathioprine as hacker and Moore only mentioned that there is increased risk of preterm and fetal anomalies. I would suggest A as an A, cuz I found a research that said it is safe and children had no problems with the vaccines. Source: hacker and Moore, https://academic.oup.com/ecco-jcc/article/5/2/95/475727 146. CTG show decrease variability 'and he is on mg sulphate cause is? A. mg sulphate toxicity A, is A Explanation: Magnesium freely crosses the placenta

management? A. Wide local excision The

is: A Explanation: In determining the management of the primary cancer, the following two factors are considered: • The presence of any multifocality • The condition of the remainder of the vulva For patients, whose tumor is unifocal and the remainder of the vulva is normal, radical local excision with surgical margins of at least 1 cm is the treatment of choice. If there is associated VIN or multifocality, a radical or modified radical vulvectomy may be necessary. Source: Hacker Moore Page 453, sixth edition. 272. Fetus after delivery by ventouse

these two conditions account for the vast majority of women with a low hemoglobin concentration during pregnancy. Source:https://www.uptodate.com/contents/anemia-in- pregnancy?search=iron%20deficiency%20anemia%20in%20pregnancy&source=search_resul t&selectedTitle=1~150&usage_type=default&display_rank=1 295. Pregnant lady admitted for seizure on Mg sulphate, now pt have 0 reflex bilateral LL, and decrease O2 saturation, what to do next ? A. Stop Mg sulphate B. Ca gluconate C. O2 supplement The

is: A Explanation: Magnesium toxicity is treated by stopping the infusion and, when severe, administering IV calcium gluconate, 10 mL of a 10% solution, along with resuscitative measures if necessary. Source: Hacker Moore Page 190, sixth edition. 296. 45y lady had amenorrhea 9 months, now willing to get pregnant again, what to do lab ? • LH and FSH • prolactin • US The A, is: N/A Explanation: All women with primary amenorrhea should have serum human chorionic gonadotropin (hCG), FSH, thyroid-stimulating hormone (TSH), and prolactin (PRL) measured, similar to the approach for women with secondary amenorrhea. 297. Postmenopausal woman on tamoxifen had fibroid 3x4 cm, now fibroid increase in size and US show endometrial thickness 5 mm, what dx ? A. Leiomyoma B. Leiomyosarcoma C. Ovarian Ca D. Endometrial Ca The A, is: D Explanation: In the P-1 study, in which over 13,000 women were assigned to tamoxifen or placebo for five years, with seven years of follow-up, 53 cases of endometrial cancer occurred in the tamoxifen group versus 17 in the placebo group (risk ratio 3.28). In addition, there were four cases of uterine sarcoma, three in the tamoxifen group. the increase in endometrial cancer was almost exclusively in women over the age of 50 years. A comparable risk ratio for endometrial cancer (2.7-fold increased risk) was found in a meta-analysis of 32 published randomized controlled trials. 298. DM pregnant is at risk of what ? A. Candida B. BV C. TRICHOMONAS The A, is: N/A Explanation: We could not find an increased risk of colonization with vaginal pathogens at early gestation in pregnant women with diabetes, compared to non-diabetic women. Large prospective studies are needed to evaluate the long-term risk of colonization with vaginal pathogens during the course of pregnancy in these women. Source: https://www.ncbi.nlm.nih.gov/pmc/articles/PMC4864213/ 299. Little girl with vaginal itchiness and frequent bleeding for 2 day The A, and explanation are: Sexually transmitted infections (STI) may cause intermenstrual bleeding in sexually active adolescents and/or girls who have been sexually abused or assaulted. Such girls who have acute vaginal bleeding unrelated to menses should be assessed for cervicitis and upper genital tract infection. The prevalence of Chlamydia trachomatis in women with AUB is underestimated. Routine screening for STI is recommended for sexually active adolescents with menstrual abnormalities. Note: the A, might not be true since the question lack so many other necessary information. 300. . lady with vaginal discharge fishy odor and clue cell, how to treat her husband ? A. No need for tx The A, is: A Explanation: Treatment of the male sexual partner does not improve therapeutic response and therefore is not recommended. Source: Hacker Moore Page 277, sixth edition 301. Lady with greenish yellow discharge, flagellated organism , what tx ? A. Metronidazole The A, is: A Explanation: Metronidazole is the drug of choice for treatment of vaginal trichomoniasis. Source: Hacker Moore Page 278, sixth edition 302. Uterine Ca, what lymph node to dissect? A. Para aortic B. Internal iliac C. External iliac D. Inguinal The A, is: C Explanation: The lateral pelvic pathway drains lymph from the pelvic organs to the medial chain of the external iliac lymph node group. Carcinomas of the cervix, endometrium, ovary, and upper two-thirds of the vagina often spread to this lymph node chain. Source: https://pubs.rsna.org/doi/pdf/10.1148/rg.2015140086 303. Lady 24 y come for routine gynecological visit, everything is normal, what must you do now for general screening ? A. Pap smear B. US The A, is: A Explanation: In women age 21 to <30 years with a normal immune system and prior results (if any) that are all normal: We suggest screening with Papanicolaou (Pap) testing. Some clinicians do Pap testing alone, whereas others do a Pap test with reflex to HPV testing. If the Pap test is negative, screening should be repeated at intervals of every three years. Source:https://www.uptodate.com/contents/screening-for-cervical- cancer?search=pap%20smear&source=search_result&selectedTitle=1~142&usage_type=defa ult&display_rank=1#H417367560 304. Pregnant with vaginal wart, what is the safest treatment? A. Cryotherapy The A, and explanation are: trichloroacetic acid has no systemic absorption and no known fetal effects

this is always an abnormal finding. However, in some cases, membranes may be prolapsed or ruptured. we perform serial TVUS examinations and make a diagnosis of cervical insufficiency when cervical length is ≤25 mm before 24 weeks Source:https://www.uptodate.com/contents/cervical- insufficiency?search=cervical%20incompetence&source=search_result&selectedTitle=1~50 &usage_type=default&display_rank=1 277. What is the best time to check for GA by ultrasound: A. 11 to 13 B. 20 to 22 C. 26 to 28 D. 30 to 32 The

is: A Explanation: Measurement of fetal crown-rump length between 6 and 11 weeks' gestation can define gestational age to within 7 days. At 12 to 20 weeks, gestational age can be determined within 10 days by the average of multiple measurements (e.g., biparietal diameter, femur length, abdominal and head circumferences). Thereafter, measurements become less reliable with advancing gestation (3 weeks in the third trimester). Source: Hacker Moore Page 83, sixth edition. 278. What is the most common cause of morbidity in multiple pregnancy: A. birth asphyxia B. prematurity C. cerebral palsy D. birth trauma The A, is: b Explanation: Worldwide, preterm labor and delivery are major causes of perinatal morbidity and mortality. Source: Hacker Moore Page 156, sixth edition. 279. 17 years- old female athlete in her school came due to delayed menarche

this is thought to be a transient functional defect in production of GnRH from the hypothalamus, due to individual variations that can be genetic. Constitutional delay of growth and puberty (CDGP) tends to have familial patterns of inheritance, often following an autosomal dominant pattern such that family members often have a history of being "late bloomers," with a late growth spurt or late puberty compared with their peers. Source: https://www.uptodate.com/contents/approach-to-the-patient-with-delayed- puberty?search=Delayed%20Puberty&source=search_result&selectedTitle=1~113&usage_ty pe=default&display_rank=1#H2405642217 327. Strawberry appearances in cervix ? A. Trichomonas The

is: A Explanation: Trichomonas vaginitis is associated with a profuse, purulent, malodorous vaginal discharge that may be accompanied by vulvar pruritus. In patients with high concentrations of organisms, a patchy vaginal erythema and colpitis macularis ("strawberry" cervix) may be observed. Microscopy of the secretions may reveal motile trichomonads. Source: Hacker Moore Page 278, sixth edition = 328. Female with PPH give OXYTOCIN and UTRINE MASSAGE not stop next step: The A, is : pharmacologic treatments > bimanual compression and massage of the uterine corpus > large-volume balloon catheter >embolization> hysterectomy Source: Hacker Moore Page 144, sixth edition 329. Menopause patient has vulvar itching and bleeding after duration she have pea like size, DX ? A. vulvar carcinoma B. Bartholin cyst The A, is: A Explanation: Patients generally present with a vulvar lump, although long-standing pruritus is common. The lesions may be raised, ulcerated, pigmented, or warty in appearance, and definitive diagnosis requires biopsy of the lesion with the patient under local anesthesia. Source: Hacker Moore Page 451, sixth edition 330. Female postpartum came to the hospital with chest pain then died suddenly what is the most likely cause? A. MI B. Postpartum cardiomyopathy The A, is: A Explanation: Spontaneous coronary artery dissection (SCAD) is a rare cause of acute myocardial infarction (AMI) in young, otherwise healthy patients, which may even lead to sudden death. The etiology of this condition is still not fully understood, it affects mostly women and nearly two-thirds of cases occur during childbirth or postpartum without any history of atherosclerotic disease. Note: Chest pain is not associated with cardiomyopathy. Source: https://www.sciencedirect.com/science/article/pii/S1878540913001382 https://www.ahajournals.org/doi/10.1161/CIR.0000000000000564 331. admitted for ectopic after 3 weeks bhcg is platua (remains the same) ? What to do? A. Admit patient for surgery again B. Do nothing it will go Normal C. Hysterectomy The A, is: N/A Explanation: In addition, there is a 10-20% risk of residual trophoblastic tissue whenever the products of conception are dissected from the fallopian tube (i.e., when salpingostomy or salpingotomy is performed). Patients who do not have resection of the affected tubal area should have repeat hCG titers 3 to 7 days postoperatively to confirm that no hCG-producing cells remain to reinvade the tube. When repeat hCG titers fail to decline appropriately, methotrexate (MTX) therapy can be started Source: Hacker Moore Page 451, sixth edition 332. Pregnancy have fever and not seek medical advice? A. Neglect The A, and explanation are: Numerous risk factors for intrapartum fever have been reported, such as nulliparity, prolonged labor, and prelabor rupture of membranes [1]. These characteristics describe women likely to develop intraamniotic infection (IAI) and/or receive neuraxial anesthesia, which are the two most common causes of intrapartum fever. In the absence of a preexisting febrile disorder (eg, respiratory infection), most pregnant women who develop fever in labor are presumed to have IAI and are treated with broad spectrum antibiotics. Source:https://www.uptodate.com/contents/intrapartum- fever?search=Fever%20during%20pregnancy&source=search_result&selectedTitle=1~150& usage_type=default&display_rank=1#H1 Note: this is not a clear question. 333. Baby with mucous vaginal discharge what is your management! A. reassurance The A, is A Explanation: Hymens change with age. Newborns have redundant, estrogenized, thick, elastic hymens, often with a prominent ridge at six o'clock. White discharge due to maternal and fetal/newborn estradiol-stimulated mucus production may appear at the hymenal orifice. Source:https://www.uptodate.com/contents/gynecologic-examination-of-the-newborn-and- child?search=Baby%20with%20mucous%20vaginal%20discharge&source=search_result&se lectedTitle=4~150&usage_type=default&display_rank=4# 334. Hep B vaccine type? A- recombinant vaccine The A, is A. Source:https://www.uptodate.com/contents/hepatitis-b-virus-immunization-in- adults?search=hepb%20vaccine&source=search_result&selectedTitle=2~146&usage_type=d efault&display_rank=1 335. . Risk factor for endometriosis? A. Nullipara B. Low weigh The A, is: N/A Explanation: Factors associated with an increased risk of endometriosis include: • nulliparity • prolonged exposure to endogenous estrogen (eg, early menarche [before age 11 to 13 years] • late menopause shorter menstrual cycles (defined as ≤27 days) • heavy menstrual bleeding • obstruction of menstrual outflow (eg, Müllerian anomalies • exposure to diethylstilbestrol in utero height greater than 68 inches • lower body mass index • exposure to severe physical and/or sexual abuse in childhood or adolescence • a high consumption of trans unsaturated fat. Source: https://www.uptodate.com/contents/endometriosis-pathogenesis-clinical-features- and- diagnosis?search=endometriosis&source=search_result&selectedTitle=1~150&usage_typ e=default&display_rank=1#H1 336. What is true of endometriosis: A- Not related to any cancer B- Related to ovarian cancer C- Related to thyroid cancer The A, is: B Explanation: Endometriosis appears to be associated with some epithelial ovarian cancers (EOC)

the other is an adjuvanted recombinant vaccine (Shingrix), which the ACIP recommends deferring until after pregnancy (if indicated) because of a lack of safety data. Source:https://www.uptodate.com/contents/immunizations-during- pregnancy?search=Vaccine%20contraindicated%20in%20pregnancy&source=search_result& selectedTitle=1~150&usage_type=default&display_rank=1#H262425443 315. Patient with vaginal bleeding and absent fetal movement, next step: A. US B. admission to ward C. discharge the

is: A Explanation: fetal death should be suspected when the patient reports the absence of fetal movements, particularly if the uterus is small for dates. Diagnostic confirmation has been greatly facilitated since the advent of ultrasonography. Real-time ultrasonography confirms the lack of fetal movement and absence of fetal cardiac activity. Source: Hacker Moore Page 168, sixth edition 316. Pic of Anembryonic pregnancy 317. Picture of uteroplacental insufficiency 318. Picture of head compression variability CTG 319. 16 years old came with right lower quadrant pain with vomiting, on ultrasound there was abdominal fullness Diagnosis? A- Appendicitis B- Torsion ovary The A, is : B Explanation: Questions regarding symptoms associated with ovarian torsion, including pelvic pain and nausea. The patient should be asked regarding a history, or recent diagnosis, of an adnexal mass. Ultrasound is the initial imaging study of choice for patients with suspected ovarian torsion. Ultrasound findings: • An ovary that is rounded and enlarged compared with the contralateral ovary. • An ovarian mass • Heterogeneous appearance of the ovarian stroma • Multiple small peripheral follicles ("string of pearls") • Abnormal ovarian location • Decreased or absent Doppler flow within the ovary Source: https://www.uptodate.com/contents/ovarian-and-fallopian-tube- torsion?search=Torsion%20ovary&source=search_result&selectedTitle=1~71&usage_typ e=default&display_rank=1#H849965 320. 6-year-old girl mother complain of the girl has vaginal itching and bleeding discharge? Dx? A. Candidiasis B. sexual abuse C. foreign body the A, is: C Explanation: Foreign bodies in children can cause acute and chronic recurrent vulvovaginitis. Chronic vaginal discharge, intermittent bleeding or spotting, and/or a foul-smelling odor are the usual clinical manifestations. N.B: children who are victims of sexual abuse may present with physical findings, including anogenital problems, enuresis, or fecal incontinence. Source:https://www.uptodate.com/contents/overview-of-vulvovaginal-complaints-in-the- prepubertalchild?search=vaginal%20itching%20in%20adolescent&source=search_result&sel ectedTitle=1~150&usage_type=default&display_rank=1#H8 https://www.uptodate.com/contents/functional-fecal-incontinence-in-infants-and-children- definition-clinical-manifestations-and-evaluation?topicRef=6605&source=related_link 321. Female with clear presentation of PCOS and they put some tests of PCOS in the question and ask what would you order also? A. TSH B. prolactin C. glucose and lipid profile the A, is: C Explanation: Abdominal adiposity in women with PCOS preferentially worsens with weight gain, as does the prevalence of metabolic syndrome (elevated blood pressure and blood glucose with excess body fat around the waist). Metabolic syndrome, along with its underlying insulin resistance, occurs two to three times more frequently in women with PCOS than in age- matched controls. Source: Hacker Moore Page 391, sixth edition 322. Women just had vaginal delivery and think that her child is going to not live long even though the child is healthy? A. postpartum depression B. Postpartum psychosis The A, is: B Explanation: Homicidal behavior is rare in postpartum psychosis. Approximately a third of women hospitalized for postpartum psychosis expressed delusions about their infants, and 9 percent had thoughts of harming their infants. Approximately 4 percent of women with postpartum psychosis have been found to commit infanticide. Disorganization and confusion in the mother add to the potential risks for the infant, who should not be left alone in the care of a mother with postpartum psychosis. Source:https://www.uptodate.com/contents/postpartum-psychosis-epidemiology- pathogenesis-clinical-manifestations-course-assessment-and- diagnosis?search=Postpartum%20psychosis&source=search_result&selectedTitle=1~10&usa ge_type=default&display_rank=1 323. Female in labor facial presentation what to do? A. Cs B. instrumental delivery C. normal delivery The A, is: N/A Explanation: Approximately 60% of face presentations are mentum anterior at the time of diagnosis, whereas 15% are mentum transverse and 25% mentum posterior. If the mentum rotates anteriorly under the symphysis pubis, vaginal delivery should be expected. Forceps, but not vacuum, can be applied to assist if prerequisites are met. However, if the mentum rotates posteriorly, the fetal head will be unable to extend farther to complete the expulsive process. Thus, mentum posterior cases and those with persistent mentum transverse must be delivered by cesarean delivery. Source: Hacker Moore Page 181, sixth edition 324. Pregnant lady with Crohn's disease? A. Avoid mtx The A, is: A Explanation: Methotrexate is contraindicated in pregnancy as it is a potent abortifacient, and its use during pregnancy is associated with multiple skeletal abnormalities. Women who plan to conceive should therefore discontinue methotrexate and use contraception for at least three months, and ideally six months, prior to conception. Source: https://www.uptodate.com/contents/fertility-pregnancy-and-nursing-in-inflammatory- bowel- disease?search=crohn%27s%20disease%20in%20pregnancy&source=search_result&selected Title=1~150&usage_type=default&display_rank=1#H2130501697 325. Rh positive partner when to al Rh negative pregnant lady when to give ? A. Give Rh immune globulin if mother negative and husband positive The A, is: A Explanation: If the father is RhD-positive, his Rh genotype should be determined using quantitative polymerase chain reaction. If he is homozygous for the D antigen, the fetus will be RhD-positive and potentially affected. Rh immune globulin (300 μg) should be given routinely within 72 hours of delivery to all Rh-negative, anti-D-negative women who deliver an Rh-positive child. Source: Hacker Moore Page 196,200, sixth edition 326. 13 years old female short stature, amenorrhea, brought by her mom concerning about that her daughter didn't develop pubic hair and breast ? Where is the problem? A. hormonal problem B. Constitutional The A, is: B Explanation: This is the most common cause of delayed puberty

diagnosis : A. Caput succedaneum B. Cephalohematoma C. Other choice (not subdural hematoma) The

is: B Explanation: Cephalohematoma presents as a swelling of the scalp that does not cross suture lines due to a subperiosteal hematoma. It is more common after instrumented delivery. Most cases resolve spontaneously over weeks, but there can be complications, such as calcification or ossification, infection, and sepsis. Source: https://www.uptodate.com/contents/skin-lesions-in-the-newborn-and- infant?search=Cephalohematoma&source=search_result&selectedTitle=2~22&usage_type=d efault&display_rank=2 273. During delivery of a baby, there was stylomastoid foramen trauma. Which of the following features will be evident when you examine this baby? A loss of eye closure B. Loss of face sensation C. Loss of mastication D Loss of posterior 2/3 of tongue sensation The A, is: A Explanation: Congenital facial paralysis can result from developmental defects or delivery traumas. Perinatal traumas are the most frequent causes of congenital paralysis. The main reported risk factors associated to traumatic facial paralysis are: mother's first child, birth weight greater than 3500 g, use of forceps, cesarean birth and prematurity. These cases have usually a favorable prognosis, with infants recovering the full functionality of the seventh cranial nerve within few months without sequelae. Source: https://www.ncbi.nlm.nih.gov/pmc/articles/PMC4677084/ 274. CTG variable deceleration dx: A. MgSo4 toxicity The A, is: A Explanation: magnesium freely crosses the placenta

management? A. Laparoscopic cholecystectomy B. Conservative. The

is: B Explanation: Supportive care includes pain control, intravenous fluid therapy, and nutritional support, as needed, and antibiotic therapy depending upon the clinical presentation. In case of Acute cholecystitis — Definitive, prompt surgical therapy is required for any patient with cholecystitis and signs of sepsis, suspected gangrene, or perforation, as well as disease progression while on antibiotic therapy. In the absence of the previous signs treatment for acute cholecystitis depends on the gestational age. Source:https://www.uptodate.com/contents/gallstones-in- pregnancy?search=pregnant%20with%20cholelithiasis&source=search_result&selectedTitle =1~150&usage_type=default&display_rank=1#H7581759 269. Threatened abortion: advice to the patient? A. Complete bed rest B. Resume daily activities The A, is: N/A Explanation: management consists essentially of reassurance

as a result, the cord blood concentration approximates the maternal serum concentration. Maternal therapy causes a decrease in baseline fetal heart rate, which generally remains within the normal range, and a decrease in fetal heart rate variability, which may be absent or minimal. Source:https://www.uptodate.com/contents/preeclampsia-management-and- prognosis?search=magnesium%20sulphate%20toxicity&source=search_result&selectedTitle =2~148&usage_type=default&display_rank=1#H17 275. Uterine atony, HTN ,HTN, givenocin but still bleeding what to do? A. Suture B. -tamponade The

is: B Explanation: intrauterine balloon tamponade is indicated when uterotonic drugs and bimanual compression of the uterus fail to control bleeding. Its use is indicated before resorting to more invasive surgical approaches requiring laparotomy. Balloon tamponade is often successful, obviating the need for arterial embolization or an open surgical intervention. Source:https://www.uptodate.com/contents/intrauterine-balloon-tamponade-for-control-of- postpartum- hemorrhage?search=balloon%20tamponade&source=search_result&selectedTitle=1~43&usa ge_type=default&display_rank=1#H4 276. Pregnant (don't remember GA but before 20 presented with lower abdominal pain

whether women with endometriosis are at risk for other types of cancers is unclear, but the overall risk appears to be low. Source: https://www.uptodate.com/contents/endometriosis-pathogenesis-clinical-features- and- diagnosis?search=endometriosis&source=search_result&selectedTitle=1~150&usage_type=d efault&display_rank=1#H1391765200 337. Pregnant Leopold maneuver 1st and 2nd finding: soft not hard fetal heat beat at umbilicus presentation: A- Face B- Brow C- Shoulder D- Breech The

is: N/A Explanation: If the fetal heartbeat is loudest below the mother's bellybutton, the baby is probably in the vertex position. (b) If it is loudest above her bellybutton, the baby may be breech. Note: Nothing found regarding heartbeat heard at the umbilicus Source: https://www.open.edu/openlearncreate/mod/oucontent/view.php?id=41&printable=1 338. . Pregnant w/t asymptomatic UTI ..(not mention any weeks or anything) Tx: A- Nitro B- Cipro C- Cephalon D- Augmentin The A, and explanation are Augmentin seems to be the safest. Source:https://www.uptodate.com/contents/urinary-tract-infections-and-asymptomatic- bacteriuria-in- pregnancy?search=asymptomatic%20UTI&source=search_result&selectedTitle=2~150&usa ge_type=default&display_rank=2#H6 339. Married 3 years ago with irregular menses and milk expressed manually .. next step: A. b-HCG B. Prolactin The A, is: B Explanation: Hyperprolactinemia in premenopausal women causes hypogonadism, with symptoms that include infertility, oligomenorrhea, or amenorrhea and less often galactorrhea. Note: it might be more reasonable A, since pregnancy present with amenorrhea. Source:https://www.uptodate.com/contents/clinical-manifestations-and-evaluation-of- hyperprolactinemia?search=Galactorrhea&topicRef=6630&source=see_link#H5 340. Mother has gastroenteritis and breast feeding her child (4months old) Cming for vaccines: A-delay vaccines B-give hep b only C-give OPV only D-give them all The A, is: D Explanation: Indications and procedures for vaccination of postpartum women are those described for the general population. Both inactivated and live vaccines (except smallpox and yellow fever vaccine) may be administered to nursing mothers, and breastfeeding does not adversely affect success or safety of vaccination. Source: https://www.uptodate.com/contents/immunizations-during- pregnancy?search=vaccine%20and%20breastfeeding&source=search_result&selectedTitle=1 ~150&usage_type=default&display_rank=1#H35 341. Pic of Hysteroscopic picture for female presented with repeated episodes of bleeding? A-Endometrial polyp B-Endometrial carcinoma C-Fibroid The A, is: A 342. Patient with history of previous myomectomy, now came with bleeding mostly during period increases associated with pain, not relieved with analgesics, on Abdominal Examination uterus was small, on US uterus was normal size ,Dx? A- endometriosis B- adenomyosis C- fibroid The A, is: A Explanation: The characteristic triad of symptoms associated with endometriosis is dysmenorrhea, dyspareunia, and, less frequently, dyschezia. Early in the clinical course, women tend to have cyclic pelvic pain, which starts 1 to 2 days before the menstrual flow and resolves at the end of the menses. The uterus is fixed and retroverted in a substantial number of women with endometriosis. Source: Hacker Moore Page 316-317, sixth edition 343. Pregnant, started to have small cysts of different sizes unilateral around areola. diagnosis? A. fibrocystic (Case wasn't going with it) B. lactiferous duct C. Montgomery glands The A, is: C Explanation: The periareolar glands of Montgomery (Montgomery tubercles, also called Morgagni tubercles) are small papular projections at the edge of the areola that may play a role in lactation . Obstruction of these glands may result in acute inflammation, an asymptomatic subareolar mass (cyst of Montgomery), and/or drainage of clear to brownish fluid Fibrocystic change and fibroadenoma are usually located in the upper outer breast quadrants. Mammary duct ectasia and cysts of Montgomery are subareolar. Source: https://www.uptodate.com/contents/breast-masses-in-children-and- adolescents?search=Montgomery%20glands&source=search_result&selectedTitle=1~5&usa ge_type=default&display_rank=1# 344. Diabetic mother, how to know if baby will be normal? A- FBS B- post prandial C- HgA1C The A, is: C Explanation: The analysis also showed that each one-unit increase in FBG at the time of diagnosis increased the probability of using insulin by 1.062-fold and each one-unit increase in the HbA1c increased it by 3.1-fold. Note: Need for insulin= high-risk pregnancy=more fetal adverse effects. Source: https://www.ncbi.nlm.nih.gov/pmc/articles/PMC4607073/ 345. . 23 years old married for 3 years when should do Pap smear? A. Now The A, is: A Explanation: In women age 21 to <30 years with a normal immune system and prior results (if any) that are all normal: We suggest screening with Papanicolaou (Pap) testing. Source:https://www.uptodate.com/contents/screening-for-cervical- cancer?search=pap%20smear&source=search_result&selectedTitle=1~142&usage_type=defa ult&display_rank=1#H417367560 346. Long scenario with BHCG (80,000) and snowstorm appearance? A-complete molar B-choriocarcinoma C-partial molar D-ectopic The A, is: A Explanation: Sonographic features suggestive of a complete mole include: ● Absence of an embryo or fetus. ● Absence of amniotic fluid. ● Central heterogeneous mass with numerous discrete anechoic spaces - This has classically been described as a "snowstorm or Swiss cheese pattern" on older ultrasounds. Source: https://www.uptodate.com/contents/hydatidiform-mole-epidemiology-clinical- features-and- diagnosis?search=complete%20mole&source=search_result&selectedTitle=2~49&usage _type=default&display_rank=2#H13 347. Old lady multipara with urine incontinence during laugh and cough and sneeze first line treatment? A-surgery B- Kegel exercise C-medication The A, is: B Explanation: Initial treatments for most types of incontinence (stress, urgency, or mixed) include lifestyle modifications and pelvic floor muscle exercise(Kegel), along with bladder training in women with urgency incontinence and in some women with stress incontinence. Source: https://www.uptodate.com/contents/treatment-of-urinary-incontinence-in- females?search=stress%20incontinence&source=search_result&selectedTitle=1~150&usage_ type=default&display_rank=1#H16614991 348. Female with Thin gray vaginal discharge wiff test positive and Ph above 5 treatment? A- cipro B- amoxicillin C- metronidazole D- cefoxime The A, is: C Explanation: Ideally, treatment of BV should inhibit anaerobes but not vaginal lactobacilli. Metronidazole, an antibiotic with excellent activity against anaerobes but poor activity against lactobacilli, is the drug of choice for the treatment of BV. Source: Hacker Moore Page 277, sixth edition 349. Pregnant 8 week vaginal bleeding os closed fetus seen by US? A-complete B-incomplete C-threatened D-septic The A, is: C Explanation: Vaginal examination at this stage usually reveals a closed cervix. Approximately one-third of pregnant women have some degree of vaginal bleeding during the first trimester. Source: Hacker Moore Page 80, sixth edition 350. . Pregnant lady presents with cough of white sputum then cough increased in severity and she developed SOB she is allergic to dust, mites and pollens what is the most imp investigation? A- Spirometry B- Chest X-ray C- CT The A, is: A Explanation: he laboratory evaluation of a patient with suspected asthma is predominantly focused on pulmonary function testing. Other laboratory studies, including chest radiography, blood tests, and tests for allergy, are useful in selected patients but cannot of themselves establish or refute a diagnosis of asthma. The spirometric evaluation of asthma in pregnant patients is similar to that of non-pregnant patients, as airway mechanics do not change significantly during pregnancy. Note: All of this is based on the possibility of asthma diagnosis in the patient. Source: https://www.uptodate.com/contents/asthma-in-adolescents-and-adults-evaluation- and- diagnosis?search=asthma&source=search_result&selectedTitle=2~150&usage_type=default &display_rank=2#H4 https://www.uptodate.com/contents/asthma-in-pregnancy-clinical-course-and-physiologic- changes?search=azthma%20in%20pregnancy&source=search_result&selectedTitle=2~150& usage_type=default&display_rank=2 351. Female with fever, tenderness, abdominal pain, dx? A- Cervicitis B- Salpingitis The A, is: B Explanation: Pelvic inflammatory disease (PID) refers to acute and.subclinical infection of the upper genital tract in women, involving any or all of the uterus, fallopian tubes, and ovaries

A. admit for observe B. follow up 4wks C. admit for evacuation The

is: N/A Explanation: Women with an incomplete, inevitable, or missed abortion can be managed with surgical uterine aspiration or medication uterine evacuation or with expectant management. All three approaches have similar efficacy, and the choice of treatment method depends mainly upon patient preference. Note: surgical evacuation is required for women with heavy bleeding or intrauterine sepsis in whom delaying therapy could be harmful. It is also required for women with contraindications to misoprostol . Source:https://www.uptodate.com/contents/spontaneous-abortion- management?search=Missed%20Abortion&source=search_result&selectedTitle=1~32&usag e_type=default&display_rank=1&id=spontaneous-abortion-management&languageCode=en 289. Pregnant asymptomatic with urine analysis showing bacteriuria diagnosis? A. Asymptomatic bacteriuria The A, is: A Explanation: For asymptomatic women, bacteriuria is formally defined as two consecutive voided urine specimens with isolation of the same bacterial strain in quantitative counts of ≥105 colony-forming units (cfu)/mL or a single catheterized urine specimen with one bacterial species isolated in a quantitative count of ≥102 cfu/mL [2]. In clinical practice, however, only one voided urine specimen is typically obtained, and diagnosis (and treatment initiation) is made in women with ≥105 cfu/mL without obtaining a confirmatory repeat culture. Source:https://www.uptodate.com/contents/urinary-tract-infections-and-asymptomatic- bacteriuria-in- pregnancy?search=Asymptomatic%20bacteriuria&source=search_result&selectedTitle=2~6 9&usage_type=default&display_rank=2#H253489103 290. Pregnant with asymptomatic bacteriuria , allergic to penicillin and sulfa drugs? A. Nitrofurontoin The A, is: A Explanation: Asymptomatic bacteriuria is treated with an antibiotic tailored to the susceptibility pattern of the isolated organism, which is generally available at the time of diagnosis. Potential options include beta-lactams, nitrofurantoin, and fosfomycin. Source:https://www.uptodate.com/contents/urinary-tract-infections-and-asymptomatic- bacteriuria-in- pregnancy?search=Asymptomatic%20bacteriuria&source=search_result&selectedTitle=2~69 &usage_type=default&display_rank=2#H6 291. Patient come to ER hx of abruptio placenta hemodynamically unstable what your management? A. Transfer to ICU and call obstetric B. Shift to other center after give tow unit of blood C. It is an emergence, contact multispecialty team the A, is: C Explanation: Unstable mother — Cesarean delivery is the best option when vaginal delivery is not imminent and rapid control of bleeding is required because of maternal hemodynamic instability or significant coagulopathy. Blood and blood products for correction of hypovolemia and coagulopathy should be replaced prior to and during the cesarean delivery. Note: Thus, a multispecialty team choice makes more sense. Source:https://www.uptodate.com/contents/placental-abruption-management-and-long-term- prognosis?search=abruptio%20placenta&source=search_result&selectedTitle=2~150&usage _type=default&display_rank=2#H1389223030 292. 50 years pt. Uterine fibroid 6x5 what is your management ? A. Regular follow up with US yearly B. Regular follow up with us and CBC every two month The A, is: A Explanation: Since symptoms tend to stabilize or regress once an individual reaches menopause, perimenopausal patients can typically be managed expectantly. Postmenopausal individuals with fibroids should be evaluated periodically to ensure that the fibroid or fibroids have not increased in size. Source: https://www.uptodate.com/contents/uterine-fibroids-leiomyomas-treatment- overview?search=Uterine%20fibroid&source=search_result&selectedTitle=1~150&usage_ty pe=default&display_rank=1#H4210734308 293. Pregnant with Crohn's disease on immunosuppressant med, asking about her future baby vaccine, what to do ? A. Delay vaccine untill 6 month The A, is: A Explanation: the recommendation that live vaccines are contraindicated for the first six months in newborns with in utero immunosuppressant exposure. Source: https://www.ncbi.nlm.nih.gov/pmc/articles/PMC4155388/ 294. Pregnant with signs of anemia, what is the common cause ? A. Iron def anemia B. Thalassemia C. B12 dee The A, is: A Explanation: Physiologic anemia of pregnancy and iron deficiency are the two most common causes of anemia in pregnant women

examples include placenta previa and previous classical cesarean delivery. In this question the type of CS is not determined, but it is not common nowadays to see classical CS, so probably the

is B, especially if there are recurrent variable deceleration. Based on expert opinion, ECV is generally also contraindicated in the following settings because they are associated with a low likelihood of successful version and/or increased risk of fetal harm from the procedure: • Severe oligohydramnios. • Nonreassuring fetal monitoring test results. • Hyperextended fetal head. • Significant fetal or uterine anomaly (eg, hydrocephaly, septate uterus). • Placental abruption. • Ruptured membranes. • Active labor with fetal descent. Multiple gestation. However, internal or external version is widely considered an acceptable approach for the second twin after delivery of the first twin. Source: UpToDate 483. What of these is a contraindication to ECV: a. Amniotic fluid index of 22cm b. Low lying placenta c. Smoking 10 cigarettes daily d. Umbilical cord length of 35-38cm A, is B Explanation: AFI of 22cm is considered normal (abnormal is above 23 or 25 depending on the resource used), smoking is irrelevant of ECV, unless it caused a condition that is a contraindication to ECV, and umbilical cord length Any absolute contraindication to labor and vaginal birth regardless of fetal presentation is an absolute contraindication to ECV

however, polyps that are removed should be submitted to the laboratory for histological study. Source: UpToDate, Hacker & Moore's essentials of obstetrics & gynecology, page 254 498. Patient with active vaginal bleeding, what is your first line management? a. D&C b. Iv conjugated estrogen c. Endometrial ablation d. Hysterectomy

is B Explanation: the treatment for acute excessive bleeding in nonpregnant women of reproductive age is generally medical and it includes hormonal manipulation. Keep in mind that conjugated estrogen is an old way of treating this, newer therapies include progestin only therapy as a first line. When a woman with heavy uterine bleeding does not respond to the initial therapy within 12 to 24 hours, or she is hemodynamically unstable, more invasive therapies are indicated. A dilation and curettage is performed to remove the remaining endometrium. When needed, a balloon may be placed within the uterine cavity to tamponade bleeding vessels, and this might be the initial step of dealing with abnormal uterine bleeding, in hemodynamically unstable women, so that could be the A, of other questions (unless there is ABC A, or something like that). Selective embolization of uterine blood vessels can be done by an interventional radiologist if persistent active bleeding continues. As a last resort, and only rarely, hysterectomy is indicated Source: UpToDate, Hacker & Moore's essentials of obstetrics & gynecology, page 324-325 499. Patient with hx of surgery removal of endometriosis and came with recurrence in other ovary, she completed her family, what is your management? a. Mass removal b. Hysterectomy and ovarian removal A, is B Explanation: Definitive surgery is offered to women with debilitating symptoms that are likely from endometriosis, have completed childbearing, and have failed other treatment options. Definitive surgery includes hysterectomy, with or without oophorectomy. The main disadvantages of definitive surgery are the surgical risks, loss of fertility, and symptoms of surgical menopause. This question is incomplete in regard of previous management methods, and patient views on the surgery, If there is no clues or hints in the question that suggest that surgery isn't the favorable method, then our A, is the correct one. There are more conservative approaches but ablation of certain endometriotic lesions, but this is usually associated with recurrence, and it is favorable in women who still want to preserve their fertility or hormone production. Source: UpToDate 500. Patient with lower abdominal pain, amenorrhea, with rigid abdomen, tenderness on palpation, the patient was hypotensive and pregnancy test was positive, what you will do? a. Refer to GS b. Do surgical intervention after stabilizing her c. Medical A, is B Explanation: the question might be incomplete, but if this was an ectopic pregnancy question then the A, is B Source: Toronto notes 2020, page 21 501. Pregnant 32 weeks with HTN and proteinuria, what is your management? a. Mg, stabilize and postpone delivery b. Delivery now c. C/s A, is not clear Explanation: there is no clear reason for giving Mg, and there is no clear indications for delivery now, or cesarean delivery. The A, is incomplete and probably missing a lot of important information, read about management of preeclampsia in preparation of similar questions. This question was probably about a patient with headache, which makes the A, A, since we need to stabilize the patient and try to push the delivery to beyond 34 weeks. Turky Mehssen Alyamy 35 2 1 2 0005 Ethics/Research/Miscellaneous 2. Female abused by her husband and looks sad: A. Introduce yourself with a smile and take history B. Try to figure out what happened and the reason of sadness C. Ask about the chief complaint A,: A. 3. A 7-year-old girl appears underweight and neglected. When asking the mother, she complains that she has other younger children to look after that she is busy with: A. Contact social worker B. Refer mother to psychiatrist A,: A 4. Example of moderate reinforcement teacher should do: A. Focus on good A,s (things) the child do B. Give him moderate tasks A,: A. 5. A child that screams and hits and always says no, what to do? A. Hit B. Positive reinforcement A,: B. 6. Perforated appendicitis post laparotomy, presented with fever, abdominal pain CT done showed foreign material , re-exploration was done, gauze was found intraabdominal. What to do? A. apologize to the patient and tell her what happened A,: A. 7. Patient worried and wants to do hysterectomy and be admitted. What would you do? A. Admit her to gain confidence B. Calm her down and discuss her options A,: B. 8. 15-year-old patient with (acute meningitis) and you want to take lumbar puncture (LP), from whom will you take consent? A. From parents B. From the patient C. From both D. No need to take consent as it is an emergency case A,: A. Consent the parents, Ascent the Child 9. Pregnant physician has a preterm baby with multiple congenital anomalies, and she asked not to resuscitate her baby. What will you do? A. Respect her decision B. Refuse her request and resuscitate A,: A. 10. A patient wearing revealing clothing and talks in a seductive way and the nurse doesn't speak her language. What would you do? A. Bring a nurse that speaks her language B. Kick her out of the clinic C. Call security D. Refer her to another doctor A,: A. 11. Refuse c-section with risk of fetal death A. VD B. Refer to another doctor C. Hospital committee A,: C. 12. A 40 years old patient whose surgeon told him that he needs to do amputation, but the patient refused. What to do? A. Assess the mental capacity of the patient B. Informed consent DAMA A,: B. 13. Physiotherapist of stroke patient saw darkening of the patient buttock, and told the nurse to tell the resident and then she forget, the patient developed infected ulcer? How to prevent this incident from happening? A. Good communication A,: A. 14. Ob/Gyne question that I forgot, and what ratio should be used A. Odds ratio B. Relative ratio A,: ? Know How to Calculate Both 15. Couple came to infertility clinic, history of 10 year infertility, found that wife has bilateral tubal blockage. Whom to tell? A. Couple B. Husband C. Wife D. Father A,: A. 16. You have a schizophrenic patient on medication , and asked him (do you think you are intellectually ill)? A. Judgement B. Insight A,: B. 17. During counseling with patient's family, doctor said "I'll discuss the diagnosis with u, I'll clarify the treatment options, that said any question to be A,ed"". What did he do? A. Probing B. Agenda setting C. Reflection D. Summarization A,: B. 18. Elderly man ICU and ask not to resuscitate him, then he died, and his son was angry. What will you do? A. Tell him that you respected his father wish B. Ask ethics committee C. Tell him we do not resuscitate braindead patients A,: A. 19. Patient DNR by an intensivist, the thoracic surgeon want to do debulk surgery for the patient as it will help in maintaining his life, but there is high risk for dying in the surgery what to do? A. Inform the patient and take high-risk consent B. Discuss the case with the intensivist C. Talk to the hospital committee D. Do not do the surgery as the patient is DNR A,: A. 20. What is the most important feature in a test if you want to make a diagnostic test? A. Sensitivity B. Specificity A,: B. 21. Elderly patient admitted as a case of pneumonia in the night she developed agitation and irritability. What should nurse do? A. call the doctor to give lorazepam B. Calm the patient and tell her where she is and what for C. Call her family to send a family member to stay with her A,: B. Anther recall: copd patient same scenario what shell be done first: elevate head + nasal oxygen 22. A doctor prescribed medication to the patient but the pharmacist thinks that the patient is allergic to it. What would the resident do? A. Ask him to sign it A,: A. 23. Consultant told resident to write piperacillin Tazobactam and pharmacist said px had? Penicillin allergy. What should the resident do? A. Refused to write B. Write then inform chief registrar. C. Write and ask consultant to sign. D. Refuse to write A, is: A "Refuse gently" 24. Gynecologist explained to the patient that hormonal replacement therapy will not help her menopausal symptoms at this stage, but the patient insisted to have. What should the doctor do? A. Do not give it to her A,: A. 25. Couple with infertility ask about surrogate pregnancy you are not sure whether it is allowed in Saudi Arabia or not: A. Warn them it might not be allowed B. Another appointment and search about it C. Refer to another doctor D. Tell them it is not allowed in this hospital A,: B 26. Consultant at OR and ask to prepare right kidney for surgery but the intern read in notes it is left kidney what is the most appropriate action by intern? A. Inform the consultant A,: A. 27. Case of (embola?) want to discharge A. Accept B. Refuse C. Consult ethic D. Call security A,: D. 28. Pregnant lady with husband fell in the abdomen, everything is normal and no placental abruption. What will you do? A. Admission and report social worker A,: A. 29. Female with abnormal uterine bleeding needed endometrial sampling. What is the prerequisite of this procedure? A. Verbal consent B. Setting up the room A,: A. Recall: Post menopausal have bleeding 2 month. do endometrial biopsy Thickness 15mm and you want to do hysterectomy next ? A- Take verbal concent B- written consent A,: B 30. Young girl was complaining of abdominal pain. Physical examination was normal. Blood and urine tests were normal. The mother mentioned it's mostly behavioral. Which of the following is considered a positive action? A. More TV time B. Focus on monitoring positive actions C. Document all bad behaviors A,: B. 31. Foreign patient presents to the emergency department for delivery. She is anxious and fears for her baby. A. Call another doctor B. Try to understand her fears C. Say her fears cannot be understood because of language barrier A,: B. 32. Internal medicine resident did diabetic foot examination, but she forgot to close the curtains when she exposed the patient's abdomen. What is the ethical principle that she broke? A. Privacy B. Dignity C. Autonomy D. Confidentiality A,: A. 33. If you want to do cancer screening you choose A- highly sensitive B- highly specific C- cheap test A,: A 34. Patient with irregular cycle and take OCP before marriage, after marriage the husband want baby but she refused ? a-Mutual consent on OCP b-Be with women c-With husband d-Show empathy A,: A 35. Patient refused amputation: A-Accept and DNMR A,: A 36. Child with spinal muscular dystrophy, his condition now mandates intubations but parents refused : a- Intubate anyway b- Respect their wish c- Consult ethical committee A,: A 37. Father don't believe in vaccines his son develop infection measels? A. tell him about the benefit and evidence of vaccine 38. Case about hiv mother's what to tell her A. To do CS to avoid HIV transmission 39. Asthmatic child in ER with asthma attack and father carless smoker what important thing ? a) Ignore father and treat child b) Child protection A,: B. 40. Female with incomplete abortion, her husband wants to sign for surgery and she is refuse.. Who should sign for a consent? a) Husband b) Family Patient Doctor if there is wife chose it.. 41. Female come to emergency department with bruises multiple area. Which indicate domestic violence ? A. Second marriage B. Tobacco A,: A. 42. Recall: Best predictions of domestic violence: A. Smoking B. Second marriage C. Personal? D. Marital dissatisfaction A, is: D 43. Another form: when does Violence increase During Pregnancy then in the holiday 44. Couple came for infertility after test show the husband have HIV , what to do? * informs the husband 45. Recall: couple came in the fertility clinic, husband found to have HIV. A. tell the husband B. tell wife C. Tell a close relative. D. no tell the couples in the choices The A, is: A If patient is HIV ve+, tell him to till his wife but the doctor should tell the authorities. 46. But If pt say plz don't tell my wife, then what you will do? a) You shouldn't tell her b) Tell health authority's only Note: a guy from SCFHS gave a lecture and brought this Q, and he said the right A, is A 47. Scenario during surgery you injured CBD and the patient is ok what to do ? * informs the patient and apologize * 48. Scenario of suspicion Appendicitis and during surgery appendix is normal , but you have to remove it even though it's normal , what to do? Tell the patient 49. Patient forced you to do the surgery and threatened you? Inform the administration 50. While you came to the clinic late the patient is angry? Explore the reason of being angry 51. Couple Post coital ( after intercours) bleeding , they are sad A. Acknowledge their emotions 52. Child abuse: a. call child protection team b. call ethical committee A,: A. 53. Child born extra marital relationship ,abandoned you need to do hernia repair ( consent) If stable -> hospital consent If emergence -> do 54. Child with BA and his father smoking? Explain to father if refuse call child protection 55. What are the 4 principles for effective reporting in child abuse? A- Observe, document, report, self-protection B- Observe, document, referrals, follow up C- Document, confirm, report, conference parents A,: A. 56. Patient aborted (Ethics) what to do next A. show empathy and talk about management B. show empathy and explore emotion A,: most likely B. 57. A doctor after explaining to family that their baby has CF, he repeat to them the management and prognosis and their concerns, what the doctor did? A. Summarizing B. Mirroring Forgot other options A,: A. 58. 38 w with IUFD, with Hx of cs 4 y ago, what to do? A. Let pt decide B. Tell pt benefit of SVD than CS D. do immediate CS and don't let pt become more sad A, is B 59. screening program of a rare disease, the pt who tested positive in screening what is the significance of having this rare disease? A. True positive to have the disease B. False positive C. True negative D. False positive A,: B. Note: Low prevalence: - If asked what they have: They will have: High true negative, High false positive If asked what test you should use: The test for them should be High in sensitivity (inherently has high true positive rate) to overcome the population obstacles If asked what test will have bad results in them: positive predictive value 60. Patient diagnosed with bronchiectasis how to tell him about the diagnose A. show him the report B. tell him he has bronchiectasis C. explain hr has damaged alveolar with infection A,: C. 61. Patient worried and want to do hysterectomy and go br admited A. admit her to gain her confidence B. calm her down and discuss her options 62. The prevalence of the disease affect which value? A. Sensitivity B. Specificity C. Likelihood ratio D. Predictive value A,: D. 63. Patient came to your clinic has COPD and refused to stop smoking what should you tell him A. Motivation him to stop B. Call his family C. Tell him not to come again A,: A. 64. A doctor sits with the family tells them the diagnosis and the prognosis, after that he will listen to their qs and A,s them, what does this called? A. improving communication B. give information care A,: A. Patient had cancer family ask doctor to give him a high dose of analgesic but the doctor said it will have adverse effect! What kind of something he did have no clue about this ? A. totality B. double effect C. informed Note : A, is B Principle of double effect If he has cancer or critically ill 65. Long scenario: Radiologist asked by the treating team to do CT guided Fluid aspiration. Who should get the informed consent? A. Radiologist B. The resident from the treating team C. The nurse A, is A 66. Moms gave birth to baby who died in uterine, she was asking why he was not in the nursery. What dose she have : A. Denial B. Acceptance C. Grief A, is A 67. About a male patient have Charcot knee, doctors decided that he need amputation, he refused ! , what is your action: A- let him sign DAMA B- contact hospital ethical committee/guidelines C- others i forget unfortunately A, is A 68. Mother brought her sick son, he is for operation, the mother is afraid but not talking Arabic, she looks anxious and worried because she lost her previous kid from the same condition, you are the intern what should you do: A. show her empathy that you understand her anxiety. B. Ignore as she don't speak your language A, is A 69. Child not talking Arabic, before operation he seems very anxious, how should you act as an intern: A- auscultate his chest then let him auscultate yours 70. Pregnant 20+ weeks with a twisted ovarian cyst you explain she needs a laprotomy to save the baby's life and she refuses what to so? A- Accept her refusal after obtaining DAMA B- ignore her and proceed with the surgery C- take permission from her husband D- call the ethical committee A,: A. 71. Ovarian torsion case. She refused emergency surgical intervention. What should you do? A. Refuse B. Respect A,: B 72. BREAKING CONFIDENTIALITY TO PREVENT HARM TO OTHERS: An exception to the privacy rule is in the circumstance of protecting other people. Relatives, employers, friends, and spouses Other physicians: If a physician seeks medical information about a patient, you cannot release it without the express consent of the patient. Members of law enforcement: You cannot release medical information to courts or police without a court order or subpoena. If a patient has a transmissible disease, such as tuberculosis or HIV, the physician can violate the patient's confidentiality to protect innocent third parties. 73. Patient came to your clinic regarding consent for hysterectomy. What will you do? A. Document B. Explain the details of the surgery A, : b 74. Patient wants you to do his surgery forcefully and threatens you A. do his surgery B. refuse to do it C. call the admin A, : call security if it is an option if not C 75. Female patient want female doctor to treat her because she is embarres about her condition. what will you do? A- Respect her wish A,: A 76. While surgeon explaining the details of the surgery and possible complications he was Interrupted by the patient telling him she didn't want to know anything about the surgery. The surgeon respect her wish. But the anesthesiologist was willing to tell her about the possible complications. What the surgeon should do? A- Let the anesthesiologist what he want B- Refer to another anesthesiologist The correct A, is: A 77. Child diagnosed with asthma, father smoke and don't care, what will you do? A- Report child protection B- Explain that smoking affect their child wellbeing The correct A, is: A

however you would see early decelerations. Uteroplacental insufficiency is associated with variable decelerations. Source: Hacker & Moore's essentials of obstetrics & gynecology, page 133, Toronto notes 2020, page OB34, First Aid USMLE step 2 CK 10th edition, page 329. 372. Bacterial vaginosis? A. Increase lactobacilli B. Epithelial cells

is B Explanation: this is a description of clue cells, which are vaginal epithelial cells covered with Gardnerella. Bacterial vaginosis is associated with fishy odor, and it is not a sexually transmitted disease but rather is associated with sexual activity. Usually vaginal PH is elevated to more than 4.5 in bacterial vaginosis. A is wrong because usually the disruption of the number of Lactobacillus is what causes bacterial vaginosis, and usually it is due to decrease in their numbers and not increase. Source: First Aid for the USMLE STEP1 2019, page 148, Kaplan lecture notes: Obstetrics and gynecology 2019. 373. Asthmatic pregnant what to tell her about her medication? A. They are safe in pregnancy you can continue taking it No other A,s were provided Explanation: asthma drugs are not teratogenic agents and can be safely taken during pregnancy, Uncontrolled asthma is associated with adverse events on pregnancy (increased risk of perinatal mortality, pre-eclampsia, preterm birth, low-birth-weight infants). Poorly controlled asthma or asthma exacerbations may have a greater fetal/maternal risk than what is associated with appropriately used asthma medications. Optimal control of asthma prior to and during pregnancy is recommended. Apart from systemic glucocorticoids, inhaled glucocorticoid Short acting or long-acting β2-agonist have not been associated with any significant fetal complications. Source: First Aid USMLE step 2 CK 10th edition, page 322, Hacker Moore Page 218, sixth edition 374. Female come to er with bruises in multiple area which of the following can indicate domestic violence case? A. Second marriage B. Tobacco A, suggested by the editor was A Explanation: we don't have other A,s to compare these A,s to. Factors in the history which might raise the suspicion of a domestic violence include: • An inconsistent explanation of injuries. • Delay in seeking treatment. • Frequent emergency department or urgent visits. Typically abusers do not want their victims to form an ongoing allegiance with one clinician. They may feel the victim will be less likely to find an ally in an emergency department where care may be more fragmented. • Missed appointments. The patient may not keep appointments because the abuser will not allow medical attention. In one study, 17 percent of victims of IPV felt that their partner interfered with their access to practitioner visits, compared with 2 percent of those not suffering from abuse. • In pregnancy, late initiation of prenatal care. • Repeated abortions. Unplanned pregnancy may result from sexual assault and/or not being allowed to use birth control (reproductive coercion). • Medication nonadherence. Victims may not take medicines because the batterer has taken them away or not allowed the partner to fill prescriptions. • Inappropriate affect. Victims may appear jumpy, fearful, or cry readily. They may avoid eye contact and seem evasive or hostile. A flat affect or dissociated appearance may suggest posttraumatic stress disorder. • Overly attentive or verbally abusive partner. The clinician should be suspicious if the partner is overly solicitous or A,s questions for the patient. If the partner refuses to leave the examination room, the clinician should find a way to get the partner to leave before questioning the patient. Partner reluctance to leave the patient alone is an important sign. • Apparent social isolation. Reluctance to undress or have a genital or rectal examination, or difficulty with these examinations. Source: uptodate. 375. Couple after 2 years failed to get pregnant, husband sperm normal what is the cause of infertility A. Vaginal factor B. Coital factor C. Husband factor A, is not clear. Explanation: the question is incomplete, we do not know what all the A,s are, in general after excluding a male factor by doing semen analysis, we look for ovulation factors since they are the most common cause of female infertility. Other tests might include assessing the fallopian tube patency and uterine cavity. Source: uptodate, Hacker & Moore's essentials of obstetrics & gynecology, page 397. 376. Us showing snowstorm appearance, B-hcg = 80,000 A. Complete hydatidiform mole B. Incomplete hydatidiform mole C. Choriocarcinoma A, is A Explanation: pelvic US revealing a snowstorm appearance without a fetus and elevated B-hcg are characteristic findings in gestation trophoblastic disease particularly complete molar pregnancy, since there was no mention for findings of fetal tissue. Complete mole findings on US: - Absence of an embryo or fetus - Absence of amniotic fluid - Snowstorm appearance also called swiss cheese pattern previously, The small cystic areas correspond to the hydropic villi, which are one of the characteristic gross and microscopic pathological findings. - Ovarian theca lutein cysts Incomplete mole findings on US: - Fetus might be identified - Amniotic fluid is present, but the volume may be reduced - Placenta with one or more abnormal findings: o Enlarged, cystic spaces ( swiss cheese pattern) o Increased echogenicity of chorionic villi - Increased transverse diameter of the gestation sac - Theca lutein cysts are usually absent Source: First Aid USMLE step 2 CK 10th edition, page 343, uptodate. 377. A woman with smelly vaginal discharge and itching. Her husband has a Hx of urethral discharge, asking about treatment? A. Progesterone creams B. Ceftriaxone C. Estrogen cream D. Defer all vaccines A, is B Explanation: based on the provided A,s it seems like it is a question about gonorrheal infection. The treatment for gonorrhea is ceftriaxone, but since there is high rates of co- infection with chlamydia you always cover that with either azithromycin or doxycycline. Source: Toronto notes 2020, page GY27 378. 37 years old pregnant female came with severe placental abruption +DIC+IUFD cervical dilatation 6cm What u should to do? A. Observation B. Augmentation of labor C. CS D. Hysterectomy E. Defer all vaccines A, is not clear Explanation: the A, depends on if the mother is hemodynamically stable or if she's not. If the mother is not stable then cesarean delivery is indicated to save the mother's life. If the mother is stable with dead fetus such as in this case, then the goal is to minimize maternal morbidity and mortality, and this means in many cases avoiding cesarean delivery. Source: uptodate, Toronto notes 2020, page OB15 379. Patient k/c of endometriosis, she was on progesterone for withdraw blood, now the patient has stopped the drug, she is at risk of what of the following complications? A. Endometrial cancer A, is not clear Explanation: the question is probably incomplete and written poorly, probably the question had some sort of unopposed estrogen excess that was mentioned here, that increased the risk of getting endometrial cancer. 380. 40y/o female patient underwent PAP smear histopathology showed ASCUS, your next step? A. Do HPV test B. Colposcopy C. Re-evaluate after 6 months D. Surgery A, is A Explanation: Source: Toronto notes 2020, page GY44 381. Female patient married, k/c of type 1 DM on insulin, came with her husband for counseling as they're planning to have a baby in next two years, her labs showed high FBG and hba1c was 8, what's your advice? a. Try to make hba1c as low as possible but safely A, is A Explanation: the question is incomplete, but the provided A, makes sense, reduction in mothers hba1c before pregnancy is associated with reduction in rates of congenital malformations and miscarriages Source: UpToDate 382. Picture of CTG showed sinusoidal pattern, what's the most likely cause? a. Fetal anemia b. Uteroplacental insufficiency A, is A Explanation: Sinusoidal pattern can be seen in severe fetal anemia. a pseudosinusoidal pattern may also occur during maternal meperidine use. Source: First Aid USMLE Step 2 CK - 10th Ed, P327 383. Picture of CTG showed late deceleration, what is most likely cause? a. Uteroplacental insufficiency b. Head compression c. Umbilical cord compression A, is A Explanation: Source: First Aid USMLE step 2 CK 10th edition, page 329. 384. There were questions about External cephalic version (ECV), but we don't have the question stem. 385. There were questions about type of abortions, but we don't have the question stems. Explanations: Source: First Aid USMLE step 2 CK 10th edition, page 325 386. Patient with Amenorrhea, lab high FSH, LH what is she at risk for? a. Ovarian cancer b. Endometrial cancer A, is B Explanation: assuming that this is a PCOS patient with high LH/FSH ratio, the A, is endometrial cancer. Keep in mind that FSH level is not elevated in PCOS, but maybe this is a mistake by whoever have written the question. Both LH and FSH are elevated in menopause women, and they are at the risk of osteoporosis, but this wasn't one of the mentioned A,s, for this reason this is probably PCOS with high LH/FSH ratio. Source: Toronto notes 2020, page GY24 387. Patient C/O missed period for 2 month and Rt leg edema What important question? a. Hx of using COCP No other A,s were provided Explanation: read about causes of amenorrhea 388. Female complaining of amenorrhea for several months with hx of D&C procedure what is name of this syndrome? b. Asherman's syndrome c. Sheehan syndrome A, is A Explanation: Endometrial ablation procedure destroys the endometrium by heat, cold, or microwaves. It leads to an iatrogenic Asherman syndrome with intrauterine adhesions and minimal or menstrual blood loss. Fertility will be affected. Source: Kaplan lecture notes: Obstetrics and gynecology 2019 389. Women with midline bulge known as multiparity a. Diverticula recti b. Weakness in transverse abdominis muscle c. Incisional hernia A, is A Explanation: this is another name for rectus abdominis diastasis which is a condition in which an abnormally wide distance separates the two rectus muscles. However, there is controversy regarding what constitutes a normal inter-rectus distance, at what level measurements should be taken, and by what means, and thus when, the distance can be considered abnormal. Uptodate considers any separation more than 2 cm to be abnormal. However, it is possible to have clinically evident diastasis recti when the inter-rectus distance is less than 2 cm. Pregnancy increases the risk of developing RAD. Rectus abdominis diastasis (RAD) may or may not be associated with symptoms. Excessive abdominal muscle separation can compromise the function of the anterior abdominal wall. RAD is usually quite apparent on physical examination. When a patient with RAD raises their head and begins to sit up, the increase in intra-abdominal pressure as the two rectus muscles contract can result in a diffuse fusiform bulge, often with a protrusion of abdominal contents into the thinned, bulged midline fascia, which can be seen as a prominent ridge extending from the xiphoid to the umbilicus. However, protrusion of abdominal contents may not occur. But, RAD does not represent an abdominal wall hernia

examples include placenta previa and previous classical cesarean delivery. In this question the type of CS is not determined, but it is not common nowadays to see classical CS, so probably the

is B. C could be an A, in different settings, but as an absolute contraindication we think B is the most appropriate A,. Based on expert opinion, ECV is generally also contraindicated in the following settings because they are associated with a low likelihood of successful version and/or increased risk of fetal harm from the procedure: • Severe oligohydramnios. • Nonreassuring fetal monitoring test results. • Hyperextended fetal head. • Significant fetal or uterine anomaly (eg, hydrocephaly, septate uterus). • Placental abruption. • Ruptured membranes. • Active labor with fetal descent. • Multiple gestation. However, internal or external version is widely considered an acceptable approach for the second twin after delivery of the first twin. Source: UpToDate, Toronto notes 2020, p868 429. Case of galactorrhea and irregular period what you are going to order first? a. Progesterone b. Prolactin A, is B Explanation: Source: Toronto notes 2020, page E18 430. Case of PCOS patient, with increased endometrial thickness, what is the cause of that finding? a. Unopposed estrogen No other A,s were provided Explanation: estrogen causes endometrial proliferation, and due to the unopposed estrogen secretion in PCOS, patients have increased long term risk of endometrial cancer. Source: First Aid USMLE step 2 CK 10th edition, page 368 431. Pleural effusion, ascites, and ovarian tumor? a. Refer for gynae oncology There were no other A,s provided Explanation: The question is incomplete, but this presentation is consistent with Meigs syndrome which is a triad of ovarian fibroma, ascites, and pleural effusion. On occasion, ovarian fibroma is associated with ascites caused by the transudation of fluid from the ovarian tissue. The flow of this ascitic fluid through the transdiaphragmatic lymphatics into the right pleural cavity. Read about the management of benign ovarian neoplasms. Most benign-appearing ovarian cysts may be observed and followed with ultrasonography. If they are symptomatic or enlarging, laparoscopic management is usually appropriate. Source: Hacker & Moore's essentials of obstetrics & gynecology, page 262-263 432. Ovarian mass what to do to confirm the diagnosis? a. Histopathological study No other A,s were provided Explanation: the question is incomplete, we don't have enough information to explain anything, read about the work up of ovarian masses. Generally speaking, surgical exploration is performed if the initial evaluation results in sufficient suspicion of a malignant adnexal mass. Surgical evaluation allows a definitive histologic diagnosis. Source: UpToDate 433. Pregnant with high blood pressure in 15 weeks? a. Essential Hypertension b. Preeclampsia c. Eclampsia d. Gestational hypertension A, is A Explanation: Source: Toronto notes 2020, page OB24, First Aid USMLE step 2 CK 10th edition, page 334 434. Pregnant with blood pressure 160/110 with protein in urine what first thing to give? a. Magnesium sulfate b. Hydralazine A, is not clear Explanation: the question is incomplete, the writer of the question indicated that the A, was A, read about the topic so when faced with different clinical scenarios during the exam you can A, them, the topic is relatively simple and straightforward. 435. Pregnant in 32 weeks with pre-eclampsia what to do? a. Admit to induction of labor (IOL) b. Admit to observation A, is not clear The question is incomplete, the writer of the question indicated the A, was B. Probably the patient didn't have severe preeclampsia to induce delivery. Source: Toronto notes 2020, page OB25-26 436. Pregnant unbooked present with painless vaginal bleeding, fundal height 34 weeks what is the first thing to do, considering she lives far away and has difficulty in transport? a. Admit to ward b. Us A, is not clear Explanation: the details of the questions are incomplete, the writer of the question indicated that the A, was A, but we don't have enough information regarding why B is wrong, and what are the other options, in case this was placenta previa and the question is inpatient vs outpatient care, then in general asymptomatic women with placenta previa can be managed outpatient, findings from observational studies suggest that women who have not experienced any antepartum bleeding are at low risk of sudden hemorrhage requiring an emergency cesarean delivery for control of bleeding. In UpToDate, there are recommendation regarding some risk factors that might be taken into account when considering inpatient care and these include: • Short cervical length on transvaginal ultrasound examination (eg, increased risk for preterm labor if cervical length is ≤25 mm, and consider hospitalization if ≤15 mm) • Rapid cervical shortening (eg, >10 mm over a one- to two-week period on transvaginal ultrasound) • Inability to get to the hospital promptly (within approximately 20 minutes) • Lack of home support in case of an emergency. Source: UpToDate 437. Pregnant with horizontal lie fetus at 32 weeks, cervix dilated (the writer of the question can't remember size) with bulging fetal membrane what to do? a. Tocolytics b. Cs A, is A Explanation: we are trying to delay the preterm labor to initiate corticosteroids therapy. This is what should be done initially. Contraindications to tocolysis include: • Intrauterine fetal demise • Lethal fetal anomaly • Nonreassuring fetal status • Preeclampsia with severe features or eclampsia • Maternal hemorrhage with hemodynamic instability • Intraamniotic infection • Preterm prelabor rupture of membranes • Medical contraindications to the tocolytic drug Since our patient have intact membranes, it is not contraindicated to initiate it. Source: UpToDate 438. Pregnant at 38 weeks hx of rupture of membrane 24 hours ago, what to do? a. IOL (induction of labor) b. CS A, is A Explanation: the recommended management of patients with rupture of membrane, is to induce labor unless there are contraindications to labor or vaginal delivery, in which case cesarean delivery is performed as soon as possible. Induction of labor is necessary due to the increased risk of maternal and fetal infection if the patient was just observed and the plan was expectant management. Note that here was no IV antibiotic in choices. Source: Toronto notes 2020, page OB18, UpToDate 439. Mother can't feel fetal movement, non-stress test reactive, biophysical profile score 8 what to do ? a. Repeat biophysical profile after 24 hours b. Repeat biophysical profile after 1 week + reassure A, is B Explanation: Women who experience a brief period of decreased fetal movement followed by resumption of normal fetal activity during fetal evaluation and a normal evaluation can resume routine prenatal care. They are instructed to continue to monitor fetal movement and call their provider if they perceive recurrent persistent decreased fetal movement. Source: UpToDate, Medscape, Toronto notes 2020, page OB10 440. Mother with history of prior delivery of child with severe jaundice upon birth, what will u want to investigate in her second pregnancy? a. Maternal blood type No other A,s were provided Explanation: the question is incomplete. Source: First Aid USMLE step 2 CK 10th edition, page 341-342 441. Knee pain and swelling, joint aspiration negative needle shape (gout) what the treatment? a. Indomethacin b. Febuxostat A, is A Explanation: for acute gout the treatment is NSAIDS and not anti-hypouricemic drugs. Source: Toronto notes 2020, page RH26 442. Pregnant with symptoms of TB, what is best next step to reach definitive diagnosis? a. Sputum culture b. Sputum acid fast bacilli smear c. Chest x ray A, is A Explanation: Conventional culture is the most sensitive tool for detection of TB and can detect as few as 10 bacteria/ml

there is no fascial defect and, therefore, no risk of incarceration or strangulation. Ventral hernia can coexist with RAD, however, particularly if there has been a previous laparotomy, so differential diagnosis is important. Source: UpToDate 390. G3 P2 GA20weeks presented with history of preterm delivery twice on examination cervical is 30mm. What is the management? a. Strict bed Rest b. Cerclage now c. Progesterone

is C Explanation: since this woman doesn't have a short cervix (between 10-20 mm in hacker's and moore, and less than 24 in uptodate) she doesn't have cervical incompetence, therefore, cerclage is not indicated for her. For women with a history of preterm labor progesterone is indicated until 36 weeks of gestation. Lastly bed rest is only effective as a management of a patient coming now with a preterm labor it is not effective in prevention of future preterm labor. We are assuming this case is asking about prevention of preterm, and not initial management of a patient coming to ER with preterm labor. Source: Hacker & Moore's essentials of obstetrics & gynecology, page 158, Toronto notes 2020, page OB16, UpToDate 391. What vaginal infection can cause incompetency? a. Bacterial vaginitis b. Candida c. Trichomoniasis A, is A Explanation: infection can cause acute cervical incompetence, however, we found multiple organisms leading to this, and there was no clear A, for this question, the only thing we could find is the most common of them all is A. Source: UpToDate 392. Late decelerations reflect? a. Metabolic acidosis b. Respiratory acidosis c. Asphyxia A, is C Explanation: Keep in mind that metabolic acidosis will result from the uteroplacental insuffcicieny or severe enough hypoxemia, asphyxia implies hypoxia of sufficient degree to cause metabolic acidosis Source: First Aid USMLE step 2 CK 10th edition, page 329. 393. There were questions about placenta previa but we don't have the question stems. 394. Risk factor for endometriosis or endometrial cancer: a. Nullipara b. Low weight A, is A Explanation: nulliparity is a risk factor for both conditions. Source: Toronto notes 2020, page GY11 and GY38 395. What is true regarding endometriosis: a. Not related to any cancer b. Related to ovarian cancer c. Related to thyroid cancer A, is A Explanation: the classical teaching it is not associated with any malignancy, and the A, is most likely is A, however, know that endometriosis appears to be associated with some epithelial ovarian cancers (EOC). While there appears to be an association between endometriosis and EOC, endometriosis is not considered a premalignant lesion, and screening is not recommended. There are no data indicating that prophylactic removal of endometriosis lesions reduces the risk of EOC. So if they ask about what specific malignancy that is associated with endometriosis, then the A, is ovarian, if they ask similar question to the one we have here, then we think the A, will not be that it is associated with malignancy. Source: UpToDate, Toronto notes 2020, page GY11 396. Pregnant Leopold maneuver 1st and 2nd finding: soft not hard, fetal heart beat at umbilicus presentation: a. Face b. Brow c. Shoulder d. Breech A, is D Explanation: The first maneuver involves palpating the fundus to determine which part of the fetus occupies the fundus. The head is round and hard, whereas the breech is irregular and soft. I found no strong source for the following information: If you hear the heartbeat loudest below the mother's bellybutton, the baby is probably head down. If you hear the heartbeat loudest above the mother's bellybutton, the baby may be in the breech position Source: Hacker & Moore's essentials of obstetrics & gynecology, page 92 397. Pregnant with asymptomatic UTI, the question did not mention any weeks or anything Tx: a. Nitrofurantoin b. Ciprofloxacin c. Cephalosporin d. Augmentin A, is A Explanation: in UpToDate, amoxicillin, and nitrofurantoin are viable options, in Toronto notes, first line was amoxicillin. Now in this question they mention Augmentin which is amoxicillin + clavulanic acid, we chose nitrofurantoin as the most probable A,. Source: UpToDate, Toronto notes 2020, page OB29 398. Married 3 years ago with irregular menses and milk expressed manually, next step: a. B-HCG b. Prolactin A, is B Explanation: this is a difficult question if there is no other information, as pregnancy should be expected in all women in relationships and who are fertile. The irregular menses is another symptom that is hard to determine since cessation of menses can be a difficult symptom to evaluate because some women have irregular menstrual cycles and many women have occasional prolongation of a cycle. Furthermore, vaginal bleeding/spotting is relatively common in early normal pregnancy and often occurs at or near the time that a menstrual period would be expected, so it is not necessary for a woman to have amenorrhea at the start of pregnancy. Another point is the secretion of milk, usually the effect of the physiological increase of prolactin during pregnancy is suppressed by progesterone so there shouldn't be any milk production. We chose B as the most probable A, when considering all these points, however, during the exam, there might be other information that will lead you to choose the proper A,, such as, inconsistent condom use, visual changes, other pregnancy symptoms Source: Toronto notes 2020, page E18, UpToDate 399. Baby with ventose delivery, hematoma in skull, what is the diagnosis: a. Caput succedaneum There were no other A,s provided Explanation: injuries that can occur from vacuum assisted delivery are caput succedaneum, Cephalohematoma, and Subgaleal hemorrhage. • Caput succedaneum is an edematous swelling of the scalp above the periosteum, which is occasionally hemorrhagic. It presents at birth after prolonged engagement of the fetal head in the birth canal or after vacuum extraction. Unlike cephalohematoma, it extends across the suture lines. Caput succedaneum is generally a benign condition, and it usually resolves within a few days and requires no treatment. • Cephalohematoma is a subperiosteal collection of blood caused by rupture of vessels beneath the periosteum (usually over the parietal or occipital bone), which presents as swelling that does not cross suture lines. The swelling may or may not be accompanied by discoloration, rarely expands after delivery, and does not generally cause significant blood loss. Cephalohematoma is estimated to occur in 1 to 2 percent of all deliveries and is much more common when forceps or vacuum delivery is performed. The majority of cephalohematomas will resolve spontaneously over the course of a few weeks without any intervention. However, calcification of the hematoma can occur with a subsequent bony swelling that may persist for months. Significant deformities of the skull may occur when calcification or ossification of the cephalohematoma occurs. Other complications of cephalohematoma include infection and sepsis, with Escherichia coli being the most commonly reported causative agent. Infected cephalohematomas present as erythematous, fluctuant masses that may have expanded from their baseline size. Needle aspiration and culture of the hematoma are considered to be mandatory to diagnose infection for suspected cases. Osteomyelitis is a reported complication of an infected cephalohematoma. In these affected infants, treatment includes incision and drainage of the abscess with debridement of the necrotic skull and a prolonged course of parenteral antibiotics (eg, vancomycin, gentamicin, and third-generation cephalosporin [cefotaxime or ceftazidime]). • Subgaleal hemorrhage (SGH) develops when blood accumulates in the loose areolar tissue in the space between the periosteum of the skull and the aponeurosis. The injury occurs when the emissary veins between the scalp and dural sinuses are sheared or severed as a result of traction on the scalp during delivery. SGH has been estimated to occur in 4 of 10,000 spontaneous vaginal deliveries and 59 of 10,000 vacuum-assisted deliveries. The potential for massive blood loss (20 to 40 percent of a neonate's blood volume resulting in a loss of 50 to 100 ml) into the subgaleal space contributes to the high mortality rate associated with this lesion. The subgaleal space extends from the orbital ridges anteriorly to the nape of the neck posteriorly and to the level of the ears laterally. In infants with SGH, the reported mortality is approximately 12 to 14 percent. Infants who died had massive volume loss resulting in shock and coagulopathy. SGH presents as a diffuse, fluctuant swelling of the head that may shift with movement. Expansion of the swelling due to continued bleeding may occur hours to days after delivery. Affected neonates may have tachycardia and pallor due to blood loss, although blood loss may be massive before signs of hypovolemia become apparent. Source: UpToDate 400. Pregnant screening for DM, FBS: high , OGTT: 1st hour normal, 2nd hour high, 3rd hour normal, what is your next step? a. Repeat b. Adjust diet A, is B Explanation: since this is probably the one step oral glucose tolerance test, one abnormal reading is sufficient for diagnosis, and management is by diet modification. In hacker they mentioned two abnormal or more values and not only one. Source: Toronto notes 2020, page OB27, Hacker & Moore's essentials of obstetrics & gynecology, page 204 401. Patient with negative pap smears. Last time pap smear shows low-grade intraepithelial lesion. What would you do next? a. Colposcopy b. Repeat cytology after 12 months A, is A Explanation: low-grade squamous intraepithelial lesions or LSIL further evaluation depends on two factors, age and HPV status according to The 2019 American Society of Colposcopy and Cervical Pathology (ASCCP) Risk-Based Management Consensus Guidelines for Abnormal Cervical Cancer Screening Tests and Cancer Precursors. • Ages 30 years and older: o HPV negative: The preferred approach is repeat HPV/cervical cytology co-testing in one year. If co-testing results are both normal cytology and HPV negative, co- testing should be repeated in three years. If co-testing results are either positive cytology (atypical squamous cells of undetermined significance [ASC-US] or a more severe abnormality) or HPV positive, colposcopy should be performed. Alternatively, colposcopy may be performed directly after a finding of HPV- negative LSIL. o HPV positive: Colposcopy should be performed. o HPV unknown: Colposcopy should be performed. This applies to women who were initially screened with cytology alone, typically due to the lack of availability of HPV testing. • Ages 25 to 29 years: Colposcopy should be performed. This is the preferred management for all women in this age group, even those tested for HPV who have a negative HPV result Despite HPV triage, most women with LSIL will require colposcopy. This is because the majority of women with LSIL are HPV positive, regardless of age. Source: uptodate, Toronto notes 2020, page GY44 402. Multiple questions about vaginal discharge and infections: classic presentations of (bacterial vaginosis, candidiasis, trichomonas and treatments) 403. Years old woman who has abnormal uterine bleeding, last peroid 12 years ago, on examination pale & dry vagina and loss of rugae, uterus normal size with no masses. Ultrasound show endometrial thickness of 15 mm otherwise nothing to report. What is the next step: a. Pelvic mri b. Endometrial sampling c. Exploratory laparotomy A, is B Explanation: In women with postmenopausal bleeding and not on hormonal replacement therapy, an endometrial thickness of less than or equal to 4 or 5 mm is associated with a low risk of endometrial disease. Cancer becomes increasingly more frequent relative to benign disease as the endometrial thickness approaches 20 mm. Any woman who presents with postmenopausal bleeding should undergo transvaginal ultrasonography. If the endometrial thickness is greater than 4 mm, endometrial evaluation is necessary. An outpatient endometrial biopsy is usually feasible with a sampling device such as a Pipelle, gynosampler, or Vabra aspirator. Source: uptodate, Hacker & Moore's essentials of obstetrics & gynecology, page 459 404. Female patient with itching and vaginal discharge and strawberry cervix, her husband had urethral discharge, treatment? a. Metronidazole There are no A,s Explanation: the clinical picture is highly suggestive of Trichomonas infection and treatment for that is metronidazole Source: First Aid USMLE step 2 CK 10th edition, page 371 405. Case of pregnant lady 8 weeks gestation, her crown rump length is 7 weeks, she presented with vaginal bleeding and brown discharge, how will u manage: a. Evacuation b. Observe c. Misoprostol A, is B Explanation: this is most likely a threatened abortion and management of that is rest and follow up US to assess fetal viability, other choices are for other scenarios, women with an incomplete, inevitable, or missed abortion can be managed with surgical uterine aspiration or medication uterine evacuation or with expectant management. All three approaches have similar efficacy, and the choice of treatment method depends mainly upon patient preference. Source: First Aid USMLE step 2 CK 10th edition, page 325, UpToDate 406. Lady gestational week 8 aborted, Hx of two abortion in second trimester due to cervical incompetence, cause for this abortion? a. Chromosomal abnormalities b. Cervical incompetence A, is A Explanation: The classic obstetric history of women with structural cervical weakness leading to recurrent cervical insufficiency is characterized by recurrent second-trimester pregnancy losses/deliveries that were associated with no or minimal mild symptoms, and since this current pregnancy loss is not in the second trimester this makes cervical incompetence not the correct A,. Furthermore, chromosomal abnormalities are the most cause of spontaneous abortions in the first trimester Source: UpToDate, First Aid USMLE step 2 CK 10th edition, page 324 407. Lady feel vaginal fullness increase when standing long period when urinate need to push with her hand diagnosis: a. Cystocele b. Enterocele c. Rectocele A, is A Short story: vaginal heaviness and discomfort are nonspecific for one type, however, Manuel manipulation to empty the bladder and urinary incontenance indicates the presence of a cystocele. Manual manipulation for complete bowl elimination indicates rectocele Long story: In Advanced cystocele Women with cystocele will describe symptoms of vaginal fullness, heaviness, pressure, and/or discomfort that often progress over the course of the day and are most noticeable after prolonged standing or straining. The prolapse may "kink" the urethra, thereby resulting in symptoms of obstructed voiding, such as a slow urine stream, the need to change position or applying manual pressure to empty the bladder completely, a sensation of incomplete emptying and, in rare cases, complete urinary retention. Other symptoms include stress urinary incontinence (SUI), urinary urgency, and frequency. Source: UpToDate, Hacker & Moore's essentials of obstetrics & gynecology, page 292-293 408. Disorder associated with period: a. Mood swings b. Depression c. Behavioral A, is not clear Explanation: The most common affective or behavioral symptom of PMS is mood swings. Other frequent nonphysical behavioral symptoms include irritability, anxiety/tension, sad or depressed mood, increased appetite/food cravings, sensitivity to rejection, and diminished interest in activities. The most common physical manifestations of PMS are abdominal bloating and an extreme sense of fatigue. Other common symptoms include breast tenderness, headaches, hot flashes, and dizziness. Hot flashes in women who are neither postpartum nor peri-or postmenopausal are highly suggestive of PMS or PMDD. These flushes also occur cyclically premenstrually and have been shown to be physiologically similar to menopausal hot flashes. Source: UpToDate 409. Breast feeding mother with swelling hotness and mass diagnosis: a. Fibroadenoma b. Ectasia c. Abscess A, is C Explanation: this is very clear picture of a woman with mastitis with breast abscess. Source: Toronto notes 2020, page OB47 410. Pt with spontaneous ruptured membrane want to do C/S, what to do? a. Give Abx There were not any other A,s provided Explanation: the question is incomplete and we are not sure of many points in the question, if the question is asking about C-section in itself, then For all women undergoing cesarean delivery, administer a single dose of cefazolin in the 60 minutes before making the skin incision and add a single dose of azithromycin 500 mg intravenously for those in labor or with ruptured membranes. If they are asking about general management of rupture of membrane then it depends on what gestation age the rupture occurs, if it is before 37 then prophylactic antibiotics are given, if after 37, intrapartum group B streptococcus (GBS) antibiotic prophylaxis is given, but uptodate mention that near term or at term prophylactic antibiotics is not associated with benefits for either the fetus or the mother, and it might pose some issues to the fetus. Source: uptodate, Toronto notes 2020, page OB18 411. Patient who is 25 years old came due to difficulty getting pregnant, her period comes every 23 days, lasts for 4 days what test will you measure? a. Urine FSH/LH b. Mid 21 progesterone A, is B Explanation: Women who have regular menses approximately every 28 days with molimina symptoms prior to menses (breast tenderness, bloating, fatigue, etc.) Are most likely ovulatory. In women who do not describe their cycles as such, laboratory assessment of ovulation should be performed. Ovulation is most easily documented by a mid-luteal phase serum progesterone level which assesses the level of luteal function. This should be obtained approximately one week before the expected menses. For a typical 28-day cycle, the test would be obtained on day 21. An alternative is to have the patient use an over-the-counter urinary ovulation prediction kit. These kits detect luteinizing hormone (LH), which assesses the duration of luteal function. In spite of ovulation, an inadequate luteal phase may be responsible for infertility, so yes this woman seems ovulatory but that still might be her issue. Source: uptodate, Hacker & Moore's essentials of obstetrics & gynecology, page 399-400 412. Cause of fetal bradycardia? a. Cord prolapse b. Maternal hyperthermia A, is A Source: Toronto notes 2020, page OB34 413. A concerned mother comes with her 13 years old girl. She's upset because of her height. Her height is 152 Cm (the writer of the question forgot the weight). No signs of breast development or pubic hair in her body what is the cause of her short stature? a. Constitutional b. Chromosomal c. Hormonal A, is not clear Explanation: this patient presents with presumably short stature and delayed puberty

elderly female patient had asymptotic redness at the urethra al orifice and it bleeds when you touch it? a. Urethral prolapse b. Urethral furuncle c. Urethral caruncles

is C Explanation: we will explain all these conditions below. Read about vulvar lesions in postmenopausal women for many more other conditions. • Urethral caruncles: o They are single, asymptomatic, bright-red papules at the urethral meatus in older women. They may be pedunculated or dome shaped. The surface bleeds easily. The differential diagnosis is urethral prolapse • Urethral prolapse: o It is a red papule at the urethral meatus that is seen in premenarchal or postmenopausal women. Dysuria may be present. The lesion is differentiated from the urethral caruncle by its presentation as a circumferential, bright-red nodule around the meatus, making a donut shape around the urethral orifice • Urethral furuncle or furunculosis: o Deep, walled-off collection of pus typically 2 to 4 cm in diameter and usually associated with a follicle. When not associated with a follicle, the lesion is called an abscess. Lesions begin as a tender, red, firm papule that rapidly enlarges and becomes a deep-seated nodule that lasts for days. The nodule may be capped by a white or yellow-white pustule. It may gradually expand under the skin or point at the surface and rupture purulent material. Rarely, aggregates of infected furuncles form large, deep red masses. They occur in areas of skin trauma, especially where there is moisture from hyperhidrosis along the edges of the mons pubis and perianally. Furuncles and abscesses may be self-limited or recurrent. o These are painful lesions. When large and deep, they may also be associated with systemic symptoms of malaise, fever, and chills. Source: UpToDate 462. Pregnant taking Paroxetine what to do? a. Continue b. Stop it because risk of preterm c. Stop it because it affects fetus A, is C Explanation: paroxetine may be associated with congenital heart defects, results across different observational studies are inconsistent. The manufacturer suggests discontinuing paroxetine or switching to another antidepressant unless the benefits of therapy justify continuing treatment during pregnancy

examples include placenta previa and previous classical cesarean delivery. Based on expert opinion, ECV is generally also contraindicated in the following settings because they are associated with a low likelihood of successful version and/or increased risk of fetal harm from the procedure: • Severe oligohydramnios. • Nonreassuring fetal monitoring test results. • Hyperextended fetal head. • Significant fetal or uterine anomaly (eg, hydrocephaly, septate uterus). • Placental abruption. • Ruptured membranes. • Active labor with fetal descent. Multiple gestation. However, internal or external version is widely considered an acceptable approach for the second twin after delivery of the first twin. Source: UpToDate 484. 6 weeks pregnant came with vaginal bleeding, had 2 previous miscarriages in 2nd trimester, in the last one D&C done, what is the cause of this miscarriage? a. Chromosomal anomalies b. Aschermann's syndrome

is not clear Explanation: both of these can cause miscarriages, but since this one is little bit earlier it seems like it is due to another etiology not similar to the first two which were probably due to chromosomal anomalies, so after we discussed this we thought the A, was probably Aschermann's syndrome. In the question, during the exam look for other hints for Aschermann's syndrome as that might guide you into choosing this A, choice. 485. Most common valvular cancer: a. Squamous b. Basal cell c. Melanoma d. Adenocarcinoma A, is A Explanation: Source: Toronto notes 2020, page GY47 486. F with Hx of PUD Tx with Abx Now want to get pregnant her hysterosalpingogram shows normal uterus but no contrast in fallopian tube, Her husband semen normal, She has low LH and FSH how to treat? a. Induce ovulation then intercourse b. Induce ovulation with semen injection in uterus c. IVF A, is C Explanation: For patients with access to in vitro fertilization (IVF) services, IVF is first-line treatment for tubal factor infertility due to bilateral tubal obstruction. Source: UpToDate 487. Vulvar lesion at 5 clock tender red swelling, what is the diagnosis? a. Bartholin abscess No other A,s were provided Explanation: these are usually explained as 4 or 8 o'clock cysts and in some places 5 or 7 o'clock cysts. Source: UpToDate, American Academy of Family Physician, Toronto notes 2020, page GY30 488. Pap smear when to begin? No A,s were provided Explanation: the recommendations are different in different guidelines, we chose UpToDate and it states that the starting age is 21 in US. Generally speaking, in many countries, it starts between 21 and 25 Source: UpToDate 489. Young girl with hair at axilla and body, adult odor + description of Turner syndrome (short and obese) what is the hormone you will look for? a. Estrogen b. FSH A, is not clear Explanation: the other A,s to this question were other hormones the writer forgot them. The question is incomplete, there is no clear reason to which hormone we would choose in this case, are we looking for FSH to check the ovary reserve? Or checking LH or progesterone for ovulation problems? Read about turner syndrome and depending on the question the A, should be easy. 490. Female with mass at the vulva at 7 o'clock not responding to warm baths, Dx? a. Bartholin cyst b. Skene cyst A, is A Explanation: these are usually explained as 4 or 8 o'clock cysts and in some places 5 or 7 o'clock cysts. Source: UpToDate, American Academy of Family Physician, Toronto notes 2020, page GY30 491. Ab contraindication in pregnancy? a. Ceftriaxone b. Ciprofloxacin c. Penicillin A, is B Explanation: fluoroquinolones are generally not used during pregnancy. Source: UpToDate 492. Pregnant with polyhydramnios and came with abdominal pain and rupture of membrane and fetus become bradycardia? a. Cord prolapse No other A,s were mentioned. Explanation: Cord prolapse usually presents with the abrupt onset of severe, prolonged fetal bradycardia or moderate to severe variable decelerations in a patient with a previously normal tracing. The fetal heart rate changes typically occur soon after membrane rupture or an obstetric intervention that dislodges the presenting part. Note that there was polyhydramnios which is one of the risk factors for this condition. Source: UpToDate 493. Pregnant tells you that she never complained of chickenpox, what to do? a. Varicella vaccine b. Avoid exposure A, is B Explanation: Varicella vaccine is a live vaccine and should not be administered to pregnant women. Pregnant women without evidence of immunity to varicella should receive the varicella vaccine postpartum, ideally prior to discharge after delivery. Note that routine varicella immunization of the children of susceptible pregnant women is not contraindicated. The risk of transmission of vaccine virus from a healthy individual to a susceptible household contact is low. Source: UpToDate 494. Anembryonic pregnancy management. a. Expectant management No other A,s were provided Explanation: patients with blighted ovum can be managed with surgical uterine aspiration or medication uterine evacuation or with expectant management. All three approaches have similar efficacy, and the choice of treatment method depends mainly upon patient preference. Source: UpToDate 495. Ground glass uterus on US, her symptoms dyspareunia, dysmenorrhea what do you expect? a. Infertility b. Malignancy A, is A Explanation: the writer of the question probably made the mistake of writing that this is a uterus US, probably it was a cyst in the ovary (endometrioma), that usually comes with endometriosis, and this explains the symptoms and the ground glass appearance on US. Endometriosis is not associated with malignancy, but it is associated with infertility. Source: UpToDate, Toronto notes 2020, page GY11-12 496. Sexual abuse where is the tear? a. 6 o'clock No other A,s were provided Explanation: postcoital fissures usually occur in posterior fourchette which is the 6 o'clock location. Furthermore, posterior fourchette is one of the most common areas of injures in victims of forced vaginal penetration Source: Forensic Emergency Medicine by Jonathan S. Olshaker MD, Second edition, page 98-99, UpToDate 497. Patient with post coital bleeding and pic of cervical polyp, what is your management? a. Admit for investigation b. Office polypectomy A, is B Explanation: Polyps should be removed if they are symptomatic (eg, bleeding, excessive discharge), large (≥3 cm), or appear atypical. Malignancy is rarely found in a cervical polyp

thus, these findings warrant diagnostic evaluation for the disease. Peripheral edema in preeclampsia may be due to capillary leaking or represent "overfill" edema. https://www.uptodate.com/contents/preeclampsia-clinical-features-and- diagnosis?search=edema%20in%20pregnancy&source=search_result&selectedTitle=1~150& usage_type=default&display_rank=1# 360. . 30yrs Female Pt e PCO obese comes to you she doesn't want to get pregnant what is action? A. give metformin B. progesterone only bills C. combined estrogen & progesterone The

is: C Explanation: We suggest combined estrogen-progestin oral contraceptives (COCs) as first-line therapy for menstrual dysfunction and endometrial protection. COCs provide a number of benefits in women with PCOS, including: ● Daily exposure to progestin, which antagonizes the endometrial proliferative effect of estrogen ● Contraception in those not pursuing pregnancy, as women with oligomenorrhea ovulate intermittently and unwanted pregnancy may occur ● Cutaneous benefits for hyperandrogenic manifestations Source: https://www.uptodate.com/contents/treatment-of-polycystic-ovary-syndrome-in- adults?search=Pcos&source=search_result&selectedTitle=2~150&usage_type=default& display_rank=2#H771175086 361. Years old female with pubic hair, no clitoromegaly, obese, and her height is above the 90th percentile. What to give this patient? A- Testosterone B- 5a-dihydrotestosterone sulfate C- 17 hydroxylase D- LH or FSH Suggested A,: B 362. Antiphospholipid syndrome, perhaps the question was asking about what would you see in this syndrome? a. Recurrent miscarriage A, is A Explanation: Recurrent spontaneous abortions are one of the well-known complications of Antiphospholipid syndrome, and it is part of the clinical criteria used for diagnosis. Source: Toronto notes 2020, Page RH13 363. Case of HELLP syndrome asking the diagnosis? A,s were not provided Explanation: HELLP syndrome is characterized by: a. H: hemolysis b. EL: elevated liver enzymes c. LP: low platlets Source: Kaplan lecture notes: Obstetrics and gynecology 2019 364. Late deceleration, perhaps asking for which of the following is the cause? A. Placental insufficiency B. Head compression C. Umbilical cord compression A, is A Explanation: Source: First Aid USMLE step 2 CK 10th edition, Page 329. 365. Incomplete question: Normal CTG? b. Reassuring A,s were not provided Explanation: Source: Toronto notes 2020, Page OB35. 366. The next step in case of placenta previa? A,s were not provided Explanation: Source: Toronto notes 2020, page OB14 & First Aid USMLE step 2 CK 10th edition, Page 338. 367. Cases ectopic pregnancies asking about treatment. No A,s were provided Explanation: if the patient is stable, the management can be medical or surgical, medical is preferable now due to low cost and similar outcomes. If there are any contraindications to medical treatment then you go for surgical treatment. Source: Toronto notes 2020, Page GY21 & Hacker & Moore's essentials of obstetrics & gynecology Page 312. 368. Diagnosis of anembryonic pregnancy: No A,s were provided Another name for this condition is blighted ovum. 369. Side effect of evacuation of a woman: A. Amniotic emboli B. Uterine perforation A, is B Explanation: if the question is asking about most concerning or most common immediate complication, this is the A,, if it is asking about the most common, then it could be one of the items mentioned in this box: However, in uptodate, hemorrhage was said to be extremely rare. So choose your A, wisely. Source: uptodate, Hacker & Moore's essentials of obstetrics & gynecology Page 360. 370. A picture of a white uterus with black dots? No A,s were provided A, could be uteroplacental apoplexy, also known as Couvelaire uterus. 371. Fetal bradycardia on CTG with everything else being normal? c. Anemia d. Head compression e. Uteroplacental insufficiency A, is unclear. Explanation: we are not sure that the reader of the CTG is aware of what he is seeing, which makes this hard to A,. Looking at the A,s, anemia was the A, suggested by the writer of the question as well as the editors, however in anemia you are expected to see tachycardia and not bradycardia. They suggested A as the A, in case the fetus had sinusoidal pattern, which the writer incorrectly identified as bradycardia. Sinusoidal pattern can be seen in severe fetal anemia. Head compression can cause fetal bradycardia by vagal stimulation

however, polyps that are removed should be submitted to the laboratory for histological study. Source: UpToDate, Hacker & Moore's essentials of obstetrics & gynecology, page 254 477. A question about classic features of turner. Explanation: Source: First Aid USMLE step 2 CK 10th edition, page 395 478. Infertile woman with regular cycle how to assess? a. Day 21 progesterone No other

s were provided Explanation: Women who have regular menses approximately every 28 days with molimina symptoms prior to menses (breast tenderness, bloating, fatigue, etc.) Are most likely ovulatory. In women who do not describe their cycles as such, laboratory assessment of ovulation should be performed. Ovulation is most easily documented by a mid-luteal phase serum progesterone level which assesses the level of luteal function. This should be obtained approximately one week before the expected menses. For a typical 28-day cycle, the test would be obtained on day 21. An alternative is to have the patient use an over-the-counter urinary ovulation prediction kit. These kits detect luteinizing hormone (LH), which assesses the duration of luteal function. In spite of ovulation, an inadequate luteal phase may be responsible for infertility, so yes this woman seems ovulatory but that still might be her issue. Read about infertility work up, as there might be other options which are better, since this woman ovulation is probably normal, there might be other investigations that are better done. Source: UpToDate, Hacker & Moore's essentials of obstetrics & gynecology, page 399-400 479. Pregnant 38 weeks UTI, what to give her? a. Cephalexin b. Nitrofurantoin A, is A Explanation: nitrofurantoin use is limited in the first trimester, and its use is contraindicated in pregnant patients at term (38 to 42 weeks' gestation), during labor and delivery, or when the onset of labor is imminent. Therefore, if any of the two situations occur you try to avoid it. Source: UpToDate 480. Scenario of female with DVT which of the following you should ask about a. Ask about OCP No other A,s were provided Explanation: Source: Toronto notes 2020, page H35 481. Case of ovarian cancer treatment? a. Chemotherapy b. Surgery followed by a chemotherapy c. Surgery followed by a radiotherapy The A, is not clear Explanation: the question is incomplete, the treatment largely depends on the type of the tumor and the stage at which this tumor is at, the writer of the question chose B, which is the correct A, for a lot of ovarian carcinoma types, just read about the topic further to be able to A, similar questions. 482. Pregnant with previous cs now came with breech and want ECV what is the absolute contraindication: a. Previous cs b. Variable deceleration c. Anterior placenta A, is B Explanation: Any absolute contraindication to labor and vaginal birth regardless of fetal presentation is an absolute contraindication to ECV

the sensitivity and specificity of sputum culture are about 80 and 98 percent, respectively. Source: UpToDate 443. High prolactin next step in the investigation? a. Brain MRI No other

s were provided Explanation: the question is incomplete Source: Toronto notes 2020, page E18 444. Couple present to infertility clinic everything normal after taking history and doing examination which first investigation to start a. Semen analysis b. FSH and LH A, is A Explanation: usually both partners should be tested in any case of infertility, and the work up is extensive to look for a cause, the evaluation of the male occurs early so that questions about coital frequency can be addressed and azoospermia or severe oligospermia or asthenospermia (low motility) can be identified. Also, in First Aid they mentioned first history and physical exam of the woman, then semen analysis before further work-up in the woman, as it is simple and noninvasive. Source: Hacker & Moore's essentials of obstetrics & gynecology, page 397, First Aid USMLE step 2 CK 10th edition, page 368. 445. Hepatitis B pregnant, recommendation about lactation. a. Continue b. Discontinue A, is A Explanation: Maternal hepatitis due to hepatitis B virus has been shown to be compatible with breastfeeding after proper immunoprophylaxis. Transmission of HCV by breastfeeding theoretically is possible but has not been documented Source: UpToDate 446. 38y pregnant develop HTN 160, no other symptoms previously well before this visit, diagnoses? There were no A,s provided. Explanation: this is probably gestational HTN, read about hypertensive diseases in pregnancy. 447. Pregnant woman with right abdominal pain, doctor rolled out obstetric condition, diagnosis? a. Cholecystitis b. Appendicitis A, is B Explanation: appendicitis is the most common cause of acute surgical abdomen in pregnancy, if this is a case of acute abdomen then this is the A,. Keep in mind that cholecystitis can seen in pregnant ladies, and make sure to find other clues and hints in the question before ruling this out. Source: UpToDate 448. After doing salpengotomy for ectopic, what is next? a. Every week bhcg b. No need for follow up A, is A Explanation: in the question, the spilling of the word salpengotomy is wrong, the A, that the writer chose was A. We kept it to show you that there could be two procedures and with every procedure, there is different follow up: • Salpingostomy: o For women who undergo salpingostomy, serum human chorionic gonadotropin (hcg) is measured weekly until the level is undetectable. • Salpingectomy: o For women who undergo salpingectomy, if the pathology evaluation confirms a tubal gestation, many surgeons do not check a postoperative hcg, and others check a single postoperative hcg to confirm a large decline in the level. Source: UpToDate 449. Pregnant (diagnosed 3 months back), with bacterial vaginosis, what's the most appropriate treatment? a. Metronidazole There was no other provided A,s Explanation: The therapeutic options include: • Metronidazole 500 mg orally twice daily for seven days • Metronidazole 250 mg orally three times daily for seven days • Clindamycin 300 mg orally twice daily for seven days Some clinicians avoid use of metronidazole in the first trimester because it crosses the placenta, and thus has a potential for teratogenicity. However, meta-analysis has not found any relationship between metronidazole exposure during the first trimester of pregnancy and birth defects, and the CDC no longer discourage the use of metronidazole in the first trimester. An additional concern is that the drug is mutagenic in bacteria and carcinogenic in mice, but there is no evidence of harm in humans. Source: UpToDate 450. Pregnant lady, 41 GA in labor on epidural analgesia, mg sulfate for pre- eclampsia and oxytocin, CTG showed prolonged (variable, in another question) deceleration and the mother was hypotensive, most likely cause of the CTG finding? a. Mg sulfate b. Oxytocin c. Epidural analgesia A, is B Explanation: although all of the A,s can cause hypotension, we chose oxytocin because it can cause uterine hyperstimulation, which can lead to variable decelerations, and it can also lead to uterine rupture which will lead to hypotension in this pregnant lady. If there are normal contractions, or CTG was normal, then think of epidural, if there were signs of magnesium sulfate toxicity (loss of deep tendon reflexes, respiratory paralysis...etc) then that could be the A,. Source: UpToDate, https://www.ncbi.nlm.nih.gov/books/NBK546627/ 451. Pregnant lady in latent phase of labor for 4 hours with no progression, what to do next? a. Artificial rupture of membrane b. Oxytocin c. Immediate c/s A, is not clear Explanation: 4 hours is not long enough to call this abnormal progression of labor, the question is also missing other information regarding whether you should choose cervical ripening (artificial rupture of membrane), or induction (oxytocin), as choosing which strategy to use is dependent on the bishop score. Read about this topic so when you have similar questions you can A, it. Source: UpToDate, Hacker & Moore's essentials of obstetrics & gynecology, page 104-105 452. A patient with bishop score 3 (70% effacement, 1cm dilation, station is -3) 32 weeks gestation, had abdominal pain but stable, what to do? a. Dexamethasone b. Tocolytic c. Furosemide A, is B Explanation: tocolysis is given before dexamethasone, to allow sometime for steroids to work, this is the only reason that makes us choose this A,, your A, might change depending on different questions. Source: UpToDate 453. Old women vaginal itching slightly improved after antibiotics and topical medication Dx? No A,s were provided Explanation: question is incomplete 454. Symmetrical/uniform uterine enlargement + severe + dysmenorrhea + menorrhagia. Dx? a. Adenomyosis No other A,s were provided Explanation: know the differences between endometriosis and adenomyosis presentation Source: Toronto notes 2020, page GY13 455. Dm during labor with CTG finding (writer forgot the finding) management? a. Stop oxytocin b. Change positions A, is not clear Explanation: the question is incomplete, and we are not sure the A,s are actual A,s of the question 456. Most common site for genital warts: a. Uterine body b. Uterine fundus c. Vagina d. Vulva A, is D Explanation: the types of HPV that causes genital warts usually affect the anogenital area, keep in mind that vagina and cervix could also be involved. Source: UpToDate 457. Most high yield diagnostic examination in gynecology routine clinics: a. Abdomen b. General appearance c. Pelvic digital examination d. Vaginal examination A, is B Explanation: this is a dumb question, the A, mentioned by the writer was general appearance. We could not find an evidence to support this A, choice. This A, however is the most logical A,. 458. On Pap smear, SCC is spotted on cervix, what to do next? a. Hysterectomy b. Cone biopsy c. Colposcopy A, is C Explanation: Source: Toronto notes 2020, page GY44 459. Atypical hyperplastic endometrium, how to manage? a. Hysterectomy b. Oral progesterone A, is A Explanation: the management of endometrial hyperplasia is dependent on the histology and we can categorize it into three parts: • Women with simple endometrial hyperplasia without atypia: o They may be managed with progestin-only therapy, oral contraceptives, or expectant management. Hormonal therapies result in resolution of endometrial hyperplasia and a return to a normal uterine bleeding pattern in most women. Expectant management is also reasonable for some women, but may not lead to resolution of endometrial hyperplasia in patients with continued exposure to excess estrogen or with other risk factors for endometrial cancer. • Complex endometrial hyperplasia without atypia: o They may be managed with progestins or expectant management. For women with complex hyperplasia without atypia that progresses to endometrial hyperplasia with atypia on progestin therapy, hysterectomy is the preferred treatment. Also, hysterectomy is reasonable in postmenopausal women who decline or have contraindications to progestins. • Endometrial hyperplasia with atypia: o Postmenopausal women and women who do not desire fertility: For this category of patients the recommendation is to do hysterectomy, and not progestin therapy. o Women who wish to preserve fertility, decline hysterectomy, or are at high surgical risk: These patients can be offered progestin therapy and follow-up endometrial sampling. Source: UpToDate 460. Ultrasound showed ground glass appearance what do you expect: a. Endometrioma There were no other A,s provided Explanation: the question is incomplete, but probably the question about a mass or cyst that was seen on US of the ovary. Endometrioma of the ovaries can be seen on US as a homogenous echo pattern of the cyst content, this can be described as a ground glass appearance, and it is very characteristic. In case you were wondering what is this ground glass appearance thing that keeps popping up in all sort of images. It refers to that homogenously opaque part of this glass. Source: UpToDate 461. Picture of female urethra

Fetus's both legs and arms are flexed what type of breech presentation is this? A. Complete B. Incomplete

: A. 48

24 years old married, never had pap test? A. Do it now

: A. 40

Young girl with DUB . ( bleed every 3 week ) What to give? A. OCP

: A. Toronto Notes 2020, P505 27

PROM then fetal developed Bradycardia? A. Cord prolapse

: A. If artificial rupture of membrane, painless vaginal bleeding , fetal bradycardia >> VASA PREVIA Toronto Notes 2020. 44

otherwise, cesarean delivery is the better option. We agree with an American College of Obstetricians and Gynecologists (ACOG) practice bulletin that considers the following scenarios potentially appropriate reasons for operative vaginal delivery. ● Maternal exhaustion and an inability to push effectively. ● Maternal medical indications, such as maternal cardiac disease and a need to avoid pushing in the second stage of labor. ● Prolonged second stage of labor. ● Suspicion of immediate or potential fetal compromise. However, no indication is absolute, and cesarean delivery is also an option in these clinical settings. Note: The fetal head is not engaged thus C-section is more reasonable

. Source. https://www.uptodate.com/contents/cesarean-delivery-preoperative-planning-and- patient- preparation?search=cesarean%20section&source=search_result&selectedTitle=2~150&usage _type=default&display_rank=2#H4 314. Vaccine contraindicated in pregnancy A. zoster B. hep B C. Meningococcal The A, is: A explanation: Two zoster vaccines are available for prevention of herpes zoster and postherpetic neuralgia. One is a live vaccine (Zostavax) that should not be administered to pregnant women

What is the type of infertility in PCOS: A. Ovulation B. Hormonal

: A. Hacker & Moore's Ess. of OB&GYN.Ch 33 34

tanner 5 but later than her friends, Dx: A. hypogonadotropic hypogonadism B. gonadal agenesis C. transverse vaginal septum The

is: A Explanation: functional forms of HH, characterized by a transient defect in GnRH secretion, are relatively common in women, in response to significant weight loss, exercise, or stress leading to hypothalamic amenorrhea. Source: https://academic.oup.com/jcem/article/98/5/1781/2536708 280.3 3 typical cases of Bacterial Vaginitis (very easy) ask about Dx and Mx and what to see A. under microscope: The A, and explanation are: Office-based testing is required to diagnose BV. microscopy reveals clue cell which is an epithelial cell with "serrated" edges caused by bacteria. The addition of potassium hydroxide to the vaginal secretions (the "whiff" test) releases a fishy, amine-like odor. Clinicians who are unable to perform microscopy can use alternative diagnostic tests such as a pH and amines test card, detection of G. vaginalis ribosomal RNA, or Gram stain. Culture of G. vaginalis is not recommended as a diagnostic tool because of its lack of specificity. Management: Metronidazole 500 mg orally twice a day for 7 days* OR Metronidazole gel 0.75%, one full applicator (5 g) intravaginally, once a day for 5 days OR Clindamycin cream 2%, one full applicator (5 g) intravaginally at bedtime for 7 days Source: Hacker Moore Page 277, sixth edition. 281. Typical cases of trichomonas, one about Dx and Mx The A, and explanation are: Microscopy of the secretions may reveal motile trichomonads (Figure 22-2) and increased numbers of leukocytes, but the sensitivity of this test is poor (50%). For this reason, nucleic acid amplification testing is recommended when trichomoniasis is suspected but not confirmed by microscopy. Metronidazole is the drug of choice for treatment of vaginal trichomoniasis. Both a single- dose (2 gorally) and a multidose (500 mg twice daily for 7 days) regimen are highly effective and have cure rates of about 95%. Source: Hacker Moore Page 278, sixth edition. 282. Pregnant in 14 wk , BP reading is 180/110. What is the diagnosis? A. preeclampsia B. pregnancy induced HTN C. essential HTN The A, is: C Explanation: The diagnosis of chronic hypertension requires at least one of the following: known hypertension before pregnancy or the development of hypertension before 20 weeks' gestation. Most pregnant women with chronic hypertension will have essential hypertension. Source: Hacker Moore Page 185, sixth edition. 283. Pregnant in 20 something week BP of 150/90 ,(picture shows urine dipstick +2 protein) no other Sx, what is the Mx: The A, and explanation are: The safest, most efficacious drugs for the acute control of severe hypertension complicating preeclampsia are labetalol and hydralazine. Source: Hacker Moore Page 190, sixth edition. 284. long case and pic of ctg shows fetal bradycardia, what is the cause: A. epidural anesthesia B. MgSo4 C. oxytocin The A, is: A Explanation :Fetal bradycardia — Initiation of neuraxial analgesia may be followed by fetal heart rate abnormalities (eg, bradycardia, reduced variability, late decelerations) as a result of hypotension, uterine hyperactivity, or increased uterine tone. Source:https://www.uptodate.com/contents/adverse-effects-of-neuraxial-analgesia-and- anesthesia-for- obstetrics?search=epidural%20anesthesia%20%20effects%20on%20the%20fetus&source=se arch_result&selectedTitle=3~150&usage_type=default&display_rank=3 285. Case of turner syndrome asks about the most diagnostic test: A. chromosomal study The A, is: A Explanation: Genetic testing should be performed to confirm the diagnosis of Turner syndrome in any patient with the characteristic clinical features. The first step is a karyotype analysis, usually using peripheral blood mononuclear cells. To detect mosaicism. Source: https://www.uptodate.com/contents/clinical-manifestations-and-diagnosis-of-turner- syndrome?search=turner%20syndrome&source=search_result&selectedTitle=1~150&usage_ type=default&display_rank=1#H623378612 286. Hypotensive unstable ectopic? A. laparoscopic B.MTX The A, is: A Explanation: MTX is contraindicated and surgery is required when the patients is Hemodynamically unstable. Source:https://www.uptodate.com/contents/ectopic-pregnancy-choosing-a- treatment?search=ectopic%20pregnancy%20surgery&source=search_result&selectedTitle=2 ~150&usage_type=default&display_rank=2 287. Pt with herpes, MS ,Hx of rheumatic fever she's in labor which of following is strong indication of ventose ? A. Herpes B. MS C. rheumatic fever D. prolonged labor The A, is :D Explanation: operative vaginal delivery is indicated in prolonged second stage of labor. In nulliparous women, this is defined as lack of continuing progress for 2 hours without regional anesthesia or 3 hours with regional anesthesia. In multiparous women, it is defined as lack of continuing progress for 1 hour without regional. Note: I case of herpes offer cesarean delivery as soon as possible after the onset of labor/rupture of membranes to women with a history of genital HSV and either of the following: ● Active genital lesions (including those that have crusted) ● Prodromal symptoms (eg, pain, burning). Source: Hacker Moore Page 229, sixth edition. https://www.uptodate.com/contents/genital-herpes-simplex-virus-infection-and- pregnancy?search=herpes%20in%20pregnancy&source=search_result&selectedTitle=1~131 &usage_type=default&display_rank=1 288. Missed abortion what to Do

no bleeding, cervix length is 1.5 cm and fetal membranes prolapse through it Dx? A. Threatened abortion B. cervical incompetence Note: cervical length < 25 mm indicates cervical incompetence The

is: B Explanation: Early in the course of cervical insufficiency, the cervix may be soft and closed, with minimal effacement. Provocative maneuvers such as suprapubic or fundal pressure or the Valsalva maneuver may reveal fetal membranes in the endocervical canal or vagina

free fluid may also be seen with torsion. In addition, the classic history of cyst rupture is the sudden onset of pelvic pain at midcycle, often following sexual intercourse. Source: UpToDate, Kaplan lecture notes: Obstetrics and gynecology 2019 472. Scenario of a female bleeding after intercourse what is the place of bleed? a. Cervix No other

s were provided Explanation: this question is incomplete, read about causes of postcoital bleeding. 473. 14-year female complain of menstrual pain, management? a. NSAID No other A,s were provided Explanation: this is perhaps a case of primary dysmenorrhea, which is the most common gynecological complaint among adolescent females. The treatment is by NSAID's, other options include hormonal therapies such as oral contraceptives, but these are second options in cases of young females, but may be first line in case of sexually active females if you want to prevent pregnancy and treat dysmenorrhea. Source: UpToDate, Hacker & Moore's essentials of obstetrics & gynecology, page 269 474. Long case of pregnant lady in labor, on epidural anesthesia (or spinal?) Was given oxytocin and now is 100% effacement and station something and other things something suddenly the contractions stopped, what is the next step? a. Stop anesthesia infusion b. C/S c. Give oxytocin d. Change position of the mother A, is not clear Explanation: the question is missing a lot of information, this topic is covered thoroughly in hacker and moore chapter 11 (uterine contractility and dystocia), we recommend reading about this. 475. What's the predominant estrogen in pregnancy a. Estrone b. Estradiol c. Estriol d. Cortisol A, is C Explanation: Estrone is prominent in postmenopausal woman, while estradiol is prominent in females in their childbearing years, estriol is the main estrogen in pregnancy. Useful way to remember this is the following: • A married woman with her husband are two, estra'di'ol • A married woman with her husband and the child in her womb are three, Es'TRI'ol • A lonely postmenopausal woman because her husband and her reckless kid died in a car accident when the kid was learning how to drive, or a postmenopausal woman who just outlived everyone because women tend to live longer is one (choose whichever you like), Estr'one Source: Hacker & Moore's essentials of obstetrics & gynecology 476. Woman in clinic doctor found vaginal polyps, what to do? a. Excision in clinic if large b. Pap smear c. Go home (reassurance) A, is not clear Explanation: we don't think these choices are the true choices of this question, and we don't think this was a vaginal polyps question, it was probably a cervical polyp Cervical polyps should be removed if they are symptomatic (eg, bleeding, excessive discharge), large (≥3 cm), or appear atypical. Malignancy is rarely found in a cervical polyp

this could result from either hypergonadotropic (chromosomal in our

s) or hypogonadotropic (hormonal) hypogonadism. For this reason, with no sufficient information to A, either, it is difficult to A, this. In similar questions look for features of some of the diseases that will cause either type (Turner's syndrome, kallmann syndrome, hypopituitarism...etc) Source: Hacker & Moore's essentials of obstetrics & gynecology, page 381 414. Dm mother with hypoglycemic infant, which preferred line? a. Central b. Peripheral c. Orogastric d. Nasogastric A, is not clear Explanation: similar question came in another file, and the question stem "newborn of diabetic mother. From where to take samples, and the A, was central (although this A, is wrong and it is peripheral sampling), this question now is asking what line are you going to use and I think even that A, in that file was because this question was written poorly. Hypoglycemia is treated by parenteral glucose infusion, and the route is dependent on your goals of management. The maximum dextrose concentrations for fluid administered through a peripheral IV catheter or a low lying umbilical venous catheter is 12.5 percent, and through a central venous catheter (including a centrally positioned umbilical venous catheter) is 25 percent. If no IV access has been established, dextrose at a concentration of 12.5 percent or less can be infused through an umbilical arterial catheter while exploring other possible forms of vascular access. In severe cases, rates of fluid administration required to deliver sufficient glucose to treat hypoglycemia may be greater than the rate of maintenance fluids. In these cases, the patient's fluid and clinical status should be monitored closely for volume overload, looking for evidence of pulmonary edema, heart failure, and hyponatremia. Infants who depend upon high infusion rates or a dextrose concentration greater than 12.5 percent require placement of a central venous catheter. In some cases diuretics may be indicated. Source: UpToDate 415. Patient who had 100000 bhcg, examination of cervix revealed an aggressive mass: a. Partial mole b. Choriocarcinoma A, is B Explanation: On speculum examination you might see probably metastasis in the vagina, these are generally identifiable as a vascular implant, most commonly located in the suburethral area or fornices. These lesions should not be biopsied because they are extremely vascular, and hemorrhage may occur. Source: UpToDate, Hacker & Moore's essentials of obstetrics & gynecology, page 471 416. Pregnant in her 10th week, presented with nausea ,vomiting and abdominal pain and hemoptysis, her fundal height is large for gestational age, bhcg levels are extremely high, speculum exam showed irregular aggressive mass protruding from the cervix and liable to bleeding when touched, what's your next step in management? a. Biopsy the mass b. Dilatation and evacuation c. Staging and chemo d. Hysterectomy A, is C Explanation: the workup of a patient with choriocarcinoma is the same as that for patients with hydatidiform mole (which will be mentioned at the end of this explanation), but it should also include a computed tomographic scan of the abdomen, pelvis, and head. In addition, a lumbar puncture should be performed if the computed tomographic scan of the brain is normal, because simultaneous evaluation of the β-hcg level in the cerebrospinal fluid and serum may allow detection of early cerebral metastases. Because the β-subunit does not readily cross the blood-brain barrier, a ratio of serum to cerebrospinal fluid β-hcg levels of less than 40 : 1 suggests central nervous system involvement, with secretion of the β-hcg directly into the cerebrospinal fluid. Then the treatment is based on the metastasis, but generally speaking it is by chemotherapy, and role of surgery (hysterectomy) is only implemented in selected cases, for example in chemotherapy resistant disease or with mets. Dilatation and evacuation is the A, if this was a complete mole, and biopsy of the mass isn't done. The A, is staging and chemotherapy. Work up for complete mold includes: a complete blood count done to exclude anemia, which might require a transfusion. They require an assessment of platelet count, prothrombin time, partial thromboplastin time, and fibrinogen level, because an occasional patient may experience disseminated intravascular coagulation. Liver and renal function tests should be performed. Blood should be typed and cross-matched in the event that excessive bleeding is encountered at the time of evacuation of the mole. A chest film should be obtained, as should an electrocardiogram if tachycardia is present or if the patient is older than 40 years of age. Source: uptodate, Hacker & Moore's essentials of obstetrics & gynecology, page 469-472 417. There were multiple questions on turner syndrome, but we don't have the question stems. Read about turner thyroid problems 418. Case about amenorrhea with pubic hair and breast? a. Flow obstruction There weren't any other provided A,s. Explanation: the question is incomplete and is lacking other choices, but we'll A, and explain this assuming the case was due to Mullerian anomalies. This lady is presenting with proper sexual development, but with amenorrhea, this can be either due to certain Mullerian anomalies, or secondary amenorrhea with normal Mullerian development. The list of the differentials seems very huge and daunting, but you can determine the etiology based on the accompanying symptoms and signs, we'll focus on select Mullerian anomalies but go through all the etiologies which can be found in Hacker & Moore's chapter 33. An imperforate hymen should be suspected in adolescents who report monthly dysmenorrhea in the absence of vaginal bleeding. Clinically, these patients often present with a vaginal bulge and a midline cystic mass on rectal examination. Ultrasonography confirms the presence of a normal uterus and ovaries with a hematocolpos. These patients should be treated with hymenectomy. Alternatively, females may present with similar symptoms, but without a vaginal bulge. When ultrasonography confirms a normal uterus and ovaries, a transverse, obstructing vaginal septum or cervical agenesis should be suspected. MRI is the diagnostic procedure of choice in these patients. If an MRI scan confirms a transverse septum, surgical correction is indicated. Surgical construction of a functional cervix is extremely difficult. In general, it is recommended that women with cervical agenesis undergo hysterectomy. Source: Hacker & Moore's essentials of obstetrics & gynecology, page 381-384 419. Case about simple test for turner pt with amenorrhea? a. FSH /LH b. TFT A, is not clear Explanation: we are not sure of the choices mentioned in this question, whether the FSH and LH were in the same choice or in separate choices. Usually to check for ovarian function the test that is used is either FSH, or AMH, and this is done to turner patients at around the age of 10-11 to estimate the ovarian function and predict the need for estradiol replacement. AMH is a reasonably sensitive marker of prepubertal follicular development and can be used to assess ovarian function throughout childhood. Source: UpToDate 420. Case of endometrial cancer with pt has risk factor and stop progesterone Explanation: probably the question was asking about what is the complication of stopping progesterone, or what is a cause of estrogen excess, the question is incomplete. 421. Case about pregnant with anemia does not take folic acid in first 3 month. She is at high risk of what? a. Higher risk of NT defect. No other A,s were provided 422. Pregnant lady just delivered and she's known to have bronchial asthma. Which of the following uterotonic medications you would avoid giving? a. Misoprostol b. Carboprost F2 alpha c. Oxytocin d. Methylergonovine A, is B Explanation: carboprost is a PG-F2Alpha analog and these are known to cause constriction in the airways and increase in inflammation making their use in asthmatics patients very limited. Source: UpToDate, Medscape 423. Couple came to the infertility clinic. Semen analysis was normal, cervical exam not done yet. The wife has regular menstrual cycles every 36-40 days. Investigations show monophasic basal temperature. What is the cause of infertility? a. Coital b. Vaginal c. PID d. Anovulatory A, is D Explanation: basal body temperature recording is an old but inexpensive way of detecting ovulation although it is of limited clinical utility due to the variability of recording, and inaccuracies that can occur, and it is not commonly used. However, it can be used to detect changes in temperature that occur after the LH surge. Progesterone released from the corpus luteum at the time of ovulation has potent effects on the hypothalamus, one of which is to increase body temperature. Temperature changes are sufficient to retrospectively identify ovulation, but they occur too late to be useful for timing intercourse. This patient has monophasic basal temperature which indicates no change in the body temperature which gives us a hint that this might be a ovulation issue. Source: UpToDate, Hacker & Moore's essentials of obstetrics & gynecology, page 399 424. Female patient wants to preserve her fertility, has a 6 cm fibroid. How would you remove it? a. Hysteroscopy resection b. Laparoscopic myomectomy c. Laparotomy myomectomy d. Hysterectomy A, is B Explanation: we have a patient with fibroid that is 6 cm in size, but we don't know the location of it. Hysteroscopic myomectomy is the procedure of choice when only small (<5 cm) intracavitary myomas or submucosal myomas are removed since it offers the advantages of a faster recovery and less perioperative morbidity. This makes the first choice most likely not the correct one due to size limitations. Laparoscopic is preferred over open laparotomy due to shorter recovery and overall lower risk of complications, however it is limited by the size and number of fibroids, laparoscopic myomectomy, with or without robotic assistance, is an option for individuals with a uterus less than 17 weeks in size or with a small number (typically less than five). Last choice is off the table since the patient is desiring the preservation of her fertility. Source: UpToDate 425. Acceptable breastfeeding immediately post-partum? a. Direct breastfeeding of mother with HIV b. Extracted breast milk of a mother with HIV c. Direct breastfeeding of mother with active TB d. Extracted milk of mother with active TB A, is D Explanation: HIV is transmitted through breast milk so this makes all the A,s with HIV in them wrong. TB is not transmitted through milk, however, you cannot allow a mother with active TB to directly breastfeed her infant because of airborne transmission of the disease, which makes the last choice the correct A,. Mothers with active pulmonary TB can transmit infection to their infants. Therefore, women with known or suspected active TB at the time of delivery should be separated from the infant until both have been evaluated. Breastfeeding should be encouraged among women on treatment for latent TB and after at least two weeks of treatment for active TB. The small concentrations of first-line antituberculous drugs (isoniazid, rifampin, ethambutol, and pyrazinamide) in breast milk do not produce toxic effects in the nursing infant. Source: UpToDate 426. 34 week preeclampsia patient came with epigastric pain, headache, blurred vision .. Non stress test reassuring, management? a. Mg sulfate + deliver b. Mg and wait for 37 week c. Call anesthesiologists for deliver A, is A Explanation: this patient is presenting with severe preeclampsia, and the management for that is by delivering the fetus as well as giving seizure prophylaxis. In patients with preeclampsia, severe headaches, visual changes, sustained clonus, or a positive Chvostek sign can be prodromal symptoms or signs of eclampsia. Seizure prophylaxis with magnesium sulfate should be instituted in patients with severe preeclampsia during the initial period of stabilization and again during the intrapartum period, and it should be continued for 24 hours postpartum or until there is evidence of resolution of the disease. Source: Toronto notes 2020, page OB25-26, First Aid USMLE step 2 CK 10th edition, page 336, Hacker & Moore's essentials of obstetrics & gynecology, page 190 427. Women want to get pregnant in winter what to give her before pregnancy? a. Rubella b. Influenza c. Varicella A, is B Explanation: All women who are pregnant or might be pregnant during the influenza season should receive the inactivated influenza virus vaccine as soon as it becomes available and before onset of influenza activity in the community, regardless of their stage of pregnancy. Pregnant women with influenza are more likely to develop severe illness and to die than the general population. The other two A,s are correct only if the patient has no evidence of immunity to the infections. Evidence of immunity includes either: • Documentation of age-appropriate completion of vaccination • Laboratory evidence of immunity • Laboratory confirmation of disease And at this day and age, majority of patients are indeed vaccinated, which makes the other choices less likely to be the correct one. Source: UpToDate 428. ECV in 34 weeks pregnant absolute contraindication? a. Previous CS b. Ultrasound finding of low lying placenta c. CTG finding and gestational age A, is B Explanation: Any absolute contraindication to labor and vaginal birth regardless of fetal presentation is an absolute contraindication to ECV

Contraindications to give methotrexate in ectopic pregnancy. A. Far away from medical services

: A. Noncompliant patient is exclusion criteria for Methotrexate. Who knows if they will follow up? Patients need to return for evaluation to know if the treatment worked and if they need a second dose or surgery. Master the Boards USMLE Step 2 CK - 5th Ed, P900. 42

treatment of depression during pregnancy should incorporate the clinical expertise of the mental health clinician, obstetrician, primary health care provider, and pediatrician. The ACOG also recommends that therapy with paroxetine be avoided during pregnancy if possible and that fetuses exposed in early pregnancy be assessed with a fetal echocardiography (ACOG 2008). Other guidelines note that treatment with paroxetine should not be initiated in pregnant women (Bauer 2013). According to the American Psychiatric Association (APA), the risks of medication treatment should be weighed against other treatment options and untreated depression. The use of paroxetine is not recommended as first line therapy during pregnancy. For women who discontinue antidepressant medications during pregnancy and who may be at high risk for postpartum depression, the medications can be restarted following delivery (APA 2010). Treatment algorithms have been developed by the ACOG and the APA for the management of depression in women prior to conception and during pregnancy (Yonkers 2009). Keep in mind that SSRI's in general might be associated with preterm labor, however the data regarding this issue is inconsistent, and for this reason we chose effect on the fetus to be the more likely

. Source: UpToDate 463. Scenario of high suspicion of cervical cancer, asking for the highest diagnostic yield? a. Colposcopy There were no other A,s provided Explanation: the question is incomplete 464. When to test for GDM? a. 24 weeks No other A,s were provided Explanation: Source: Toronto notes 2020, page OB27 465. Mother comes around 2 weeks postpartum complaining of clear discharge that is positive for leukocytes and epithelial cell? a. Reassure b. Urine culture c. Antibiotics A, is A Explanation: The basal portion of the decidua remains after the placenta separates. This decidua divides into two layers: The superficial layer is shed and the deep layer regenerates new endometrium, which covers the entire endometrial cavity by the 16th postpartum day. Normal shedding of blood and decidua is referred to as lochia rubra (red/red brown) and lasts for the first few days (up to 3-4 days) following delivery, this is red due to presence of erythrocytes. Vaginal discharge then becomes increasingly watery, called lochia serosa (pinkish brown), which lasts for two to three weeks (4 days to 10 days in hacker). Ultimately, the discharge turns yellowish white, the lochia alba. Microscopically, lochia consists of serous exudate, erythrocytes, leukocytes, decidua, epithelial cells, and bacteria. Foul-smelling lochia suggests endometritis. Source: UpToDate, Hacker & Moore's essentials of obstetrics & gynecology, page 113 466. Patient with PCOS doesn't want to be pregnant? a. OCP There were no other A,s provided Explanation: the question is incomplete. Source: Toronto notes 2020, page GY24 467. Question about pregnant with symptom of cholecystitis asking for the best management? a. 2nd trimester surgery There were no other A,s provided Explanation: In the guidelines for diagnosis, treatment, and use of laparoscopy for surgical problems during pregnancy (2007), the Society of American Gastrointestinal and Endoscopic Surgeons (SAGES) organization stated that laparoscopic cholecystectomy can be performed safely during any trimester of pregnancy. Of note, laparoscopic is the recommended method of removing the gallbladder. Cholecystitis can be managed medically in pregnancy, and this might have been one of the A, choices as the question is incomplete, so read about management of surgical conditions during pregnancy to cover this topic. Source: UpToDate, Hacker & Moore's essentials of obstetrics & gynecology, page 222 468. Case scenario show a cyst on 9 clock? a. Bartholin cyst No other A,s were provided Explanation: these are usually explained as 4 or 8 o'clock cysts and in some places 5 or 7 o'clock cysts. Source: UpToDate, American Academy of Family Physicians 469. Pregnant with 33 week gestation presented with headache protein urine +3, Bp 150/100 , asking what to do ? a. Immediate C/S b. Delay for a week and give steroid c. Admitted for observation A, is C Explanation: this is a woman with features of severe preeclampsia (headache), with a fetus that is 33-week gestation, in this case the mother can be offered to terminate the pregnancy, or admission and observation. Although cesarean section can be done to these patients, however usually vaginal delivery with induction is first step, unless it is not favorable or there is other obstetric indications for C/S. Delaying and giving steroids is almost a reasonable A,, but it lacks the emphasis that the mother should be admitted for observation, we cannot allow the mother to go out of the hospital for that week. The headache that is under the severe preeclampsia features is a severe headache (ie, incapacitating, "the worst headache i've ever had") or headache that persists and progresses despite analgesic therapy and not accounted for by alternative diagnoses. Source: UpToDate, Toronto notes 2020, page OB25-26, First Aid USMLE step 2 CK 10th edition, page 336 470. Female came to WR (we don't know what the hell is WR maybe ER?) With lower abdominal pain, fever, no rebound tenderness, negative pregnancy test, diagnosis? a. Pelvic inflammatory disease No other A,s were provided. Explanation: Source: Toronto notes 2020, page GY30-31 471. Female came to ER, RLQ pain, vomiting, there is mass on palpation, no fever, negative pregnancy test, diagnosis? a. Ovarian torsion No other A,s were provided Explanation: Sudden onset of severe lower abdominal pain in the presence of an adnexal mass is presumptive evidence of ovarian torsion. Another differential that can look similar might be rupture of an ovarian cyst, it is often accompanied by sonographic evidence of hematoperitoneum or free fluid in the pelvis

Pregnant patient, fetus died, and she is going through AP + DIC , cervix dilated 6 cm, your next management? A. Vaginal delivery B. CS C. Total abdominal hysterectomy

: A. Note: No DIC present: Delivery must be deferred for a number of days to allow for an appropriate grief response to begin. Or if the patient wishes conservative management, follow weekly serial DIC lab tests. 90% of pts start spontaneous labor after 2 weeks. Mode of delivery: - If < 23 weeks + no fetal autopsy indicated: D & E - If ≥ 23 weeks: IOL w/ vaginal prostaglandin **** CS IS ALMOST NEVER APPROPRIATE FOR DEAD FETUS***** 10

What is the lymph drainage of the fundus of the uterus? A. Paraaortic

: A. 15

this is often accompanied by involvement of the neighboring pelvic organs. Lower abdominal pain is the cardinal presenting symptom in women with PID. The abdominal pain is usually bilateral and rarely of more than two weeks' duration [2-4]. The character of the pain is variable, and in some cases, may be quite subtle. The recent onset of pain that worsens during coitus or with jarring movement may be the only presenting symptom of PID. Source: https://www.uptodate.com/contents/pelvic-inflammatory-disease-clinical- manifestations-and- diagnosis?search=Salpingitis&source=search_result&selectedTitle=1~40&usage_type=defau lt&display_rank=1#H854376396 352. Mild dysfunctional uterine bleeding. Not menopause. On exam nothing appear in uterus, no adnexal mass. What to do? A- MRI Not clear The

N/A Explanation: The question lack so many finding and results. Note for more information regarding Abnormal uterine bleeding please referee to https://www.uptodate.com/contents/abnormal-uterine-bleeding-in-adolescents-evaluation- and-approach-to- diagnosis?search=dysfunctional%20uterine%20bleeding&source=search_result&selectedTitl e=3~150&usage_type=default&display_rank=3#H2 353. Female with painful genital ulcer, rash, conjunctivitis, there was a test done and was +ve called anti coccal something antigen, diagnosis? I think STD A- Gonorrhea The A, is: A Explanation: nucleic acid amplification testing (NAAT) is the test of choice for the initial microbiologic diagnosis of N. gonorrhoeae infection, although culture remains an important diagnostic tool when antibiotic resistance is suspected. Source: https://www.uptodate.com/contents/clinical-manifestations-and-diagnosis-of- neisseria-gonorrhoeae-infection-in-adults-and- adolescents?search=Gonorrhea&source=search_result&selectedTitle=2~150&usage_type=de fault&display_rank=2 354. Patient taking Combined oral contraceptives what contradicts it use? A. history of DVT The A, is: A Source: https://www.uptodate.com/contents/combined-estrogen-progestin-oral- contraceptives-patient-selection-counseling-and- use?search=contraceptives&source=search_result&selectedTitle=2~150&usage_type=default &display_rank=2#H3796391448 355. Contraindication to EVC A. Bicornate uterus The A, is : A Explanation: ECV is generally also contraindicated in the following settings because they are associated with a low likelihood of successful version and/or increased risk of fetal harm from the procedure: ● Severe oligohydramnios. ● Nonreassuring fetal monitoring test results. ● Hyperextended fetal head. ● Significant fetal or uterine anomaly (eg, hydrocephaly, septate uterus). ● Placental abruption. ● Ruptured membranes. ● Active labor with fetal descent. Source: https://www.uptodate.com/contents/external-cephalic- version?search=External%20cephalic%20version&source=search_result&selectedTitle=1~15 0&usage_type=default&display_rank=1#H9 356. Mother his baby die what you will do? A. Let the mother take some time with his baby The A, is: A Source: not found 357. Patient postmenopausal want HRT? A. refuse to give Note: question not clear please referee to this article for more information: https://www.uptodate.com/contents/menopausal-hormone-therapy-benefits-and- risks?search=HRT&source=search_result&selectedTitle=1~150&usage_type=default&displa y_rank=1#H256689819 358. Pregnant smoker what will help her stop ? A. behavioral therapy the A, is: A Explanation: Consistent with society recommendations, we offer smoking cessation counseling to all pregnant women and view counseling, with continued monitoring, as the first-line intervention for smoking cessation. Source: https://www.uptodate.com/contents/cigarette-smoking-in-pregnancy-cessation- strategies-and-treatment- options?search=pregnant%20smoker&source=search_result&selectedTitle=1~150&usage_ty pe=default&display_rank=1 359. . Pregnant lady in 15 WK gest comes with marked upper & lower limp edema. BP 150/95 What is your best management? A. complete evaluation B. Atenolol The A, is: A Explanation: Many pregnant women have edema, whether or not they have preeclampsia. However, sudden and rapid weight gain (eg, >5 lb/week [2.3 kg]) and facial edema are more common in women who develop preeclampsia

Pregnant 28 weeks with progressive increase breast mass, thinning of skin due to increase pressure but normal with no sign of inflammation. what next step? A. Reassure and reevaluate after delivery B. Bilateral breast US C. Mammography

: A. 51

What is the lie of the fetus ? A. Longitudinal B. Complete C. Incomplete D. Frank

: A. 8

Postmenopausal female, vaginal itching and irritation, watery vaginal discharge. Diagnosis? A. Trichomoniasis B. Atrophic vaginitis C. Candida D. BV

: B. 30

Endometrial hyperplasia with atypia + menopause, management? A. Oral progesterone B. Hysterectomy C. Colonoscopy D.

: B. Toronto Notes 2020, P534 9

Explanation: After a multidisciplinary discussion, the decision was made to deliver through a cesarean section with preparation for a possible EXIT procedure. Members of the team were gathered to prepare for the delivery and perform an EXIT procedure to secure an airway if necessary. Source: https://www.ncbi.nlm.nih.gov/pmc/articles/PMC6230665/ Note: A is mostly the right

, but no clear reference was found supporting it. 313. Patient fully dilated for 2 hours, head in station - 2, patient got exhausted from pushing A. Cs B. Wait 2 hours C. Ventouse the A, is: A explanation: Use of either forceps or vacuum is reasonable when an operative intervention to complete labor is indicated and operative vaginal delivery can be safely and readily accomplished

Weeks? pregnant, history of 2 2nd trimester abortions she did D&C once presented with vaginal bleeding and cervical incompetence. What is the cause of her vaginal bleeding? A. Cervical incompetence B. Asherman's syndrome C. Chromosomal anomaly

: If 1st trimester → chromosomal If 2nd trimester → cervical Recurrent SAB: Two or more consecutive SABs or three SABs in 1 year, causes dependent on timing. To determine possible causes, karyotype both parents, perform work-up on mother for hypercoagulability, evaluate uterine anatomy. ■ Early (< 12 weeks): Chromosomal abnormalities likely cause. ■ Late (12-20 weeks): Hypercoagulable states (eg, antiphospholipid syndrome, SLE, factor V Leiden, protein S deficiency). First Aid USMLE Step 2 CK - 10th Ed, P324 16

29 years old female had a negative pap smear. What to do? A. Repeat with cytology B. Do it after 3 years

: B. In asymptomatic, immunocompetent women, cervical cancer screening with Pap smear starts at age 21, regardless of sexual activity. In women younger than 30 years old, screening with Pap alone is done every 3 years. After age 30, either do Pap smear alone every 3 years or do Pap smear with HPV co-testing every 5 years for as long as both tests are negative. Master the Boards USMLE Step 2 CK - 5th Ed, P972 41

Patient with signs and symptoms of preeclampsia. What will indicate IUGR? A. Elevated liver enzymes B. Oligohydramnios C. Polyhydramnios

: B. Preeclampsia causes uteroplacental insufficiency which result in deficiency in amniotic fluid as in oligohydramnios. One of its fetal sequels is IUGR. 31

Post menopause women? A. LH B. FSH C. Estrogen D. Progesterone

: B. • increased levels of FSH (>35 IU/L) and LH (FSH>LH). • decreased levels of estradiol (later). Toronto Notes 2020, P528 6

Couple come by infertility after investigation done show bilateral fallopian tube obstruction what will do? A. Tell wife B. Husband C. Both

: C. 11

Female in active labor once baby present vertex , and the doctor saw cord coming out what 1st thing to do? A. Reexamine after 2 hours B. Close monitoring of baby C. CS D. Instrumental delivery

: C. Toronto Notes 2020, P884 35

Image showed pearl red single lesion at the cervix ( not clear!). What to do? A. Follow-up after 6 months B. Simple excision at the clinic C. Colposcopy D. Reassurance

: C. If polyp → excise or reassure If only lesion → colposcopy If obvious suspicious mass → Biopsy colposcopy is a clinical procedure that facilitates identification and biopsy of suspicious cells. • in colposcopy: apply acetic acid and identify acetowhite lesions, punctation, mosaicism, and abnormal blood vessels to guide cervical biopsy. Toronto Notes 2020, P539 29

G2 P1+0 , 10 weeks pregnant has gallbladder stone years on medication and now she is having pain when to do cholecystectomy ? A. Do cholecystectomy immediately B. Do cholecystectomy in the first trimester C. Do cholecystectomy in the second trimester D. Do cholecystectomy after delivery

: C. The decision to offer cholecystectomy for biliary colic during pregnancy is based upon the clinical scenario, gestational age, and other factors. For pregnant patients with a first episode of biliary colic, we suggest initial supportive care. If symptoms of biliary colic (pain, nausea/vomiting) cannot be controlled, surgery should be offered. If recurrent bouts of biliary colic occur, we feel that primary surgical management during pregnancy is reasonable because recurrence is common with conservative therapy and surgical therapy appears to be safe for mother and. For these women, cholecystectomy is ideally performed in the second or early third trimester. - If biliary colic occurs near term, we avoid cholecystectomy and reevaluate the patient after delivery. We generally wait six weeks following delivery to allow the mother to recover from the delivery, bond with the infant, and regain her strength. UpToDate. 2

therefore, it is the preferred medical treatment for pregnant women. Clearance rates are highest and recurrence rates lowest when TCA is used in the second half of the pregnancy. Cryoablation is also considered a safe and effective treatment for use in pregnancy. We prefer to begin with TCA treatment because it has fewer side effects than cryoablation. Source:https://www.uptodate.com/contents/condylomata-acuminata-anogenital-warts- treatment-of-vulvar-and-vaginal- warts?search=Condylomata%20acuminata%20in%20pregnancy&source=search_result&sele ctedTitle=1~125&usage_type=default&display_rank=1#H6 305. Pregnant lady with hx or recurrent HSV infections, what to do next ? A. Speculum vaginal exam. The

and explanation are: PCR is especially advantageous in the setting of a recurrent episode, as it has greater sensitivity in recurrent infection than viral culture. Most recurrent episodes are short-lived and do not require intervention during the first 35 weeks. Suppressive therapy with acyclovir 400 mg three times daily from 36 weeks of gestation to delivery reduces the frequency of symptomatic HSV recurrence at the onset of labor, and thus reduces the need for cesarean delivery. Source:https://www.uptodate.com/contents/genital-herpes-simplex-virus-infection-and- pregnancy?search=recurrent%20HSV%20in%20pregnancy&source=search_result&selectedT itle=1~150&usage_type=default&display_rank=1#H27 306. Woman with normal vitals and labs without any complaints except Some white vaginal discharge with normal pH

age >65, only with history of significant CIN, After total hysterectomy cytology not needed. Source: hacker and Moore p 10, bmj, uptodate 251. Female presented with valvular lesion (not described!). Which one of the following is the most likely diagnosis? A. Squamous B. Adenocarcinoma C. Squamo-adenocarcinoma

is A Explanation: Most valvular neoplasms are squamous cell carcinomas, and they occur mainly in postmenopausal women. Source: hacker and Moore p 449 252. Female had a lichen planus in her valvular which progressed to be cancerous. Which one of the following is most likely? A. Squamous carcinoma A, is A Source: hacker and Moore p 237 https://www.sciencedirect.com/science/article/pii/S0190962214015461 https://www.mdedge.com/dermatology/article/107883/nonmelanoma-skin-cancer/transition- lichen-sclerosus-squamous-cell 253. 40 years old pregnant, 11 weeks gestation, was found to have twin. One of them has 5mm NT: A. High risk of NT enlargement at 18 m B. Turner C. Neural tube defect D. Cardiomyopathy A, is most likely B Explanation: increased nuchal translucency (an echo-free area at the back of the fetal neck) between 10 and 14 weeks' gestational age. Increased nuchal translucency has been associated with both chromosomal abnormalities and other congenital anomalies. I couldn't find the exact value. Source hacker and Moore p 86 254. Female presented with sudden acute onset of abdominal pain. US showed mass in the ovary. What is the diagnosis? A. Rupture of ovarian cyst B. Ovarian torsion A, is B or A Explanation: more information is needed to differentiate. ovarian torsion is the twist of the ovaries around it's ligaments, it's considered an emergency. Excluding appendicitis is required before diagnosing OT. Rupture of an ovarian cyst may be asymptomatic or associated with a sudden onset of unilateral lower abdominal pain. Abdominal may tender on touch. Us finding are adnexal mass and fluid in the pelvis. Source: bmj, uptodate 255. dizygotic means: A. Monochorionic & monoamniotic with same gender B. Monochorionic & Diamniotic with sesame gender C. Dichorionic & Diamniotic regardless gender A, is C Source: hacker and Moore p 171 256. 37 years old female, was breastfeeding 5 months ago, is now pregnant again, but she didn't know when that happen. What's the most beneficial method to measure her gestational age? A. US at 18 months B. US at first trimester as soon as possible A, is B Source: hacker and Moore p 14 257. What is the management of carcinoma in Situ in Vulva? A. Vulvectomy A, is ... Explanation: the appropriate A, is WIDE LOCAL EXCISION if not in choices A is ok, but it is usually done to lesions occupying most of the vulva. Source: hacker and Moore p 450 258. Pregnant preterm with HTN 150/75, what to do A. Admit for observation B. Admit for delivery C. Discharge The A, is: N/A Explanation: Since the question is not clear regarding the patient exact diagnosis please referee to the 14th chapter of Hacker Moore, sixth edition 259. Pregnant 38 weeks of gestation with prevaginal bleeding and fetal bradycardia, what will be the most important intervention? A. Oxygen mask B. Search for cord prolapse C. Arrange c/s The A, is: C Explanation: Fetal bradycardia/prolonged deceleration without loss of variability — Fetal bradycardia (below 110 bpm) or a prolonged deceleration is approached in a similar way clinically. The causes of prolonged deceleration or fetal bradycardia include: • Rapid fetal descent • Cord prolapse • Placental abruption • Maternal hypotension • Uterine rupture • Tachysystole Cesarean delivery is indicated for severe, repetitive decelerations and a FHR tracing indicative of developing acidosis. Another circumstance that may require intervention is a prolonged deceleration. This condition occurs when the FHR falls to 60 to 90 beats/min for more than 2 minutes. Source: Hacker Moore Page 134, sixth edition. https://www.uptodate.com/contents/intrapartum-category-i-ii-and-iii-fetal-heart-rate-tracings- management?search=fetal%20bradycardia&source=search_result&selectedTitle=2~63&usag e_type=default&display_rank=2 260. 7 y girl with pubic hair growth no breast growth or any hair in axilla mother concern? The A, is: Premature pubarche Explanation: Premature pubarche refers to the physical finding of isolated appearance of sexual hair (ie, sexual hair without breast development in girls, or without testicular enlargement in boys) before the age of eight years in girls and nine years in boys, their plasma concentrations of dehydroepiandrosterone sulfate (DHEAS) and androstenedione are increased compared with controls but lower than those of typical premature adrenarche. Source:https://www.uptodate.com/contents/premature- adrenarche?search=pubarche&source=search_result&selectedTitle=1~15&usage_type=defau lt&display_rank=1 261. Postpartum lady on clexane (Enoxaparin sodium) developed SOB CT angio confirm PE what is your management? A. Start heparin B. Thromblytic The A, is: A Explanation Thrombolytic therapy should be reserved for pregnant or postpartum patients with life-threatening acute PE (ie, persistent and severe hypotension due to the PE). Source:https://www.uptodate.com/contents/deep-vein-thrombosis-and-pulmonary-embolism- in-pregnancy- treatment?search=Pulmonary%20embolism%20in%20postpartum&source=search_result&sel ectedTitle=2~150&usage_type=default&display_rank=2#H12 262. lady with abdx pain and +ve pregnancy test us shows empty uterus and adnexal mass of 4 cm Bhcg of 5000 She lives 80 km away from hospital What is the contraindication for medical Tx ? A. Bhcg result B. US result C. Social Hx of the pt The A, is: C Explanation: Women may also reasonably choose surgery if they value a treatment that is of shorter duration and involves less follow-up and are willing to take the risks and recovery time associated with surgery. Source:https://www.uptodate.com/contents/ectopic-pregnancy-choosing-a- treatment?search=ectopic%20pregnancy%20surgery&source=search_result&selectedTitle=2 ~150&usage_type=default&display_rank=2 263. Pic of chloasma The A, is: N/A Explanation: referee to the second chapter of Hacker Moore, sixth edition. 264. postpartum bleeding, you gave oxytocin, what will you do next? The A, is: ergonovine maleate or methylergonovine Explanation: If the uterus remains atonic and the placental site bleeding continues, 0.2 mg of ergonovine maleate or methylergonovine may be given intramuscularly. The ergot drugs are relatively contraindicated in patients with hypertension because the smooth muscle-constricting effects of these drugs may also increase vascular tone and thus increase blood pressure to dangerous levels. Source: Hacker Moore Page 143-144, sixth edition. 265. Episiotomy involving the mucosa of rectum what is the stage of anal tear? A. 1 B. 2 C. 3 D. 4 The A, is: D Explanation: Fourth-degree lacerations - Injury to the perineum that involves both the anal sphincter complex (EAS and IAS) and anal mucosa. Source: https://www.uptodate.com/contents/repair-of-perineal-and-other-lacerations- associated-with childbirth?search=Episiotomy%20and%20anal%20tear&source=search_result&selectedTitle =2~150&usage_type=default&display_rank=2 266. Treatment of hematoma post episiotomy? The A, and explanation are: if small hematoma less than 5cm: conservative management including analgesia and application of cold packs. If large hematoma more than 5cm: surgical evacuation with primary closure and compression for 12-24 hours. Surgical intervention is necessary if there is expansion of the hematoma on physical examination or imaging studies or a falling hematocrit, as persistent hemorrhage can lead to hemodynamic instability or put the tissue at risk of necrosis. Source: https://www.uptodate.com/contents/management-of-hematomas-incurred-as-a-result- of-obstetrical- delivery?search=Episiotomy%20and%20hematoma&topicRef=111631&source=see_link#H1 0 267. pregnant not feels fetal movements for one day biophysical score 8 +ve . A repeated biophysical profile after one week? The A, and explanation are: • 10/10, 8/8 (nonstress test omitted), or 8/10 (-2 points for either fetal movement, tone, or breathing but not amniotic fluid) is a normal test result. • 6/10 (-4 points for two of fetal movement, tone, breathing, but +2 points for amniotic fluid) is an equivocal test result because a significant possibility of developing fetal asphyxia cannot be excluded. The test is repeated within 24 hours to see if one of the absent acute variables returns to normal or, if the patient is at or near term, delivery is a reasonable option. • 6/10 or 8/10 with 0 points for amniotic fluid is an abnormal test, as the risk of fetal asphyxia within one week is 89/1000 with expectant management. • 0 to 4/10 is abnormal

Explanation: the full scenario is recommended, however, common sites are ovaries, the broad ligament, the p toneal surfaces of the cul-de-sac (including the uterosacral ligaments and posterior cervix), and the rectovaginal septum. Source: hacker and Moore p314 169. similar scenario asking about the modality of investigation for definitive dx: A. Laparoscopy

is A Explanation: The definitive diagnosis is generally made on the basis of the characteristic gross and histologic findings obtained at laparoscopy or laparotomy. Source: hacker and Moore p 317 170. multiple cases about ectopic , all easy and diagnosed by lab or clinical examination and US, Also management of ectopic when to do medical and when surgical. Explanation: early, unruptured ectopic pregnancies are managed with methotrexate Surgery is done in case medical management failed or the patient is unstable, or in patients whose diagnosis is uncertain. Refer to hacker and Moore p 304 for more ectopic pregnancy info Source: hacker and Moore p310 171. one case about preeclampsia very easy and diagnosed from the scenario and dipstick. Explanation: preeclampsia is the presence of new-onset hypertension in the latter half of gestation (≥140 mm Hg or diastolic blood pressure ≥90 mm Hg on two occasions 4 hours apart, accompanied by new-onset proteinuria (≥0.3 g of protein in a timed 24-hour urine collection or a protein/creatinine ratio ≥0.3) and/or other evidence of organ dysfunction. Source: hacker and Moore p 184 172. What is the mechanism of action of metformin in treating polycystic ovaries? A. Increase insulin sensitivity A, is A Explanation: insulin-sensitizing agents such as metformin may be used to reduce insulin resistance and anovulation. Source: hacker and Moore 394 173. Pregnant woman with bronchial asthma. She has shortness of breath, wheezing, and some times night attacks of asthma. What is the treatment? A. SABA AND ICS B. SABA AND Oral Cs C. SABA AND ABX D. SABA AND iv fluids A, is A Explanation: Systemic steroids are associated with anomalies like cleft palates. Source: hacker and Moore p218 174. Types of abortion. Explanation: refer to hacker and Moore p 80. 175. Lots of questions about abnormal uterine bleeding. Explanation: refer to hacker and Moore p 322 176. Female with history of two pregnancies loss. She tried to conceive, but she didn't succeed. She wants to improve her immunity. What vaccines are appropriate for her? A. rubella B. influenzas C. pneumococcal D. varicella. A, is B Explanation: all the above mentioned vaccines can be given preconception at least one month, however, only influenza and tetanus vaccines can be given throughout the pregnancy. So if the question meant antenatally influenza in the A,, if it meant preconception all of them are correct but rubella might be very important causes of miscarriage Source: hacker and Moore p 288, uptodate 177. Patient with suspected placenta abruption. What is the next step? A. pelvic US B. call rapid response team and multidisciplinary C. admit in SICU under gynecology D. transfuse FFP A, is A or B Explanation: Ultrasound is among the first tests to do in all sources I found, the reason for doing US is to to exclude placenta previa, retro placental hematoma, preplacental hematoma, increased placental thickness are findings that appear helpful in diagnosing placenta abruption (12%-25). However, B could be a safe A, too as monitoring the mother and the baby along with other investigations could be involved in this choice. Source: uptodate hacker and Moore p 139 178. A case of patient with choriocarcinoma: having uterus size more than GA and hemoptysis. What is the next step? A. Biopsy the mass B. Admit for S&E C. Admit for staging and chemotherapy D. Admit for hysterectomy A, is C Explanation: choriocarcinoma is a malignant GTD, the diagnostic method is the same of hydatiform moles, high b-hcg with ultrasound finding, CBC and coagulation profile, lft and RFT, chest X-ray. However, in choriocarcinoma CT pelvis, abdominal, head are done for possible metastasis. lumbar puncture should be performed if the CT scan of the brain is normal, because simultaneous evaluation of the β-hCG level in the cerebrospinal fluid and serum may allow detection of early cerebral metastases. Treatment is determined based on the prognosis, nonmetastatic and good prognosis metastatic will receive a single agent chemotherapy ( methotrexate, or actinomycin D) methotrexate could alternate every other day with leocovorin. Hysterectomy is considered in woman aged >40 with previous Hx of molar pregnancy as the risk of choriocarcinoma increases. Metastatic poor prognosis receive a combination therapy called modified "Bagshawe" regimen. Surgery could be considered is resistant cases. Source: hacker and Moore p 474, Kaplan step2 p 215 179. Female had epidural anesthesia. Regarding Pudendal nerve sedation, which of the following will not be affected? A. Rectum A, is A Explanation: epidural anesthesia is a form of pain relief provided to first and second stage of labor by injecting high concentrations of local anesthetics, the effect may be sustained by adding more drugs and may reach the level of T10. Pudendal block is a mode of conduction anesthesia that involves bilateral transvaginal local anesthetic injection to block the pudendal nerve (s2-s4). It relieves pain on the external genitalia and perianal area along with the lower half of the anal canal. The only A, provided in this question is rectum, however, the lower third of the rectum is affected by pudendal block, so if there is a more convincing A, choose it instead. Source: hacker and Moore p 118, Kaplan step2 p 131, snell anatomy p 749 180. Patient with symptoms and signs of Bacterial vaginosis. If a sample was taken and analyzed, what would be there? A. overproduction of Lactobacillus B. multi-nucleated giant cells C. Granular epithelial cells A, is C Explanation: lactobacillus in BV is decreased in number or even absent in some cases, hence Ph increases. A vaginal swap will show some epithelial cells covered with bacteria called clue cells. Source: hacker and Moore p277, Kaplan step 2 p175 181. Patient with endometriosis, and she is concerned about cancer. What will you tell her? A. There's no relation between them A, is epithelial ovarian cancer Explanation: overall increased risk of ovarian cancer (endometrioid, clear cell, and serous types) in women with ovarian endometriomas (ie, ovarian endometriosis

as a result, the cord blood concentration approximates the maternal serum concentration. Maternal therapy causes a decrease in baseline fetal heart rate, which generally remains within the normal range, and a decrease in fetal heart rate variability, which may be absent or minima. Source: uptodate, preeclampsia management. 147. Married when to do pap smear? A. 20-24y

is A Explanation: within 3 years of sexual intercourse, or by age 21 Source: hacker and Moore p 430 148. case in women with white discharge + pseudohyphae in microscopic? A. Candida A, is A Explanation: matches vulvovaginosis candida, pseudohyphae are noticed, and some candida types show hyphae. 149. Candida management? A. Terconazole cream topically A, is A Explanation: assuming that the question meant vulvovaginal candidiasis topical terconazole or oral fluconazole among other azoles are equally effective. Source: bmj, hacker and Moore p 278 150. Bacterial vaginosis case treatment ? A. Metronidazole A, is A Explanation: treatment of BV should inhibit anaerobes but not vaginal lactobacilli, metronidazole does that, Clindamycin is an alternative. Source: hacker and Moore p 277 151. Pt pregnant have anemia and treat with iron supplements at 20 week came with fatigue and tiredness hg low mcv low dx? A-IDA B-b thalassemia A, is A Explanation: whether the question about why she is taking iron supplements or why she came with new symptoms at 20 weeks, IDA will be a good A, cuz the demand doubles in pregnancy so changing the dosage could be a choice. B thalassemia is also likely cuz it presents with the same symptoms and laboratory values mentioned above if not detected and managed before. Source: uptodate 152. Solid mass of ovary + ca125 high what to do? A. Gynecology oncology referral A, is A Explanation: solid ovarian masses have wide deferential diagnosis to investigate, in addition to ca125 that is increased by a number or other conditions too. Source: uptodate, American academy of family physicians. 153. case clear ask about urge incontinence Explanation: reference to urinary incontinence at uptodate for more information. 154. carcinoma in Situ in vulva management? A. Wide local excision A, is A Explanation: in situ neoplasms of the vulva are superficially existed with a margin of 5mm, or vulvectomy might be done in case of widespread lesion. Source: hacker and Moore p 450 155. 19 y/o female married on OCPs develop sing of DVT in the lower limb (lower extremity pain and swelling), what is the cause? A. OCPs A, is A Source: hacker and Moore p 332 156. CTG pic shows "Late deceleration" Dx: A. Uteroplacental insufficiency A, is A Source: hacker and Moore p 132 157. CTG pic shows "Early deceleration" Dx: A. Head compression A, is A Source: hacker and Moore p132 158. multiple cases about abortion, easy and straight forward ) Explanation: refer to hacker and Moore p 80 159. contraindications of ECV: A. low lie placenta A, is A Explanation: Evidence of uteroplacental insufficiency, placenta previa, non reassuring fetal monitoring, hypertension, intrauterine growth restriction, oligohydramnios, or a history of previous uterine surgery are contraindications to external cephalic version. Source: hacker and Moore p 178 160. treatment of Trichomonas vaginalis: A. metronidazole B. doxycycline C. Clindamycin A, is A Source: hacker and Moore p 278 161. 2 cases about abruptio placenta, dx and management, easy Explanation: refer to hacker and Moore p 139 162. female multipara, complain of vaginal heaviness, and manual manipulation to empty her bladder, dx: A. cystocele B. rectocele A, is A Explanation: vaginal heaviness and discomfort are nonspecific for one type, however, Manuel manipulation to empty the bladder and urinary incontenance indicates the presence of a cystocele. Manual manipulation for complete bowl elimination indicates rectocele Source: hacker and Moore p 292 163. what is the most important sign of premenstrual syndrome: A. depression B. mood swings C. irritability D. anxiety A, is C Explanation: symptoms such as depression, angry outbursts, irritability, and anxiety, and somatic symptoms such as breast pain, bloating and swelling, and headache, these symptoms disappear once the menstrual cycle starts. Hacker doesn't mention the most important symptom, however, uptodate mentioned that more prominent anger and irritability are used to differentiate premenstrual syndrome from premenstrual dysphoric disorder, which according to uptodate is a severe form of PS. Source: uptodate, hacker and Moore p 414 164. Case of polycystic ovarian syndrome, she complain of infertility and US showed multiple small cyst on the peripheries of the ovaries: Explanation: refer to hacker and Moore p 390 165. Infertility after chlamydia infection, what is the cause: A. tubal B. cervical C. vaginal A, is A Source: hacker and Moore p 305 166. Similar question but asking about investigation: A. Hystrosalbingography A, is A Source: hacker and Moore p 281 167. Case of uterine fibroid, straight forward and easy: Explanation: refer to hacker and Moore p 248 168. case of endometriosis, the clue from scenario is mass on uterosacral ligament i think

however, this has not been confirmed as a predictor of successful treatment. (See 'Factors that impact efficacy' below.) • Patients are willing and able to comply with post-treatment follow-up and have access to emergency medical services within a reasonable time frame in case of a ruptured fallopian tube. Source: uptodate 221. Pregnant lady comes with bleeding after intercourse. The couple feels guilty about this because they had pervious abortion and they are overwhelmed by that. How will you deal with them? A. Acknowledge their feeling and empathize with them

is A Source: no source was found 222. Pregnant lady, 34 weeks gestation, suffered an IUFD. She feels guilty because she smokes. How will you deal with her? A. Tell her to wait for autopsy result B. sympathize with her A, is B Source: no source was found 223. Newly-wed 26 woman comes to the clinic asking about pap smear test. what will you do? A. Do it now B. No need C. Do it when she is 30y A, is A Source: hacker and Moore p 11 224. A woman wants to be pregnant. She is on ARB for HTN. What advice will you give her? A. Switch from ARB to ACE A, is switch to methyldopa or labetalol or CCB Explanation: methyldopa is the safest, ACE and ARB should be avoided. Source: hacker and Moore p 192 225. Pregnant lady, 9 weeks gestation, presented with blood pressure 150/90 on two occasions. What is the best treatment? A. Methyldopa B. ACE C. Diuretics A, is A Explanation: methyldopa is the safest to be used in pregnancy the goal is 130/80 mm Hg to 140/90 mm Hg. Source: hacker and Moore p 192 226. A woman comes to ER with Right iliac fossa pain and vomiting. There is a mass on palpation, no fever, and negative pregnancy test. What is the most likely diagnosis? A. Ovarian torsion B. Appendicitis C. Rupture ovarian cyst A, is A Explanation: ovarian torsion is the twist of the ovaries around it's ligaments, it's considered an emergency. Excluding appendicitis is required before diagnosing OT. Source: bmj 227. Newly-wed woman came to ER with lower abdominal pain, fever, no rebound tenderness, and negative pregnancy test. What is the most likely diagnosis? A. PID B. Ovarian torsion C. Appendicitis A, is A Explanation most likely Dx is PID cuz of newly wed and the active sexual intercorse. Appendicitis should be excluded first. Ovarian torsion is likely if there was adnexal mass and tenderness. Source: hacker and Moore p 280, bmj 228. 24 years old woman came to ER with right iliac fossa pain, fever, diarrhea, rebound tenderness, and negative pregnancy test. What is the most likely diagnosis? A. PID B. Ovarian torsion C. Appendicitis A, is C Explanation: the most likely diagnosis is appendicitis cuz of rebound tenderness (peritonitis), however, ovarian torsion should be excluded first. Source, bmj 229. 38 years old woman came to the clinic. Her menstruation is every 5 to 6 weeks. How to test for ovulation? A. FSH B. LH C. Estrogen D. 21 Progesterone A, is D Explanation: measurement of the serum progesterone level in the mid-luteal phase, 18 to 24 days after the onset of menses or seven days before the next menses are expected. Normal mid-luteal phase progesterone levels are between 6 and 25 ng/mL. Source: uptodate 230. Female patient who has three children diagnosed as endometrioma, which was removed 2 years ago, and right ovary cyst. She presented to the clinic with mild to moderate dysmenorrhea and dyspareunia during intercourse and chronic lower abdominal pain. Pelvic ultrasound shows Left ovary endometrioma cyst 6x7 in size. A. removal of cyst B. aspiration of cyst content under ultrasound guidance C. immediate hysterectomy and salpingectomy oophorectomy D. removal of cyst by laparoscopic ablation of endometrioma spots. A, is closest C Explanation: hacker and Moore mentioned that endometriomas larger than 3cm are probably going to respond only to surgery. The type of surgery is controversial, and depends on the patients' preference of fertility, laparoscopic cystictomy is done. In recurrent endometriomas, unilateral oophorectomy is the choice and definitive treatment. Bilateral oophorectomy or hysterectomy with bilateral salpingo-oophorectomy is reserved for women who have debilitating symptoms, have failed other therapies, and have completed childbearing. So C is the closest, however, not the first. Source: uptodate 231. Which of the following indicates severe preeclampsia? A. Abdominal pain B. Low platelets A, is B Explanation: criteria of sever preeclampsia includes: • Severe hypertension (systolic BP ≥160 mm Hg or diastolic BP ≥110 mm Hg) at rest on two occasions at least 4 hr apart* • Renal insufficiency (serum Cr >1.1 mg/dL or doubling of baseline values) • Cerebral or visual disturbances • Pulmonary edema • Epigastric or right upper quadrant pain • Elevated liver enzymes (AST or ALT at least two times normal level) • Thrombocytopenia (platelet count <100,000/µL)( HELLP) The note below says that the question came to her as lower abdominal pain, while it is epigastric in severe preeclampsia. Source: hacker and Moore p 184 Notes: two other choices missing in the scenario that I got on the exam, the patient had lower abdominal pain, therefore the A, to that question is platelets since patients with severe preeclampsia have EPIGASTRIC abdominal pain 232. 28 years old female complaining of dysmenorrhea and dyspareunia. Pelvic ultrasound shows a hypoechoic mass (ground glass). what statement is true? A. Highly malignant B. Partner need to be treated C. Antibiotic should be given D. Future fertility is a concern A, is D Explanation: the discretion is more likely to an endometriosis. 30 to 50 percent are estimated to have infertility. Of women who present with infertility, 25 to 50 percent are estimated to also have endometriosis. Source: uptodate 233. Scenario of female with burning urination and foul smell. The cervix shows strawberry appearance: A. Trichomoniasis A, is A Explanation: Trichomonas vaginitis is associated with a profuse, purulent, malodorous vaginal discharge that may be accompanied by vulvar pruritus. Strawberry appearance in the cervix is associated with high concentration of the organism. Source: hacker and Moore p 278 234. Bartholin cancer question self-study Explanation: reference to hacker and Moore p 454 235. Case of cTG with persistent bradycardia, what is the cause? Explanation: Cord compression Uteroplacental insufficiency Maternal hypoxia Fetal hemorrhage Source: uptodate 236. Case about a pregnant lady with symptoms of hyperthyroidism. What is the management? A. Probothuracil B. Carbamezole A, is A Explanation: Depend on which trimester FIRST PTU second and third methimasole Source: hacker and Moore p 207 237. Question about a pregnant lady with HIV and another one with herpes. Asking about best mode of delivery: A. CS A, is A Explanation: recommendations say that antivirals at 36w and CS will reduce the risk of transmission to the baby. Transmission of herpes simplex virus (HSV) to the neonate usually occurs during labor and delivery as a result of direct contact with virus shed from infected sites. Source: uptodate 238. 3 questions of pregnant with history of sudden onset of abdominal pain and bleeding. Asking about the diagnosis: A. Abruptio placentae A, is A Explanation: Clinically, the diagnosis of a placental abruption is suspected if a patient presents with painful vaginal bleeding in association with uterine tenderness, hyperactivity, and increased tone. US is done to exclude placenta previa as they can coexist. The most common of these risk factors is maternal hypertension. Source: hacker and Moore p 139 239. Woman treated for pelvic inflammatory disease with Ceftriaxone but still not healed. What is the organism? A. Gonorrhea B. Chlamydia C. HSV D. HPV A, is B Explanation: ceftriaxon is used for gonococcal infection alone ore concurrent with chlamydia infection. Chlamydia first line is Azithromycin and doxycycline. Source: hacker and Moore p 280, uptodate 240. Lymphatic drainage of uterine fundus cancer? A. Internal Iliac B. Superficial Inguinal C. Para aortic D. Deep Inguinal A, is C Source: snell clinical anatomy p 872 241. What is the cause of post-partum and cesarean section fever? A. Retained products of conception B. Endometritis A, is B Explanation: the most common cause of puerperal fever, is designated endomyometritis. Cesarean delivery, particularly after labor or rupture of the membranes of any duration, is the most accurate predictor of postpartum endomyometritis (PPE). Risk factors for include prolonged labor or rupture of the membranes, presence of bacterial vaginosis, frequent vaginal examinations, and use of internal fetal monitoring. Source: hacker and Moore p 286 242. Which of the following antibiotics is contraindicated in pregnancy? A. Nitrofurantoin B. Ciprofloxacin C. Penicillin A, is B Explanation: not recommended to use in pregnancy or breastfeeding unless necessary. Source: uptodate 243. What is the highest diagnostic yield in cervical cancer? A. Pap smear B. Colposcopy C. Cervical biopsy A, is C Explanation: (DNA) testing is much more sensitive than cervical cytology, but less specific. It is presently being investigated as a primary screening test for women after the age of 25 to 30 in many developed countries. Source: hacker and Moore p430 244. PCOS patient wants to get pregnant, how to manage her? A. OCPs B. Metformin, weight loss, and Danazol C. Metformin, weight loss, and Clomiphene A, is C Source: hacker and Moore p 392, BMJ 245. Contraindicated vaccines in pregnancy: A. Meningococcal B. Varicella C. Influenza A, is B Source: hacker and Moore p 11, uptodate 246. question about PCOS asking about investigation step by step self-study Explanation: refer to hacker and Moore p 390 247. Image of complete mole self-study Explanation: hacker and Moore p 465 248. Image of Vulvar lesion that bleeds with minimal trauma. (I believe it was cancer) self-study Explanation: hacker and Moore p 449 249. If pap smear is positive. What to do next? A, is colposcopy and or biopsy Explanation: positive Pap smear isn't enough to establish a diagnosis and start treatment. Source: uptodate 250. married woman's last pap smear was done before 3 years and it was normal, when to do it again? A, is this year Explanation: Pap smear from age 21 irrespective of sexual activity: age 21-29, every 3 yr

overall standardized incidence ratio 2.56, 95% CI 1.98-3.27) Source: uptodate, Kaplan step2 p 225 182. 59 years old female came for a checkup. She stated that she had her routine gynecological checkup 12 years ago when she was still pre-menopausal, and it showed fibroid. It was small( I do not remember the size) and it wasn't treated. Now when you examined her, the fibroid increased in size, and you found endometrial thickening( I forgot the exact sizes). What is the most likely diagnosis? A. Leiomyoma B. Leiomyosarcoma C. Metastasis from ovarian cancer D. Endometrial cancer

is B Explanation: fibroids and leomyosarcomas present with the same signs and symptoms

semen analysis for the husband, and literal phase progesterone, and urinary lutuinising hormone, and transvaginal ultrasound. PID can cause permanent injury to the fallopian tube, especially the endosalpinx. Changes to the fallopian tube, including loss of ciliary action, fibrosis, and occlusion, lead to tubal infertility. Thus I think the question targets these changes that you can detect by hysterosalpingogram. Source: uptodate, bmj best practice 141. Picture of CTG showing late decelerations and in the history the mother is in active labor and cervix is 6 cm dilated for a long time on oxytocin. There are signs of fetal distress what to do: A. C section. B. Stop oxytocin

is B Explanation: the proper intervention is guider by the category of FHR interpretation system, which in this case rather category 2 or 3. Category 2 management

now she is pregnant. What will you do? A. Discontinue paroxetine because will lead to preterm B. Discontinue paroxetine because will cause congenital malformation C. Continue paroxetine and follow up D. Continue paroxetine and add lithium The

is: C Explantion: Most studies indicate that SSRIs as a group are not major teratogens and are not associated with birth defects. In studies that have observed an association between SSRI exposure and congenital anomalies, the magnitude of the increased risk was small. Source: https://www.uptodate.com/contents/antenatal-use-of-antidepressants-and-risk-of- teratogenicity-and-adverse-pregnancy-outcomes-selective-serotonin-reuptake-inhibitors- ssris?search=paroxetine%20pregnancy&source=search_result&selectedTitle=2~148&usage_t ype=default&display_rank=1#H260269969 271. Vulvar lesion in situ and cancer

no growth on culture? A. BV B. Normal physiological discharge C. Trichomonas D. Chlamydia The

is: B Explanation: Estrogen stimulation of the vaginal mucosa produces a thickened, dull-pink mucoid surface. A thin, white, nonfoul-smelling liquid discharge (leukorrhea) normally is present in the otherwise asymptomatic patient. Source:https://www.uptodate.com/contents/the-pediatric-physical-examination-the- perineum?sectionName=Preadolescent%20and%20adolescent%20females&topicRef=5849& anchor=H10&source=see_link#H10 307. Case of endometrial cancer with pt has risk factor and stop progesterone. The A, is: N/A Explanation: Question not clear please referee to chapter 41 in Hacker Moore, sixth edition.. 308. Ectopic pregnancy and the Beta HCG is 5000 what to do? A. Laparoscopic surgery B. Open surgery C. Medication The A, is: A Explanation: Laparoscopic surgery is the standard surgical approach for ectopic pregnancy. Most ectopic pregnancies, even in the presence of hemoperitoneum, may be treated using a laparoscopic procedure. However, for patients with acute bleeding, some surgeons prefer laparotomy. The choice of surgical approach should be made by the surgeon with consultation from the anesthesiologist and by taking into consideration the clinical status of the patient. Source: https://www.uptodate.com/contents/ectopic-pregnancy-surgical- treatment?search=Ectopic%20pregnancy&source=search_result&selectedTitle=5~150&usag e_type=default&display_rank=5 309. .Ectopic pregnancy size 2*3 & beta HCG was less than 5000 whats the treatment? A. Laparoscopy B. MTX The A, is: B Explanation: MTX is the preferred treatment option when all of the following characteristics are present: • Hemodynamic stability. • Serum beta-human chorionic gonadotropin (hCG) concentration ≤5000 milli- international units/mL. • No fetal cardiac activity detected on transvaginal ultrasound (TVUS). Ectopic mass size less than 3 to 4 cm is also commonly used as a patient selection criterion. • Patients are willing and able to comply with post-treatment follow-up and have access to emergency medical services within a reasonable time frame in case of a ruptured fallopian tube. Source:https://www.uptodate.com/contents/ectopic-pregnancy-choosing-a- treatment?search=Ectopic%20pregnancy&source=search_result&selectedTitle=2~150&usag e_type=default&display_rank=2#H2976630177 310. Old female with insulin resistance small cysts on ovary high androgen? A. stein Leventhal syndrome (PCOS) the A, is: A explanation: Rotterdam criteria), PCOS is defined by the inclusion of at least two of the following three features: (1) clinical or biochemical hyperandrogenism, (2) oligomenorrhea or amenorrhea, and (3) polycystic ovaries. Source: Hacker Moore Page 390, sixth edition 311. Is breast feeding safe in hep B +ve mother and vaccinated and immunized newborn? A. yes the A, is: A Explanation: Transmission of HBV through breastfeeding is unlikely, particularly in infants who received HBIG and hepatitis B vaccine at birth. Although HBV DNA has been detected in the colostrum of HBsAg-positive mothers, a study of 147 infants born to carrier mothers revealed no evidence for a relationship between breastfeeding and the subsequent development of chronic HBV infection in the babies. Source:https://www.uptodate.com/contents/hepatitis-b-and- pregnancy?search=breastfeeding%20hepatitis%20b&source=search_result&selectedTitle=1~ 150&usage_type=default&display_rank=1#H50248214 312. female with breech orientation and bicornate uterus? A. Cs B. Ventouse C. spontaneous the A, is A


Ensembles d'études connexes

CPP (commercial package policy) (Ch 7)

View Set

Invasives - Nutrition and Elimination

View Set

MCB 150 Study Set Post Adaptive Follow ups EXAM 1

View Set

Chapter 13: Equality and Discrimination

View Set

Calcitonin And parathyroid gland

View Set